capital gains [income tax]

110
Capital Gains [INCOME TAX] Assessment Year 2016-17 Complete coverage of tax provisions for PGBP in Question & Answer format with practical illustration Compilation of CMA Suggested Answers [December 13 to June 15] CA IPCC Suggested Answers [May 98 to November 15] T K SRIDHAR

Upload: sridhartks

Post on 06-Jul-2016

144 views

Category:

Documents


10 download

DESCRIPTION

Applicable for AY 2016-17Covers1. Provisions and Computations of Capital Gains2. Past years CA-IPCC question papers upto November 2015 and CMA-Inter question papers upto June 20153. Bare Act related to Capital Gains

TRANSCRIPT

Page 1: Capital Gains [Income Tax]

Capital Gains

[INCOME TAX] Assessment Year 2016-17

Complete coverage of tax provisions for PGBP

in Question & Answer format with practical illustration

Compilation of

CMA Suggested Answers [December 13 to June 15]

CA IPCC Suggested Answers [May 98 to November 15]

T K SRIDHAR

Page 2: Capital Gains [Income Tax]

Price: ₹130

CALL OR VISIT FOR COPIES

Published by

SINGAR BOOKS AND PUBLICATIONS

A unit of SINGAR ACADEMY

Head Office: 52, Salai Road, Woriur, Trichy 620 003, Tamil Nadu

Branch Office: 76/1, New Street, Valluvar Kottam High Road,

Nungambakkam, Chennai – 600 034

Ph: Trichy: 0431-4024489 | 93451 22645 | 93446 04489 & Chennai: 93453 96855

www.singaracademy.com | [email protected]

Page 3: Capital Gains [Income Tax]

10. CAPITAL GAINS

Content

Section Important Definitions in Capital Gains Page

2(14) Capital Asset (CA)

2(29A) Long term Capital Asset (LTCA)

2(29B) Long term capital gains (LTCG)

2(42A) Short Term Capital Asset (STCA)

2(42B) Short Term Capital Gains (STCG)

2(42C) Slump sale

2(47) Transfer

2(48) Zero Coupon Bond

Content

Section Important Exemptions in Capital Gains Page

10(33) Exemption of capital gain on transfer of US64

10(36) Exemption in respect of long-term capital gains on transfer of listed shares

10(37) Exemption on compulsory acquisition of agriculture land

10(38) Exemption of LTCG from listed equity shares under head capital gain

10(41) Exemption of capital gain for power sector companies

10(43) Exemption of reverse mortgage

S Content Page

45(1) General computation of short term and long term capital gains

45(1A) Insurance claim received on loss of assets

45(2) Conversion of capital asset into stock-in-trade. (Note: indexation based on year of

conversion, not on year of sale)

45(2A) Sale of shares held as depository (FIFO method shall be adopted)

45(3) Introduction of capital asset by a partners into the firm or by a member into the AOP or

BOI

45(4) Distribution of capital asset to partners / members on dissolution of firm / AOP / BOI

45(5) Compulsory acquisition of capital asset by Govt. / Approved authority

(a) Normal Compensation and (b) Enhanced Compensation

Page 4: Capital Gains [Income Tax]

45(6) Receipt of Assets / Cash from company on liquidation in the hands of shareholder

46 Capital gains on distribution of assets by companies in liquidation

46A Capital gains on purchase by company of its own shares or other specified securities

47 Transactions not regarded as transfer

47A Withdrawal of exemption in certain cases

48 Mode of computation

49 Cost with reference to certain modes of acquisition

50 Special provision for computation of capital gains in case of depreciable assets

50A Transfer of Depreciable Assets by Power Sector Units

50B Sale or undertaking as a going concern or Slump Sale

50C Transfer of Land or Building or both at less than stamp duty value

50D If consideration received or accruing as a result of transfer of a capital asset is not

ascertainable or cannot be determined

51 Forfeiture of Advance Money received in respect of unfructified transfer

54 Profit on sale of property used for residence

54B Capital gain on transfer of land used for agricultural purposes not to be charged in

certain cases

54D Capital gain on compulsory acquisition of lands and buildings not to be charged in

certain cases

54EC Capital gain not to be charged on investment in certain bonds

54F Capital gain on transfer of certain capital assets not to be charged in case of investment in

residential house

54G Exemption of capital gains on transfer of assets in cases of shifting of industrial

undertaking from urban area

54GA Exemption of capital gains on transfer of assets in cases of shifting of industrial

undertaking from urban area to any Special Economic Zone

54GB Capital gain on transfer of residential property

54H Extension of time-limit for acquiring new asset

55 Meaning of adjusted, cost of improvement and cost of acquisition for the purpose of S.48

& 49

55A Reference to Valuation Officer

Page 5: Capital Gains [Income Tax]

Capital Gains 10.1

10. CAPITAL GAINS

Cost inflation index for different previous years

PY CII PY CII PY CII PY CII

1981-82 100 1990-91 182 1999-00 389 2008-09 582

1982-83 109 1991-92 199 2000-01 406 2009-10 632

1983-84 116 1992-93 223 2001-02 426 2010-11 711

1984-85 125 1993-94 244 2002-03 447 2011-12 785

1985-86 133 1994-95 259 2003-04 463 2012-13 852

1986-87 140 1995-96 281 2004-05 480 2013-14 939

1987-88 150 1996-97 305 2005-06 497 2014-15 1024

1988-89 161 1997-98 331 2006-07 519 2015-16 1081

1989-90 172 1998-99 351 2007-08 551

IMPORTANT DEFINITIONS IN CAPITAL GAINS

Section Description Definition

2(14) Capital Asset (CA) Includes property of any kind except stock, personal effects, Rural

agricultural land etc. [Jewellery is a capital asset]

2(29A) Long term Capital

Asset (LTCA)

Not a short term capital asset

2(29B) Long term capital

gains (LTCG)

Capital gains arising from transfer of LTCA

2(42A) Short Term Capital

Asset (STCA)

Financial Capital Assets – held for not more than 12 months

Other Capital Assets – Held for not more than 36 months

2(42B) Short Term Capital

Gains (STCG)

Capital Gains arising from transfer of STCA

2(42C) Slump sale

2(47) Transfer Sale, Exchange, relinquishment, Extinguishment of rights, Compulsory

acquisition, Conversion of CA into stock in trade, Maturity / Redemption

of zero coupon bond, Part Performance of contract, enjoyment of

immovable property.

2(48) Zero Coupon

Bond

Bond issued by Infrastructure capital Company or Infrastructure Capital

Fund or Public Sector Company or w.r.e.f AY 2009-10, Scheduled Banks

on or after 01.06.2005 with no benefit before maturity or redemption

Page 6: Capital Gains [Income Tax]

Income Tax 10.2

IMPORTANT EXEMPTIONS IN CAPITAL GAINS

Section Important Provisions

10(33) Capital gain on transfer of US64

10(36) Exemption in respect of long-term capital gains on transfer of listed shares

10(37) Exemption on compulsory acquisition of agriculture land

10(38) Exemption of LTCG from listed equity shares under head capital gain

10(41) Exemption of capital gain for power sector companies

10(43) Exemption of reverse mortgage

S Content Computation

45(1) Computation of capital gain CG = SC – CA or ICA YOT

45(1A) Insurance claim received on loss of assets CG = CR – CA or ICA YOR

45(2) Conversion of capital asset into stock-in-trade.

(Note: indexation on year of conversion)

CG = FMV – CA or ICA;

BI = SC – FMV

YOTCS

45(2A) Sale of shares held as depository

(FIFO method shall be adopted)

CG = SC – CA or ICA YOT

45(3) Introduction of capital asset by a partner

(member) into the firm (AOP or BOI)

CG = BV – CA or ICA YOI

45(4) Distribution of capital asset to partners

(members) on dissolution of firm (AOP or BOI)

CG = FMV – CA or ICA YOD

45(5) Compulsory acquisition of capital asset by Govt.

/ Approved authority

(a) Normal Compensation

(Indexation upto year of compulsory acquisition)

CG = NCR – CA or ICA YORI

(b) Enhanced Compensation CG = ECR - Expenses incurred

(c) Reduction of enhanced compensation Re-compute (b)

45(6) Repurchase of units (80CCB) by company CG = BBP – CA YOR

46 Capital gains on distribution of assets

by companies in liquidation

FMV of assets received ××

+ Cash received ××

- Dividend (S.2(22)(c)) ××

- CA or ICA ××

CG ××

YOR

46A Buy back of shares or specified securities CG = BBP – CA or ICA YOB

BBP – Buy Back Price

Page 7: Capital Gains [Income Tax]

Capital Gains 10.3

BV – Book Value

CA – Cost of Acquisition

CG – Capital Gain

CI – Cost of Improvement

CR – Claim Received

ECR – Enhanced Compensation Received

ICA – Indexed Cost of Acquisition

ICI – Indexed Cost of Improvement

NCR – Normal Compensation Received

SC – Sale Consideration

YOD – Year of Distribution

YOI – Year of Introduction

YOR – Year of Receipt

YORB – Year of Repurchase or Buyback

YORI – Year of Receipt of I installment

YOT – Year of Transfer

YOTCS – Year of Transfer of Converted Shares

S

47 Transactions not regarded as transfer

47A Withdrawal of exemption in certain cases

48 Mode of computation

49 Cost with reference to certain modes of acquisition

S

50 Computation of capital gains in case of

depreciable assets

CG = SC less (Expenses on transfer + opening

WDV + additions during the year)

YOT

50A Transfer of Depreciable Assets by Power

Sector Units

CG = SC less Original CA YOT

50B Sale or undertaking as a going concern or

Slump Sale

CG = Lump SC Less Net Worth

(No indexation for LTCG)

YOT

50C Transfer of Land or Building or both at less

than stamp duty value

CG = Value determined by Stamp Duty

Authority less CA or ICA

YOT

50D If SC of capital asset is not ascertainable SC = FMV on the date of transfer

[from AY 2013-14]

51 Forfeiture of Advance Money received in

respect of unfructified transfer

The advance is reduced from Cost of

Acquisition / WDV/ FMV

Page 8: Capital Gains [Income Tax]

Income Tax 10.4

S

54 Profit on sale of property used for residence

54B Capital gain on transfer of land used for agricultural purposes not to be charged in certain cases

54D Capital gain on compulsory acquisition of lands and buildings not to be charged in certain cases

54EC Capital gain not to be charged on investment in certain bonds

54F Capital gain on transfer of certain capital assets not to be charged in case of investment in

residential house

54G Exemption of capital gains on transfer of assets in cases of shifting of industrial undertaking from

urban area

54GA Exemption of capital gains on transfer of assets in cases of shifting of industrial undertaking from

urban area to any Special Economic Zone

54GB Capital gain on transfer of residential property

54H Extension of time-limit for acquiring new asset

55 Meaning of adjusted, cost of improvement and cost of acquisition for the purpose of S.48 & 49

55A Reference to Valuation Officer

[CMA RTP D11][CS EP D09 & J12, 5 marks & 3 marks]

Question: What are capital assets? What items are not included in capital assets [S.2(14)]?

Answer: Capital assets mean property of any kind held by the assessee, whether or not connected with

his business or profession, but does not include.

Exclusion 1 Any stock in trade, consumable stores or raw materials held for the purpose of business or

profession.

Exclusion 2 Personal effects of the assessee. It means moveable property including wearing apparel

and furniture held for his personal use or for the use of any member of his family

dependent upon him but exclude (a) Jewellery (b) Painting (c) Archaeological Collection

(d) Drawing (e) Sculptures (f) any work of art even though it is meant for personal use of

assessee

Exclusion 3 Rural agricultural land in India.

Exclusion 4 6½% Gold Deposit Bonds, 1977 or 7% Gold Bonds, 1980 or National Defense Gold Bonds,

1980 issued by Central Government.

Exclusion 5 Special Bearer Bonds, 1991

Exclusion 6 Gold Deposit Bonds issued under Gold Deposit Scheme, 1999

Important Notes:

1. "Jewellery" is a capital asset. For the purpose of this chapter, Jewellery includes the following:-

Page 9: Capital Gains [Income Tax]

Capital Gains 10.5

(a) Ornaments make of silver, gold, platinum, or any other metal or any alloy containing one or more of

such precious metal, whether or not containing any precious or semi-precious stones and whether or

not worked or sewn into any wearing apparel.

(b) Precious or semi-precious stones, whether or not set in any furniture, utensil of other article or

worked or sewn into any wearing apparel.

2. Agricultural land situated in rural area is not a capital asset: For this chapter, rural area means the

land situated:

(a) In any area within the territorial jurisdiction of a municipality having a population of 10,000 or more;

(b) In any area specified by the government.

3. Examples of personal effect are furniture, car, scooter etc. Silver bars, rupee coins for pooja, festivals

etc is capital asset and not personal effect.

4. Personal effects of movable assets are not capital assets are not a capital asset. However personal

effects of immovable assets are capital assets and hence capital gain arises on the sale of residential

houses etc.

[CMA RTP J12 & D12]

Question: Ritu received a gift, from her mother, 6 ½ % Gold Bonds of the value of ₹5 lakhs in 1980. These

bonds were redeemed by the Government on 1.10.2003 and he received gold of equivalent value,

weighting 5,000 grams approximately of fair market value of ₹10 lakhs. The gold was sold by him on

1.7.2015 for ₹24 lakhs. Examine the impact of the transactions in Ritu’s assessment.

Answer: Vide Circular No.415 dated 14.3.1985 [152 ITR (St.) 205], exchange of gold bonds for gold on

redemption does not attract Capital gains because it is not a capital asset. In case of subsequent sale of

such gold, capital gain is chargeable to tax. For this purpose:

Date of acquisition = date of redemption of such gold bonds;

Cost of acquisition of gold = market value of the gold on the date of redemption.

Computation of Capital Gains on sale of gold ₹

Consideration for transfer of gold 24,00,000

Less Indexed Cost of Acquisition [₹10,00,000 × 1081/463] 23,34,773

Long Term Capital Gains 65,227

[CMA RTP J12 & D12]

Question: A farmer resident of Bikaner sold his rural agricultural land in Nepal and received ₹5 lacs

over the cost of acquisition of this land. Explain the taxability of sale.

Answer: U/s 2(14), only rural agricultural lands in India are not a capital asset. In this given case, the

farmer has sold rural agricultural lands in Nepal and therefore, the transaction attracts the provisions of

capital gains.

Page 10: Capital Gains [Income Tax]

Income Tax 10.6

[CMA RTP J12 & D12]

Question: A Plantation company, holding several acres of land, sold trees of spontaneous growth. The

Assessing officer is of the opinion that the capital gains arises. Discuss

Answer: Sale proceeds of spontaneous growth will not result in capital gains, as they do not bring in any

profit or gain [Suman Tea & Plywood Industries Pvt. Ltd (1997) 226 ITR 34 (SC)].

Question: Write short note on long term capital asset as defined u/s 2(29A)?

Answer: A capital asset which is not a Short Term Capital Asset is a Long term Capital Asset. In other

words, if the asset is held for more than 36 months or 12 months, as the case may be, such an asset will be

treated as Long Term Capital Assets.

Question: Write Short note on Short Term Capital Assets as defined u/s 2(42A)?

Answer: A capital asset held by an assessee for not more than 36 months immediately preceding the date

of its transfer is known as a short term capital asset.

However, the following assets shall be treated as short term capital assets if the period held is not more

than 12 months immediately preceding the date of its transfer.

a. Equity or Preference shares in a company (listed in a recognized stock exchange in India) (but up to

10th July 10, 2014, whether shares are quoted or not is considered)

b. Any other securities listed in the recognised stock exchange in India.

c. Units of UTI or Units of an equity oriented mutual fund (whether quoted or not) (but up to 10th July

10, 2014 only Mutual Funds specified u/s 10(23D) (whether quoted or not is eligible) was considered)

d. Zero Coupon Bonds whether quoted or not.

Zero Coupon Bonds means notified bond issued by any infrastructure capital company or public sector

company, in respect of which no benefit is receivable before maturity.

Question: Write Short note on Capital Gains?

Answer: Since there are two types of capital assets, there will be two types of capital gains.

a. Short Term Capital Gain as defined u/s 2(42B) of the Income Tax Act, 1961-It implies that any profit

or gains arising from the transfer of short term capital assets is called short term capital gain.

b. Long Term Capital Gain as defined u/s 2(29B) of the Income Tax Act, 1961-it implies that any profit or

gains arising from the transfer of long term capital assets is called long capital gain.

[CMA RTP J12 & D12]

Question: Well Wishers & Associates, a partnership firm, is holding land. This firm is not engaged in real

estate business. The land was sold during the year. Discuss taxability, whether, this would be assessed to

tax as business income or capital gain.

Page 11: Capital Gains [Income Tax]

Capital Gains 10.7

Answer: Land held by partnership firm, which is not engaged in real estate business, would be treated as

fixed asset of the firm. Transfer of the same is assessable as capital asset, hence capital gains and not as

business income. [Mohakampur Ice & Storage 281 ITR 354 (All.)]

Question: Define slump sale

Answer: S.2(42C) "slump sale" means the transfer of one or more undertakings as a result of the sale for a

lump sum consideration without values being assigned to the individual assets and liabilities in such

sales.

[CMA RTP J12 & D12]

Question: Explain the tax treatment of income from Deep Discount Bonds (DDBs).

Answer: Deep discount bonds as clarified vide Circular No. 2/2002 as follows:

1. Income based on market value

i. Income treated as interest for investors

ii. Income treated as business income for traders

2. For original subscribers, Income = difference between market value on 31st March of the previous year

and 1st April of the previous year

For subsequent purchases, income = difference between market value on 31st March of the previous

year and cost of purchase of the bond

3. If there is a transfer before maturity:

i. For the Investor, Short term Capital Gains = Sale Price less Cost of Bond;

ii. For Traders, Business Income = Sale price less cost of bond.

4. Cost of bond = Cost of acquisition + Income already taxed up to the date of transfer.

5. If there is a redemption on maturity:

i. For the Investor, Interest Income = Redemption Price less market value as on the last valuation

date, immediately preceeding the maturity date.

In case of a trader, this interest income would be construed as Business Income.

ii. For subsequent purchasers, Interest Income = Redemption price less cost of the bond to such

purchaser.

In case of a trader, this interest income would be construed as Business Income.

Where, Cost of bond = cost of acquisition + income already taxed by the bond holder up to the

date of redemption

[CS INTER G1, D00, 5 marks]

Question: Write short note on definition of transfer u/s 2(47).

Answer: "Transfer" in relation to capital assets includes:-

Page 12: Capital Gains [Income Tax]

Income Tax 10.8

a. The sale, exchange, or relinquishment of the asset; or

b. The extinguishment of any rights therein; or

c. The compulsory acquisition thereof under any law; or

d. In a case where the asset is converted by the owner thereof into, or is treated by him as stock-in-trade

of a business carried on by him, such conversion or treatment; or

e. Any transaction involving the allowing of the possession of any immovable property to be taken or

retain in the part performance of the contract as referred in Section 53A of the Transfer of Property

Act, 1882.

f. Allotment or lease under a housing building scheme of society, company or other association.

g. The maturity or redemption of Zero Coupon bonds.

Important Points:

a. Surrender of preference shares on redemption thereof amount to 'transfer' as there is

relinquishment by the shareholder of his rights in preference shares. Decided in the case of Anarkali

Sarabhai (SC)

b. Reduction of face value of share and consequent payment to the shareholder towards such

reduction amount to transfer as it results in extinguishments of right in the shares held by the

shareholder. Decided in the case of Kartikeya Sarabhai (SC)

Question: When does the transfer complete and effective?

Answer:

1. Movable property: title to a movable property passes at the time when property is delivered

pursuant to a contract to sell. Entries in the books of account are not relevant for determining date of

transfer.

2. Immovable property:

(a) immovable property when documents are registered: transfer is compete on registration.

(b) immovable property when documents are not registered: transfer is complete on satisfaction of

conditions laid u/s 53A of the Transfer of Property Act, which are listed below.

1. there should be a contract in writing

2. the transferee has paid consideration or is willing to perform his part of the contract and

3. the transferee should have taken possession of the property.

[CA INTER N06, 1 Mark]

Question: Fill up the blanks: As per section 2(47), ______ or ________ of a zero coupon bond will be

treated as ‚transfer’‛ for the purpose of capital gains tax. 1

1Answer: maturity, redemption.

Page 13: Capital Gains [Income Tax]

Capital Gains 10.9

[CMA INTER SY12,D13, 2 Marks]

Question: Is the right of management in an Indian company a capital asset? On relinquishment directly

or indirectly, is it liable to tax?

Answer: Yes, it is a capital asset in view of the Explanation to section 2(14) inserted by the Finance Act,

2012. It is a 'transfer' in view of Explanation 2 to section 2(47). Hence the relinquishment is liable to capital

gains tax.

[CMA RTP J12 & D12]

Question: Amit owns a plot of land acquired on 1.7.2005 for a consideration of ₹4 lakhs. He enters into an

agreement to sell the property on 23.3.2016 for a consideration of ₹11 lakhs. In part performance of the

contract, he handed over the possession of land on 25.3.2016 on which date, he received the full

consideration. As on 31.3.2016, the sale was pending registration. Discuss liability of capital gains for the

assessment year 2016-17 (no computation is required)

Answer: U/s 2(47), transfer includes part performance of a contract of the nature specified in S.53A of the

Transfer of Property Act. In the given case, consideration was received by Amit and the possession was

handed over on 25.3.2016 hence, the part performance condition is satisfied. Capital gain on the above

transaction is chargeable to tax as income for the assessment year 2016-17.

[CS EP D09 & J10, 3 Marks & 3 Marks]

Question: Write Short note on taxability of Zero Coupon Bonds?

Answer: Meaning of Zero Coupon Bond: Section 2(48) defines the expression Zero Coupon Bond as

bond issued by any infrastructure capital company or infrastructure capital fund or a public sector

company on or after 1st June, 2005, in respect of which no payment and benefit is received or receivable

before maturity or redemption from such issuing entity.

Tax Implications: The income of transfer of a ZCB (not being held as stock-in-trade) is to be treated as

capital gains. S.2(47)(iva) provides that maturity or redemption of ZCB shall be treated as a transfer for

the purposes of capital gains tax

Nature of Capital Gain: ZCBs held for not more than 12 months would be treated as short term capital

assets. Where the period of holding of ZCBs is more than 12 months, the resultant long term capital gains

arising on maturity or redemption would be treated in the same manner as applicable to capital gains

arising from the transfer of other listed securities or units covered by section 112.

Question: Explain capital gain on transfer of US64 [S.10(33)]

Answer: Any income arising from the transfer of a capital asset being a unit of US64 is not chargeable to

tax where the transfer of such assets takes place on or after April 1, 2002. This rule is applicable whether

the capital asset (US64) is long term capital asset or short term capital asset.

Page 14: Capital Gains [Income Tax]

Income Tax 10.10

[CA INTER N04, 4 Marks]

Question: State the provisions relating the exemption in respect of long-term capital gains on transfer

of listed Equity Shares.

Answer: Section 10(36), inserted by Finance Act, 2003 w.e.f. A.Y. 2004-05, provides that any income

arising from the transfer of a long-term capital asset, being an eligible equity share in a company shall be

exempt provided that these were acquired on or after 1.3.2003 but before 1.3.2004 and held for a period of

12 months or more; Eligible equity share means:

(i) Any equity in a company being a constituent of BSE-500 Index of the Stock exchange. Mumbai as on

the 1.3.2003 and the transactions of purchase and sale of such equity share are entered into on a

recognized stock exchange in India.

(ii) Any equity share in a company allotted through a public issue on or after the 1.3.2003 and listed in a

recognized stock exchange in India before 1.3.2004 and the transaction of sale of such share is entered

into or a recognizes stock exchange in India.

Question: Write Short note on exemption on compulsory acquisition of agriculture land u/s 10(37)?

Answer: In the case of an individual or HUF, an income from the transfer of urban agricultural land shall

be exempt if

(a) such land was being used for agricultural purposes by such Individual or HUF or his parents, during

two years immediately preceding the date of transfer.

(b) transfer is by way of compulsory acquisition under any law, or consideration is determined by

Central Government or RBI

(c) such income has arisen from the compensation received by assessee on or after 1st April, 2004.

[CA INTER M06, 4 Marks]

Question: X is in possession of agricultural land situated within urban limits, which is used for

agricultural purposes during the preceding 3 years by his father. On 4.4.2015 this land is compulsorily

acquired by the Government of India on a compensation fixed and paid by it ₹10 lakhs. Advise X as to the

tax consequences, assuming that the entire amount is invested in purchase of shares.

Answer: Capital gain of ₹10 lakhs arising on compulsory acquisition shall be exempt u/s 10(37).

[CA INTER N05, 2 Marks]

Question: State whether True or False: Where an urban agriculatural land owned by an individual,

continuously used by him for agricultural purposes for a period of two years prior to the date of transfer,

is compulsorily acquired under law and the compensation is fixed by the State Government, resultant

capital gain is exempt.

Answer: False. As per Section 10(37), where an individual owns urban agricultural land which has been

used for agricultural purposes for a period of two years immediately preceding the date of transfer, and

the same is compulsorily acquired under any law and the compensation is determined or approved by

Page 15: Capital Gains [Income Tax]

Capital Gains 10.11

the Central Government or the Reserve Bank of India, resultant capital gain will be exempt. In this case,

however, the compensation has been fixed by the State Government and hence the exemption will not be

available.

Question: Write Short note on exemption of LTCG from listed equity shares u/s 10(38) under head

capital gain?

Answer: Long term capital gain from transfer of equity shares or units of equity - oriented mutual fund

(where funds are invested in equity shares in domestic company, more than 65% of total proceeds of

fund) shall be exempt if -

i. They are sold through recognized stock exchange.

ii. Security transaction tax is chargeable on such transaction.

iii. Transfer took place on or after 01.10.2004

"Equity oriented fund" means a setup under mutual fund scheme specified u/s 10(23D) and whose

investible funds are invested in the equity shares in the domestic companies to the extent of more than

65% of the total proceeds of such fund.

[CA INTER N09, 3 Marks][CMA RTP J12]

Question: Write short note on reverse mortgage? [S. 10(43)]

Answer: Exemption of income received in a transaction of reverse mortgage [S.10(43)]. This scheme is for

the benefit of senior citizens, who own a residential house property. In order to supplement their existing

income, they can mortgage their house property with a scheduled bank or housing finance company, in

return for a lump-sum amount or for a regular monthly / quarterly / annual income.

The senior citizens can continue to live in the house and receive regular income without the botherations

of having to pay back the loan. The loan will be given up to say, 60% of the value of residential house

property mortgaged. Also, the bank / housing finance company would undertake a revaluation of the

property once every 5 years. The borrower can use the loan amount for extension of residential property,

family's medical and emergency expenditure etc. amongst other. However, he cannot use the amount for

speculative or trading purposes. The bank will recover the loan along with the accumulated interest by

selling the house after the death of the borrower. The excess amount will be given to the legal heirs.

However, before resorting to sale of the house, preference will be given to the legal heirs to repay the loan

and interest and get the mortgaged property released.

[CA INTER M09, 3 Marks]

Question: Mr. Abhik’s father, who is a senior citizen had pledged his residential house to a bank under a

notified reverse mortgage scheme. He was getting loan from bank in monthly installments. Mr. Abhik’s

father did not repay the loan on maturity and given possession of the house to the bank to discharge his

loan. How will the treatment of long-term capital gain be made on such reverse mortgage transaction?

Answer: Reverse Mortgage - At the time of pledging the property to the bank, capital gain is not

chargeable to tax. There is no capital gain tax liability in the hands of Abhik’s Father when bank gives

Page 16: Capital Gains [Income Tax]

Income Tax 10.12

loan or monthly installment. When possession of the property is given to the bank, there is no transfer

and, consequently, capital gain is not chargeable to tax. However, when bank transfers the property to

realize the amount of loan and interest, capital gain will be taxable as the transaction is treated as

‚transfer‛. If this transfer takes place during the lifetime of Abhik’s Father (generally in the case reversed

mortgage, bank transfers the property after the death of the person who has pledged the property),

capital gain would be taxable in the hand of Abhik’s Father.

If transfer by the bank takes place after the death of Abhik’s father then capital gain will be taxable in the

hands of legal heirs of Abhik’s father.

Question: Write short note on method of calculation of capital gains u/s 48?

Answer:

Computation of Short Term Capital Gain Computation of Long Term Capital Gain

Consideration Received ××× Consideration Received ×××

Less Expenses of Transfer ××× Less Expenses of Transfer

Net Consideration ××× Net Consideration ×××

Less Cost of Acquisition ××× Less Indexed Cost of Acquisition ×××

Less Cost of Improvement ××× Less Indexed Cost of Improvement ×××

Short Term Capital Gain ××× Long Term Capital Gain ×××

Less Exemption u/s 54B, 54D, 54G, 54GA ××× Less Exemption u/s 54 to 54GA ×××

Taxable STCG ××× Taxable LTCG ×××

Question: Write short note on concept of indexation in case of long term capital gains?

Answer: If the capital gain arises from the transfer of a long term capital asset, then for the purpose of

computing capital gains:

i. 'Indexed Cost of Acquisition' is taken instead of 'Cost of Acquisition (COA)'.

ii. 'Indexed Cost of Improvement' is taken instead of 'Cost of Improvement (COI)'.

Note 1: No deduction will be allowed in respect of payment of securities transaction tax in computing

income under the head capital gains.

Note 2: COI is always indexed in case of long term capital asset irrespective of year in which such

improvement was incurred.

Page 17: Capital Gains [Income Tax]

Capital Gains 10.13

Note 3: Indexation of cost shall not be allowed in case of transfer of bonds & debentures other than

capital indexed bonds issued by the Government.

In the following cases benefit of indexation is not allowed:-

Capital Assets

Bond or Debentures (Other than capital indexed bond issued by Government)

Share or Debenture of Indian Co. acquired by utilizing convertible foreign exchange (first proviso of

section 48)

Depreciable assets

Slump Sale under Selection 50B

Transfer of units purchased in foreign currency by an assessee covered u/s 115AB

Transfer of GDR's purchased in foreign currency by an assessee covered u/s 115ACA

Transfer of Securities by Foreign Institutional Investors (FII) u/s 115AD

Question: Write Short note on cost of improvement under head capital gain?

Answer: Cost of improvement incurred before 01.04.1981 shall be ignored for all cases.

All expenditure incurred on improvement of assets by the assessee or the previous owner on or after

01.04-1981 shall be treated as cost of improvement.

[CMA RTP J12]

Question: What conditions must be satisfied for the purpose of taxing capital gain u/s 45(1)?

Answer: Any profits or gains arising from the transfer of capital assets effected in the previous year, shall

be chargeable to income tax under the head 'capital gain' and shall be deemed to be the income of the

previous year in which the transfer took place unless such capital gain is exempt u/s 54, 54B, 54D, 54EC,

54ED, 54F, 54G and 54GA

In other words, capital gains tax liability arises only when the following conditions are satisfied

1 There should be Capital Asset.

2 The capital asset is transferred by the assessee.

3 Such transfer takes place during the previous year

4 Any profit or gain arises from the result of transfer

5 Such profit or gain is not exempt from tax u/s 54, 54B, 54D, 54EC, 54ED, 54F, 54G and 54GA

All of the above conditions must be satisfied at a one point of time for the purpose of taxing capital gains.

[CMA INTER SY08, D13, 1 Mark]

Question: In case of transfer of a capital asset by an assessee who acquired the same from his father by

way of gift, the cost of acquisition of such asset for the purpose of computing capital gain is1

1 Answer: (b)

Page 18: Capital Gains [Income Tax]

Income Tax 10.14

a) Nil

b) Cost of acquisition of the asset to father

c) Fair market Value of the asset on the date of gift

d) Arm's length price of the asset on the date of gift

Question: Write short note on insurance claim received u/s 45(1A) under head capital gains?

Answer: If any person receives any money or other assets on account of insurance claim from the

insurance claim from the insurer on account of damage to, or destruction of, any capital asset from

Flood, typhoon, hurricane, earthquake & other convulsion of nature

Riot or civil disturbance

Accidental fire or explosion

Action by enemy or action taken in combating an enemy

Then any profit from the receipt of such money shall chargeable to capital gain and shall be deemed to be

the income of such person of the previous year in which such money or asset was received and money

received or FMV of the asset received, shall be deemed to be full consideration.

i. Full value of consideration = Money received or FMV of asset on date of receipt.

ii. Total period of Holding = Date of acquisition to date of destruction.

Capital Gain = Insurance Money Received or FMV of Asset minus COA, COI or ICOA and ICOI.

If compensation received for depreciable asset, then provision of S.50 shall apply.

[CA INTER N06, 7 Marks]

Question: Mr. A, is an individual carrying on business. His stock and machinery were damaged and

destroyed in a fire accident. The value of stock lost (totally damaged) was ₹6,50,000. Certain portion of

the machinery could be salvaged. The opening WDV of the block as on 1-4-2015 was ₹10,80,000.

During the process of safeguarding machinery and in the firefighting operations, Mr. A lost his gold

chain and a diamond ring, which he had purchased in April, 2009 for ₹1,20,000. The market value of these

two items as on the date of fire accident was ₹1,80,000.

Mr. A received the following amounts from the insurance company:

1 Towards loss of stock ₹4,80,000

2 Towards damage of Machinery ₹6,00,000

3 Towards gold chain and diamond ring ₹1,80,000

You are requested to briefly comment on the tax treatment of the above three items under the provisions

of the income-tax Act, 1961.

Answer: Tax Treatments:

Stock Machinery Gold & Diamond

Sale Consideration (Compensation) 4,80,000 6,00,000 1,80,000

Less Cost of acquisition 6,50,000

Less WDV of the block (assumed) 10,80,000

Page 19: Capital Gains [Income Tax]

Capital Gains 10.15

Less Indexed cost of acquisition *

+ 2,05,253

Business income (1,70,000)

Short-term capital gain (loss) (4,80,000)

Long-term capital gain (loss) [S.45(1A)] (25,253)

[CA INTER M00, M07 & N12, 5, 6 & 4 Marks]

Question: Discuss the tax implications arising consequent to conversion of a capital asset into

stock-in-trade of business and its subsequent sale.

[CS EP J10, 4 Marks]

Question: Explain with the help of suitable illustration how capital gains are computed u/s 45(2) in case

of conversion of capital asset into stock-in-trade

Answer: Capital Gain from conversion of capital asset into Stock in Trade shall be charged to tax in the

previous year in which the Stock in Trade is sold or otherwise transferred by the assessee

In the above case, FMV of the assets on the date of conversion of capital assets into stock in trade shall be

taken to the account.

Capital Gain = FMV of the asset on the date of transfer –Indexed cost of acquisition and

improvement.

PGBP = Sale consideration – FMV of the asset on the date of transfer.

Indexation shall apply on the basis of the year in which conversion takes place.

Section 45(2) is applicable only if the asset has been converted into stock-in-trade w.e.f. 01.04.1984

onwards. If the conversion is prior to 01.04.1984, no capital gains shall be computed as per Supreme

Court decision in Bai Shirinbai K.Kooka v. CIT (1962) 46 ITR 86 (SC).

[CA INTER M08, 5 Marks]

Question: Aarav converts his plot of land purchased in July, 2009 for ₹80,000 into stock-in-trade on

31st March, 2015. The fair market value as on 31.3.2015 was ₹1,90,000. The stock-in-trade was sold for

₹2,25,000 in the month of January, 2016.

Find out the taxable income, if any, and if so under which ‘head of income’ and for which

Assessment Year?

Answer:

Computation of taxable income of Mr. Aarav for A.Y.2016-17 ₹ ₹

1 Long term capital gains

Full value of consideration (Fair market value on the date of conversion) 1,90,000

Less Indexed cost of acquisition *

+ 1,29,620 60,380

2 Profits & Gains of Business or Profession

Sale price of stock-in-trade 2,25,000

Page 20: Capital Gains [Income Tax]

Income Tax 10.16

Less Fair market value on the date of conversion 1,90,000 35,000

95,380

Question: Write short note on dematerialization u/s 45(2A) under head capital gain?

Answer: Where any person had, at any time during the previous year, any beneficial interest in any

security, then any profit and gain arising from transfer made by depositary shall be chargeable to tax as

an income of the beneficial owner of the previous year in which the transfer took place.

Cost of Acquisition (COA) and period of holding (POH) shall be determined on the FIFO basis.

[CMA RTP J11]

Question: The depository account shows the following details of M’s holdings:

Date of

Credit

Particulars Quantity

10.11.2004 Shares of XYZ LTD. purchased in physical form on

10.11.2004 @ ₹20 per share

300

30.11.2005 Purchased dematerialised shares of Y Ltd. 500 on 25.11.2005 @ ₹70

per share

06.12.2007 Shares of XYZ LTD. held in physical form, were got dematerialised on 01.12.2007

M sold 600 dematerialised shares on 6th June 2015 @ ₹200 per share. Brokerage is paid @ 2% of sale price.

Compute capital gains.

Answer:

a) Person Liable: The sale of shares held under Dematerialized format with a depository is chargeable

to tax as the income of the beneficial owner.

b) Cost of Acquisition and period of holdings: The cost of acquisition and the period of holding shall

be determined on FIFO Method. [Circular No. 768 dated 24.6.1998]

i. FIFO method will be applied for each account independently.

ii. When physical stock is dematerialised, the date of credit into the depository account shall be

considered for the purpose of FIFO method, but indexed cost of acquisition shall be computed on

the basis of year of acquisition.

Consideration for Transfer ₹

600 Share @ ₹200 per share 1,20,000

Less : Brokerage 2% 2,400

Indexed Cost of Acquisition

(i) (500 × 70 × 1081 / 497) 76,127

Page 21: Capital Gains [Income Tax]

Capital Gains 10.17

(ii) (100 × 20 × 1081 / 551) 3,924 80,051

Long Term Capital Gain 37,549

Question: Write short note on transfer of capital asset by a partner to firm u/s 45(3) under head capital

gain?

Answer: Capital gain arising from the transfer of a capital asset by way of contribution to the firm or

otherwise, shall be chargeable to tax in the previous year in which such transfer takes place.

For the purpose of sale consideration, the amount recorded in the books of the firm as a value of capital

asset shall be taken

[CMA RTP D10]

Question: Vijay has three motor cars which are used by him exclusively for his personal purposes. The

cost of the cars was ₹6,50,000, ₹8,00,000 and ₹10,00,000. The first car was transferred by him on 15.1.2016

to firm in which he is a partner as his capital contribution. The market value of the car as on 15.1.2016 is

₹5,00,000, but it was recorded in the books of account of the firm at ₹6,00,000. Compute the capital gain if

any, chargeable for the AY 2016-17.

Answer: Since the car is a moveable property and was used by Mr. Ayan for his personal purposes only,

it will be treated as a personal effect.

W.e.f. A.Y. 2008-09, ‚Personal effect‛ means moveable property including wearing apparel and furniture

held for personal use by the assessee or any member of his family dependent on him but excludes:

(i) Jewellery (ii) Archaeological collections (iii) Drawings (iv) Paintings (v) Sculptures (vi) Any work or

art.

[CMA RTP J11]

Question: Nisith acquired a property by way of gift from his father in the year 1991-92 when its FMV was

₹3 lacs. His father had acquired the property during 1984-85 for ₹4 lacs. This property was introduced as

capital contribution to a partnership firm in which Nisith became a partner on 15.6.2015. The market

value of the asset as on that date was ₹40 lacs, but it was recorded in the books of account of the firm at

₹34 lacs. Is there any capital gain chargeable in the hands of Mr. Nisith?

Answer:

Computation of Capital Gains ₹

Consideration for Transfer 34,00,000

Less: Indexed Cost of Acquisition(4,00,000 × 1081/125) 34,59,200

Long Term Capital Loss 59,200

a) Full value of consideration is taken as the value at which it is recorded in the books of accounts of the

firm.

Page 22: Capital Gains [Income Tax]

Income Tax 10.18

b) Market value of the asset on the date of transfer is not relevant.

Question 17: Write short note on dissolution of firm u/s 45(4) under the head capital gain?

Answer: Capital Gains arising from the transfer of the capital assets by a firm at the time of dissolution of

the firm or otherwise, shall be chargeable to tax as an income of the firm in which such distribution takes

place. For the purpose of the sale consideration, FMV of the capital assets on the date of distribution shall

be taken.

Capital Gain = FMV on the date of transfer - Cost or Indexed COA

[CMA RTP J11]

Question: A firm consists of 3 partners X, Y & Z. Z retires from the firm on 15.10.2015. His capital balance

and the profits till the date of retirement stood at ₹16 lacs. The firm transferred its land to Z in settlement

of his account. The market value of the land as on that date was ₹30 lacs. The land was acquired by the

firm on 1.5.96 for ₹4 lacs. Compute the capital gains in the hands of the firm.

Answer:

Computation of Long Term Capital Gains for the A.Y. 2016-17 ₹

Consideration for Transfer 30,00,000

Less Indexed Cost of Acquisition (4,00,000 × 1081/305) 14,17,705

Long Term Capital Gains 15,82,295

[CMA RTP J11]

Question: PQR & Co. is a partnership firm, consisting 3 partners P, Q and R. the firm is dissolved on

31.12.15. The assets of the firm were distributed to the partners as under:

Particulars Block of machinery

(given to P)

Stock

(given to Q)

Land

(given to R)

Year of acquisition 1995-96 2007-08 1978-79

Cost of acquisition (₹) 7,20,000 4,00,000 10,000

Market value as on 31.12.15 15,00,000 6,00,000 30,00,000

WDV as on 31.12.15 10,40,000 — —

Value at which given to partners as per agreement 10,00,000 4,50,000 18,00,000

Market value as on 1.4.81 — — 2,70,000

Compute the income taxable in the hands of the firm for the assessment year 2016-17. What shall be the

cost of acquisition of such assets to the partners of the firm?

Page 23: Capital Gains [Income Tax]

Capital Gains 10.19

Answer:

Computation of Short Term Capital Gains on block of Machinery ₹

Sale consideration (i.e. the market value) 15,00,000

Less Cost of Acquisition (WDV of the block) 10,40,000

Short Term Capital Gains 4,60,000

Income from Business (on transfer of stock)

Market value of stock 6,00,000

Less Cost of Acquisition 4,00,000

Business Income 2,00,000

Computation of Capital Gains on transfer of Land ₹

Consideration for transfer 30,00,000

Less Indexed cost of Acquisition: ( 2,70,000 × 1081/100) 29,18,700

Long term capital gains 81,300

Cost of acquisition of assets to the Partners ₹

Partner“ P 10,00,000

Partner“ Q 4,50,000

Partner“ R 18,00,000

Question 18: Write short note on compulsorily acquisition u/s 45(5) u/h capital gain?

Answer: Where a Capital asset is compulsorily acquired by the Government of similar agency, under any

law then,

1. Capital gain shall be computed in the year in which the asset was acquired.

2. Capital gain for initial compensation / original compensation shall be taxable in the year in which

such compensation or part thereof is first received by the assessee.

3. Any additional compensation, increased compensation received by the assessee is taxable in the year

in which such additional amount is received.

4. For the purpose of additional compensation COA & COI Shall be zero.

5. Any expense incurred for the purpose of realization of the additional compensation shall be deducted

from the sale consideration.

6. Nature of additional compensation should be same as an initial compensation.

7. Reduction of compensation: Where original compensation / enhanced compensation is reduced by

court, then the Capital gain shall be recomputed accordingly.

Page 24: Capital Gains [Income Tax]

Income Tax 10.20

[CMA RTP D11, J12 & D12]

Question: Mr. B acquired a house property for ₹50,000 in 1969-70. On his death in October 1989 the house

was acquired by his son C. The market value of the house as on 01.04.1981 was ₹3,00,000. This house was

acquired by the Government on 15.3.2012 and a compensation of ₹32 lacs is paid to him on 25.3.2016. C

filed a suit against the Government challenging the quantum of compensation and the court ordered for

giving additional compensation of ₹14,00,000. He incurred an expenditure of ₹40,000 as expenditure in

connection with the suit. The additional compensation was received on 25.3.2017. Compute capital gains

chargeable to tax.

Answer:

Computation of Long Term Capital Gains for the A.Y. 2016-17 ₹

Consideration for transfer (being the compensation) 32,00,000

Less Indexed Cost of Acquisition(3,00,000×785/100) 23,55,000

Long Term Capital Gains / Loss 8,45,000

Computation of Long Term Capital Gains for the A.Y. 2017-18 ₹

Enhanced Compensation received 14,00,000

Less Cost of Acquisition NIL

Cost of Improvement NIL

Legal Expenses (40,000)

Long Term Capital Gains 13,60,000

Question: Assume from the previous problem, that the enhanced compensation is reduced by Supreme

Court from ₹14,00,000 to ₹10,00,000 on 7.7.2017 and the legal expenses incurred ₹20,000.

Answer:

Re-computation of Long Term Capital Gains for the A.Y. 2018-19 ₹

Revised enhanced compensation received 14,00,000

Less Cost of Acquisition NIL

Cost of Improvement NIL

Legal Expenses [40,000 + 20,000] 60,000

Long Term Capital Gains 13,40,000

Note: the assessing officer can re-compute the income of the AY 2018-19 within 4 years from the end of

the year in which order of the Supreme Court, reducing the compensation, is passed (i.e. 31.03.2023).

Question: State the provisions u/s 45(6) related to repurchase of units.

Answer: Capital gain = Repurchase price less capital value of the units

Note: capital value means the amount invested in the units referred by the assessee u/s 80CCB(2).

Page 25: Capital Gains [Income Tax]

Capital Gains 10.21

[CA INTER M02, 6 Marks]

Question: State the provisions relating to the computation of capital gains in the hands of shareholder

of a company on distribution of assets upon liquidation. (S.46)

Answer: In the hands of the company (S.46(1)): Where the assets of a company are distributed to its

shareholders on its liquidation, such distribution shall not be regarded as transfer by virtue of Section 47.

In the hands of the shareholders (S.46(2)): Where a shareholder on the liquidation of a company received

any money or other assets from the company, he shall be chargeable to income tax under this head for

such receipt and sale consideration of shares shall be

a. Distribution in Cash: Amount received less dividend u/s 2(22)(c).

b. Distribution in Kind: Fair market value (FMV) of assets on the date of liquidation less deemed

divided u/s 2(22)(c).

While computing the period of holding of such assets the period after the date of liquidation shall be

ignored.

Capital Gain on the sale of assets received at the time of Liquidation: The cost of acquisition of the asset

received in liquidation, shall be its fair market value on the date of distribution, without deducting

deemed dividend u/s 2(22)(c).

[CMA RTP D11]

Question: Define accumulated profits for a company under liquidation.

Answer: Accumulated profits for a company in liquidation includes all profits of the company upto the

date of liquidation.

Accumulated profits should include the credit balance of profit and loss account, general reserves,

investment allowance, capitalized profits and profits of the year upto the date of distribution/liquidation.

However, provisions and reserves meant for specific liability, to the extent of the liability shall not be

included. Provision for income tax, provision for dividend, reserve for depreciation do not form part of

the accumulated profits.

Securities premium is not accumulated profits.

It may consist of exempted incomes, like agricultural income.

It will include current profits and all profits of the company till the date of liquidation, subject to the

exception provided therein.

[CA INTER M08, 8 Marks]

Question: Ms. Vasumathi purchased 10,000 equity shares of Rajesh Co. Pvt. Ltd. on 28.2.2010 for

₹120,000. The company was wound up on 31.07.2015. The following is the summarized financial position

of the company as on 31.07.2015.

Liabilities ₹ Assets ₹

60,000 Equity shares 6,00,000 Agricultural lands 42,00,000

Page 26: Capital Gains [Income Tax]

Income Tax 10.22

General reserve 40,00,000 Cash at Bank 6,50,000

Provision for taxation 2,50,000

48,50,000 48,50,000

The tax liability (towards dividend distribution tax) was ascertained at ₹3,00,000 after considering refund

due to the company. The remaining assets were distributed to the shareholders in the proportion of their

shareholding. The market value of 6 acres of agricultural land (in an urban area) as on 31.07.2015 is

₹10,00,000 per acre.

The agricultural land received above was sold by Ms. Vasumathi on 29.2.2016 for ₹15,00,000.

Discuss the tax consequences in the hands of the company and Ms. Vasumathi.

Answer: In the hands of the company: As per section 46(1), in case of distribution of capital assets

amongst the shareholders on liquidation of the company is not regarded as ‚transfer‛ in the hands of the

company. Consequently, there will be no capital gains in the hands of the company.

In the hands of Ms. Vasumathi (shareholder)

Section 46(2) provides that such capital gains would be chargeable in the hands of the shareholder.

Ms. Vasumathi holds 1/6th of the shareholding of the company Amount (₹ )

Market value of agricultural land received (1 acre @ ₹10 lakhs) 10,00,000

Add Cash at bank (1/6th of ₹(6,50,000 – ₹3,00,000) 58,333

10,58,333

Less Deemed Dividend u/s 2(22) (c ) 1/6th of (₹40,00,000 – ₹50,000) 6,58,333

Consideration for computing Capital Gain 4,00,000

Less Indexed cost of acquisition of Shares

2,05,253

Long term capital gains 1,94,747

Sale consideration of agricultural land 15,00,000

Less Fair market value of the agricultural land on the date of distribution 10,00,000

Short term capital gain 5,00,000

Note: Dividend u/s 2(22) (c) ₹6,58,333 will be exempt under section 10(34).

Note: Since the question states that there is refund due to the company, it is assumed that the provision

for taxation of ₹250,000 shown in Balance Sheet is in respect of dividend distribution tax. Therefore, the

tax liability in respect of dividend distribution tax ascertained at ₹3,00,000 has to be reduced from bank

balance while computing full value of consideration under section 46(2), ₹50,000, being the difference

between ₹3,00,000 and ₹2,50,000 has to be reduced from General Reserve for calculating deemed dividend

under section 2(22)(c ).

Page 27: Capital Gains [Income Tax]

Capital Gains 10.23

[CMA RTP D10]

Question: What is the tax treatment of consequence for repurchase or buy back of shares or specified

securities by a company?

Answer: As per the provisions in Sec. 46A:

1. Where a shareholder receives any consideration from the company for purchase of its own shares or

other specified securities, it is a transfer chargeable under the head Capital Gains.

2. The Capital Gains taxable in the previous year in which the shares or securities are purchased by the

Company.

3. Capital Gains = Value of Consideration Received Less Cost of Acquisition or Indexed cost of

acquisition.

4. No deemed dividend: In case of buy back of shares, there is no question of deemed dividend u/s

2(22)(d).

Reference Legal Decisions:

Shares held as

Stock-in-Trade

Incase Shares treated as Stock-in-Trade are exchanged for Shares of other companies,

then, Business Profit = Market Value of Shares exchanged Less Book Value of Original

Shares. [Orient Trading Co. Ltd. 224 ITR 371 (SC)]

Reduction of

Capital

When there is a reduction in the Face Value of the Shares and consequent payment by

the Company to the Shareholders towards such reduction, the reduction of Share

Capital is charged to Capital Gains Tax. [Kartikeya vs. Sarabhai 228 ITR 163 (SC)]

[CMA RTP J12 & D12]

Question: X Ltd. having an issued capital of ₹50,00,000 in equity shares of ₹100 each. On March 2016,

company decided to buy-back equity shares to the extent of 20%. Tushar, holding 500 shares of the

company, has received the buy-back consideration on the shares bought-back, @ ₹130 per share. He had

purchased these shares 14 months earlier @ ₹105 per share. Discuss the taxability.

Answer: Where a shareholder receives any consideration from the company for purchase of its own

shares or other specified securities, it is a transfer chargeable under the head capital gains. Such capital

gain is chargeable to tax in the previous year in which the shares or securities are purchased by the

company.

Computation of Capital Gains

Consideration for transfer of 100 equity shares [500 shares × 20%] @ ₹130 per share 13,000

Less Indexed Cost of Acquisition [100 equity shares × 105 × 1081/1024] 11,084

Long term Capital Gains 1,916

[CA INTER N03, 6 Marks]

Question: What are the transactions not regarded as transfer as per section 47 under the Income

Page 28: Capital Gains [Income Tax]

Income Tax 10.24

Tax Act, 1961. (List at least six of such transactions).

Answer: Transactions not regarded as transfer [Section 47]

The following transactions will not be regarded as transfer for the purposes of capital gains tax.

S Provisions

(i) Any distribution of capital asset on the total or partial partition of HUF.

(iii) Any transfer of capital asset under a gift or will or an irrevocable trust. However, it would

not include transfer under gift or an irrevocable trust of a capital asset being shares or

debentures or warrants allotted by a company to the employees under the ESOP scheme in

accordance with the guidelines issued by the Central Government in this behalf.

(iv) Transfer of capital asset to its wholly owned Indian subsidiary company by a holding

company

(v) Transfer of capital asset by a wholly owned subsidiary company to its Indian holding

company

(vi) Transfer of capital asset by the amalgamating company to the Indian amalgamated

company in a scheme of amalgamation

(via) Transfer of shares held in an Indian company by the amalgamating foreign company in a

scheme of amalgamation to the amalgamated foreign company, if –

(a) at least 25% of the shareholders of the amalgamating foreign company continue to

remain shareholders of the amalgamated foreign company, and

(b) such transfer does not attract tax on capital gains in the country in which the

amalgamating company is incorporated.

(viaa) any transfer, in a scheme of amalgamation of a banking company with a banking institution

sanctioned and brought into force by the Central Government

(viab)1 any transfer, in a scheme of amalgamation, of a capital asset, being a share of a foreign

company, referred to in the Explanation 5 to clause (i) of sub-section (1) of section 9, which

derives, directly or indirectly, its value substantially from the share or shares of an Indian

company, held by the amalgamating foreign company to the amalgamated foreign

company, if—

(A) at least twenty-five per cent of the shareholders of the amalgamating foreign company

continue to remain shareholders of the amalgamated foreign company; and

(B) such transfer does not attract tax on capital gains in the country in which the

amalgamating company is incorporated;

(vib) Transfer of capital asset by the demerged company to the resulting Indian company in a

scheme of demerger

1 Following clause (viab) shall be inserted by the Finance Act, 2015, w.e.f. 1-4-2016

Page 29: Capital Gains [Income Tax]

Capital Gains 10.25

(vic) Transfer of capital asset, being a share or shares held in Indian company, by the demerged

foreign company to the resulting foreign company, if –

(a) the shareholders holding not less than three-fourths in the value of the shares of the

demerged foreign company continue to remain shareholders of the resulting foreign

company; and

(b) such transfer does not attract tax on capital gains in the country, in which the demerged

foreign company is incorporated.

(vica) any transfer in a business reorganisation, of a capital asset by the predecessor co-operative

bank to the successor co-operative bank;

(vicb) any transfer by a shareholder, in a business reorganisation, of a capital asset being a share

or shares held by him in the predecessor co-operative bank if the transfer is made in

consideration of the allotment to him of any share or shares in the successor co-operative

bank.

(vicc)1 any transfer in a demerger, of a capital asset, being a share of a foreign company, referred to

in the Explanation 5 to clause (i) of sub-section (1) of section 9, which derives, directly or

indirectly, its value substantially from the share or shares of an Indian company, held by the

demerged foreign company to the resulting foreign company, if—

(a) the shareholders, holding not less than three-fourths in value of the shares of the

demerged foreign company, continue to remain shareholders of the resulting foreign

company; and

(b) such transfer does not attract tax on capital gains in the country in which the demerged

foreign company is incorporated:

(vid) any transfer or issue of shares by the resulting company, in a scheme of demerger to the

shareholders of the demerged company if the transfer or issue is made in consideration of

demerger of the undertaking;

(vii) any transfer by a shareholder, in a scheme of amalgamation, of a capital asset being a share

or shares held by him in the amalgamating company, if—

(a) the transfer is made in consideration of the allotment to him of any share or shares in the

amalgamated company except where the shareholder itself is the amalgamated company,

and

(b) the amalgamated company is an Indian company;

(viia) any transfer of a capital asset, being bonds or Global Depository Receipts referred to in sub-

section (1) of section 115AC, made outside India by a non-resident to another non-resident;

(viib) any transfer of a capital asset, being a Government Security carrying a periodic payment of interest,

made outside India through an intermediary dealing in settlement of securities, by a non-resident to

1 Following clause (viab) shall be inserted by the Finance Act, 2015, w.e.f. 1-4-2016

Page 30: Capital Gains [Income Tax]

Income Tax 10.26

another non-resident.

(viii) any transfer of agricultural land in India effected before the 1st day of March, 1970;

(ix) any transfer of a capital asset, being any work of art, archaeological, scientifi c or art

collection, book, manuscript, drawing, painting, photograph or print, to the Government or

a University or the National Museum, National Art Gallery, National Archives or any such

other public museum or institution as may be notified by the Central Government in the

Official Gazette to be of national importance or to be of renown throughout any State or

States.

(x) any transfer by way of conversion of bonds or debentures, debenture-stock or deposit

certificates in any form, of a company into shares or debentures of that company;

(xa) any transfer by way of conversion of bonds referred to in clause (a) of sub-section (1)

of section 115AC into shares or debentures of any company;

(xi) any transfer made on or before the 31st day of December, 1998 by a person (not being a

company) of a capital asset being membership of a recognised stock exchange to a company

in exchange of shares allotted by that company to the transferor.

(xii) any transfer of a capital asset, being land of a sick industrial company, made under a

scheme prepared and sanctioned under section 18 of the Sick Industrial Companies (Special

Provisions) Act, 1985 (1 of 1986) where such sick industrial company is being managed by

its workers' co-operative

(xiii) any transfer of a capital asset or intangible asset by a firm to a company as a result of

succession of the firm by a company in the business carried on by the firm, or any transfer

of a capital asset to a company in the course of demutualisation or corporatisation of a

recognised stock exchange in India as a result of which an association of persons or body of

individuals is succeeded by such company :

Provided that—

(a) all the assets and liabilities of the firm or of the association of persons or body of

individuals relating to the business immediately before the succession become the assets

and liabilities of the company;

(b) all the partners of the firm immediately before the succession become the shareholders of

the company in the same proportion in which their capital accounts stood in the books of

the firm on the date of the succession;

(c) the partners of the firm do not receive any consideration or benefit, directly or indirectly,

in any form or manner, other than by way of allotment of shares in the company; and

(d) the aggregate of the shareholding in the company of the partners of the firm is not less

than fifty per cent of the total voting power in the company and their shareholding

continues to be as such for a period of five years from the date of the succession;

(e) the demutualisation or corporatisation of a recognised stock exchange in India is carried

Page 31: Capital Gains [Income Tax]

Capital Gains 10.27

out in accordance with a scheme for demutualisation or corporatisation which is approved

by the Securities and Exchange Board of India established under section 3 of the Securities

and Exchange Board of India Act, 1992 (15 of 1992);

(xiiia) any transfer of a capital asset being a membership right held by a member of a recognised

stock exchange in India for acquisition of shares and trading or clearing rights acquired by

such member in that recognised stock exchange in accordance with a scheme for

demutualisation or corporatisation which is approved by the Securities and Exchange Board

of India established under section 3 of the Securities and Exchange Board of India Act, 1992

(15 of 1992);

(xiiib) any transfer of a capital asset or intangible asset by a private company or unlisted public

company to a limited liability partnership or any transfer of a share or shares held in the

company by a shareholder as a result of conversion of the company into a limited liability

partnership in accordance with the provisions of section 56 or section 57 of the Limited

Liability Partnership Act, 2008 (6 of 2009):

Provided that—

(a) all the assets and liabilities of the company immediately before the conversion become

the assets and liabilities of the limited liability partnership;

(b) all the shareholders of the company immediately before the conversion become the

partners of the limited liability partnership and their capital contribution and profit sharing

ratio in the limited liability partnership are in the same proportion as their shareholding in

the company on the date of conversion;

(c) the shareholders of the company do not receive any consideration or benefit, directly or

indirectly, in any form or manner, other than by way of share in profit and capital

contribution in the limited liability partnership;

(d) the aggregate of the profit sharing ratio of the shareholders of the company in the

limited liability partnership shall not be less than fifty per cent at any time during the

period of five years from the date of conversion;

(e) the total sales, turnover or gross receipts in the business of the company in any of the

three previous years preceding the previous year in which the conversion takes place does

not exceed sixty lakh rupees; and

(f) no amount is paid, either directly or indirectly, to any partner out of balance of

accumulated profit standing in the accounts of the company on the date of conversion for a

period of three years from the date of conversion.

(xiv) [CMA INTER SY12, J15, 4 Marks]

Question: State the conditions to be satisfied when a sole proprietary concern is succeeded by a

company, to avail tax exemption in respect of capital gains.

Answer: where a sole proprietary concern is succeeded by a company in the business carried on

Page 32: Capital Gains [Income Tax]

Income Tax 10.28

by it as a result of which the sole proprietary concern sells or otherwise transfers any capital

asset or intangible asset to the company:

Provided that—

(a) all the assets and liabilities of the sole proprietary concern relating to the business

immediately before the succession become the assets and liabilities of the company;

(b) the shareholding of the sole proprietor in the company is not less than fifty per cent of

the total voting power in the company and his shareholding continues to remain as such for

a period of five years from the date of the succession; and

(c) the sole proprietor does not receive any consideration or benefit, directly or indirectly, in

any form or manner, other than by way of allotment of shares in the company;

(xv) any transfer in a scheme for lending of any securities under an agreement or arrangement,

which the assessee has entered into with the borrower of such securities and which is

subject to the guidelines issued by the SEBI, in this regard;

(xvi) any transfer of a capital asset in a transaction of reverse mortgage under a scheme made and

notified by the Central Government

(xvii) any transfer of a capital asset, being share of a special purpose vehicle to a business trust in

exchange of units allotted by that trust to the transferor.

Explanation.—For the purposes of this clause, the expression "special purpose vehicle" shall

have the meaning assigned to it in the Explanation to clause (23FC) of section 10.]

(xviii)1 any transfer by a unit holder of a capital asset, being a unit or units, held by him in the

consolidating scheme of a mutual fund, made in consideration of the allotment to him of a

capital asset, being a unit or units, in the consolidated scheme of the mutual fund:

Provided that the consolidation is of two or more schemes of equity oriented fund or of two

or more schemes of a fund other than equity oriented fund.

Explanation.— For the purposes of this clause,—

(a) "consolidated scheme" means the scheme with which the consolidating scheme merges

or which is formed as a result of such merger;

(b) "consolidating scheme" means the scheme of a mutual fund which merges under the

process of consolidation of the schemes of mutual fund in accordance with the Securities

and Exchange Board of India (Mutual Funds) Regulations, 1996 made under the Securities

and Exchange Board of India Act, 1992 (15 of 1992);

(c) "equity oriented fund" shall have the meaning assigned to it in clause (38) of section 10;

(d) "mutual fund" means a mutual fund specified under clause (23D) of section 10.

1 Following clause (viab) shall be inserted by the Finance Act, 2015, w.e.f. 1-4-2016

Page 33: Capital Gains [Income Tax]

Capital Gains 10.29

[CMA RTP D11]

Question: What is a ‘Foreign Exchange Asset’?

Answer: Foreign Exchange Asset- means those ‚specified asset‛ which the assessee has acquired or

purchased with, or subscribed to in, convertible foreign exchange.

The following are the ‚specified assets‛:

(i) shares in an Indian Company (public or private)

(ii) debentures issued by an Indian Company which is not a private company ;

(iii) deposits with an Indian Company which is not a private Company, it may be even deposit with SBI

or any other banking company;

(iv) any security of the Central Government ; and

(v) such other asset as the Central Government may specify in this behalf by notification in the Official

Gazette.

[CMA RTP J12 & D12]

Question: Mohini, a non-resident Indian, purchased 5,000 shares in Happy Days Ltd, an Indian

Company, in foreign currency for $50,000 in August 2013. She sold these shares to Mrigakshi, another

non-resident in Singapore, in March 2016 for $67,000. Discuss the impact of the given transactions, if any,

in the assessment of Mohini.

Answer: As per S.47(viia), where a Non-resident Indian acquires bonds in foreign currency, and the same

is transferred outside India to another non-resident in foreign currency, then the transaction is not a

transfer and hence not chargeable to capital gains.

However, if the asset was transferred in India, then provisions of S.115AC shall apply and the Capital

Gain shall be chargeable to tax @ 10%, as it is a Long-Term Capital Asset.

Question: Explain the withdrawal of exemption u/s 47A

Answer: Withdrawal of exemption in certain cases.

S Not transfer &

not taxed u/s 45

Before

Period

Withdrawal and taxable u/s 45,

in the following situation

1 S.47(iv)

/ (v)

Transfer by subsidiary or holding

company to wholly owned holding

company or subsidiary company

8 years (a) Converting such capital asset

in to stock-in-trade

(b) ceases to be wholly owned

subsidiary company

2 S.47(xi) Transfer of membership of a recognised

stock exchange for shares

3 years Transfer of shares allotted in

exchange of membership in a

recognised stock exchange

3 S.47(xiii) Transfer of capital asset by a firm / sole

trade to a company as a result of

- The requirements of S.47(xiii) /

Page 34: Capital Gains [Income Tax]

Income Tax 10.30

/ (xiv) succession of the firm / sole trade (xiv) are not complied

4 S.47(xiiib) Transfer of capital asset by a private

company or unlisted public company to a

LLP a result of succession of the company

- The requirements of S.47(xiiib)

are not complied

Question: Write Short note on deemed cost of acquisition under the head capital gain?

Answer: When the capital asset becomes the property of the assessee in any of the manner mentioned

below, the cost of the acquisition of the asset shall be deemed to be the cost for which the previous owner

of the property acquired it. Following are the examples of the cases:

a. On the distribution of the asset on the total / partial partition of HUF

b. under a gift or will

c. By succession, inheritance or devolution.

d. On any distribution of assets on the liquidation of the company.

e. Under a transfer to a revocable or irrevocable trust;

f. On the transfer by a wholly owned India subsidiary company to its holding company or vice versa

g. On any transfer in a scheme of amalgamation of two Indian companies subject to certain conditions

u/s 47.

S w.r.t S

49 Cost with reference to certain

modes of acquisition

49(1) Where the capital asset became the

property of the assessee

49(1)(i) S.47(i) On any distribution of assets on

the total or partial partition of a

HUF

CA = CAP

49(1)(ii) S.47(iii) under a gift or will CA = CAP

49(1)(iii)(a) by succession, inheritance or

devolution, or

CA = CAP

49(1)(iii)(b) on any distribution of assets on the

dissolution of a firm, BOI, or AOP

dissolution had taken place at any

time before the 1st day of April,

1987, or

CA = CAP

49(1)(iii)(c) on any distribution of assets on the

liquidation of a company, or

CA = CA of the previous owner

Page 35: Capital Gains [Income Tax]

Capital Gains 10.31

49(1)(iii)(d) under a transfer to a revocable or

an irrevocable trust, or

CA = CA of the previous owner

49(1)(iii)(e) under any such transfer as is

referred to in S.47(iv) / (v) / (vi) /

(via) / (viaa) / (vica) / (vicb) / (xiii) /

(xiiib) / (xiv)

CA = CA of the previous owner

49(1)(iv) Such assessee being a HUF, by the

mode referred to in S.64(2) after

31.12.1969

CA = CA of the previous owner

49(2) 47(vii) transfer by amalgamating

company

CA = CA of the amalgamating company

49(2A) 47(x)

/(xa)

conversion of bonds / debentures

into share or debenture

CA = CA of such original bond or

debenture

49(2AA) 17(2)(vi) sweat equity shares or specified

securities w.e.f AY2010-11

CA = FMV

49(2AAA) 47(xiiib) conversion of company into LLP CA of asset = CA of the shares

49(2AB) 115WC

(1)(ba)

sweat equity shares or specified

securities

CA = FMV

49(2ABB)1 115AC

(1)(b)

receiving shares for Global

Depository Receipt (GDR) by NRI

CA shares = FMV of shares on redemption

of GDR

49(2AC) 47(xvii) shares received for units of trust CA shares = CA of units

49(2AD)2 47(xviii) units received for units CA units = CA of original units

49(2C) 47(vid) Shares received in resulting

company in demerger

49(2D) Shares held in demerged company

after S.49(2C)

CA in the demerged company is reduced

by CA u/s 49(2C)

49(2E) S.49(2)/(2C)/(2D apply to business reorganisation of a co-operative bank as referred to

in section 44DB.

49(3) 47(iv) /

(v)

CG by S.47A CA to the transferee company = CA of such asset

was acquired

49(4) 56(2)(vii)

/(viia)

Cost of Acquisition of

property taxable

CA = Taxable value considered for valuing the

gifts

1 Applicable w.e.f. 1.4.2016

2 Applicable w.e.f. 1.4.2016

Page 36: Capital Gains [Income Tax]

Income Tax 10.32

[CMA INTER SY08, D12, 6 Marks]

Question: Excel Ltd. allotted 1000 (sweat) equity shares of ₹10 each to Mr. Rao, General Manager. The fair

market value of the shares computed in accordance with the method prescribed under the Income-tax Act

/ Rules was ₹500 per share, whereas it was allotted at ₹300 per share. What is the perquisite value of

sweat equity shares allotted to Mr. Rao? In case these shares are sold subsequently, what would be their

cost of acquisition in the hands of Mr.Rao?

Answer: As per section 17(2)(vi) the value of sweat equity shares chargeable as perquisite shall be the fair

market value of such shares on the date on which the option is exercised by the assesses as reduced by

the amount actually paid or recovered from him in respect of such shares.

Particulars Amount(₹ )

Fair market value of 1000 equity shares @ ₹500 each 5,00,000

Less: Amount recovered from Mr. Rao 1000 x 300 each 3,00,000

Value of perquisite of sweat equity shares allotted to Mr. Rao 2,00,000

As per S.49(2AA) the cost of acquisition of such shares shall be adopted while computing the capital gain.

Hence, the cost of acquisition for Mr. Rao shall be ₹3,00,000.

[CA INTER M08 & M08, 5 & 6 Marks]

Question: Explain the computation of capital gain in case of depreciable asset u/s 50.

Answer: Computation of capital gains in case of a depreciable asset:

Section 50 provides for the computation of capital gains in case of depreciable assets. Where the f ull

value of consideration received or accruing as a result of the transfer of the asset plus the full value

of such consideration for the transfer of any other capital asset falling within the block of assets

during the previous year, exceeds the aggregate of the following amounts, namely:

1. expenditure incurred wholly and exclusively in connection with such transfer;

2. WDV of the block of assets at the beginning of the previous year;

3. the actual cost of any assets falling within the block of assets acquired during the previous year,

such excess shall be deemed to be the capital gains arising from the transfer of short -term capital

assets.

Where all assets in a block are transferred during the previous year, the block itself will cease to

exist. In such a situation, the difference between the sale value of the assets and the WDV of the

block of assets at the beginning of the previous year together with the actual cost of any asset falling

within that block of assets acquired by the assessee during the previous year will be deemed to be

the capital gains arising from the transfer of short term capital assets.

Page 37: Capital Gains [Income Tax]

Capital Gains 10.33

[CA INTER M99 & N08, 7 & 10 Marks][Similar in M99 Kishore Industries]

Question: Singhania & Co. own six machines, put in use for business in March, 2012. The

depreciation on these machines is charged @ 15%. The written down value of these machines at the

end of the previous year relevant to assessment year 2016-17 was ₹8,50,000. A new plant was bought

for ₹8,50,000 on 30th November, 2015.

Three of the old machines were sold on 10 th June, 2015 for ₹11,00,000.

You are required to:

1. determine the claim of depreciation for Assessment Year 2016-17.

2. compute the capital gains liable to tax for Assessment Year 2016-17.

3. If Singhania & Co. had sold the three machines in June, 2015 for ₹21,00,000, will there be any

difference in your above workings? Explain.

Answer:

Computation of depreciation claim for A.Y.2016-17

Particulars ₹

W.D.V. of the block as on 1.4.2015 8,50,000

Add Purchase of new plant during the year _8,50,000

17,00,000

Less Sale consideration of old machinery during the year 11,00,000

W.D.V of the block as on 31.03.16 _6,00,000

Since the value of the block as on 31.3.16 comprises of a new asset which has been put to use for less

than 180 days, depreciation is restricted to 50% of the prescribed percentage of 15% i.e. depreciation

is restricted to 7½%. Therefore, the depreciation allowable for the year is ₹45,000, being 7½% of

₹6,00,000.

(2) The provisions u/s 50 for computation of capital gains in the case of depreciable assets can be

invoked only under the following circumstances:

a. When one or some of the assets in the block are re-sold for consideration more than the value of

the block.

b. When all the assets are transferred for a consideration more than the value of the block.

c. When all the assets are transferred for a consideration less than the value of the block.

Since in the first two cases, the sale consideration is more than the written down value of the block,

the computation would result in short term capital gains.

In the third case, since the written down value exceeds the sale consideration, the resultant figure

would be a short term capital loss.

In the given case, capital gains will not arise as the block of asset continues to exist, and some of the

assets are sold for a price which is lesser than the written down value of the block.

Page 38: Capital Gains [Income Tax]

Income Tax 10.34

(3) If the three machines are sold in June, 2015 for ₹21,00,000, then short term capital gains would

arise, since the sale consideration is more than the aggregate of the written down value of the block

at the beginning of the year and the additions made during the year.

Particulars ₹ ₹

Sale consideration 21,00,000

Less W.D.V. of the machines as on 1.4.2015 8,50,000

Purchase of new plant during the year _8,50,000 17,00,000

Short term capital gains _4,00,000

50A Transfer of depreciable assets by Power Sector Units

[S.32(1)(i) depreciated under SLM)

CG = Consideration less original cost

[CA INTER M03, M06 & N08, 5, 6 & 6 Marks][CMA INTER SY08, J14, 5 Marks]

Question: What is Slump sale? How is capital gain computed in case of a slump sale?

Question: Write short notes on: Special provision for computation of capital gains in the case of

slump sale u/s 50B of Income-tax Act, 1961.

Answer: Meaning of slump sale: Sale of undertaking for a lump sum consideration without determining

the value of the individual assets and liabilities [S.2(42C)].

If any person has transferred the entire unit or part of the unit for a lump-sum consideration, capital

gain shall be computed for the entire unit instead of individual asset. Any profit or gain from the

slump sale shall be chargeable as Long Term Capital Gain in the year of transfer, if undertaking is

owned by the assessee for more than 36 months immediately preceding the date of transfer.

Otherwise it shall be short term capital gain. Benefit of indexation is not allowed in the case of slump

sale.

For computing capital gain, cost of acquisition and cost of improvement shall be Net Worth.

Net Worth shall be the total value of assets of the undertaking as reduced by the liabilities appearing

in the books of account. Revaluation shall be ignored.

A report from of Chartered Accountant is required to be furnished certifying that net worth has been

computed correctly.

While computing the net worth in case of slump sale for the purpose of computing capital gain, in the

case of capital assets in respect of which the whole of the expenditure has been allowed or is

allowable as a deduction under 35AD, its cost shall be taken as Nil as the entire cost has been allowed

as a deduction (w.e.f. 01.04.2010)

Page 39: Capital Gains [Income Tax]

Capital Gains 10.35

[CMA INTER SY08, D13, 1 Mark]

Question: In case of slum sale of any undertaking indexation benefit is _______ (allowed / not allowed)

for the purpose of computation of capital gain1.

[CMA INTER D11, 8 Marks]

Question: VERTIKA Limited has two units - one engaged in manufacture of computer hardware and the

other involved in developing software. As a restructuring drive, the company has decided to sell its

software unit as a going concern by way of slump sale for ₹385 lakh to a new company called SUMEDHA

Limited, in which it holds 74% equity shares.

The balance sheet of VERTIKA Limited as on 31st March, 2016 being the date on which software unit has

been transferred, is given hereunder—

Balance Sheet as on 31.3.2016

Liabilities in lakh in lakh Assets in lakh in lakh

Paid up Share Capital 300 Fixed Assets

General Reserve 150 Hardware unit 170

Share Premium 50 Software unit 200 370

Revaluation Reserve 120 Debtors

Current Liabilities Hardware unit 140

Hardware unit 40 Software unit 110 250

Software unit 90 130 Inventories

Hardware unit 95

Software unit 35 130

750 750

Following additional information are furnished by the management.

(i) The Software unit is in existence since May, 2012.

(ii) Fixed assets of software unit include land which was purchased at ₹40 lakh in the year 2009 and

revalued at ₹60 lakh as on March 31, 2016.

(iii) Fixed assets of software unit mirrored at ₹140 lakh (₹200 lakh minus land value ₹60 lakh) is written

down value of depreciable assets as per books of account. However, the written down value of these

assets u/s 43(6) of the Income Tax Act is ₹90 lakh.

Ascertain the tax liability, which would arise from slump sale to SUMEDHA Limited.

Answer: As per section 50B, any profits and gains arising from the slump sale effected in the previous

year shall be chargeable to income-tax as capital gains arising from the transfer of capital assets and shall

be deemed to be the income of the previous year in which the transfer took place.

1 Answer: not allowed

Page 40: Capital Gains [Income Tax]

Income Tax 10.36

If the assessee owned and held the undertaking transferred under slump sale for more than 36 months

before slump sale, the capital gain shall be deemed to be long-term capital gain. Indexation benefit is not

available in case of slump sale as per section 50B(2).

Ascertainment of tax liability from slump sale of software unit

Particulars (₹ in lakh)

Sale consideration for slump sale of Software Unit 385

Less Cost of acquisition being the net worth of Software Unit 185

Long term capital gains arising on slump sale 200

(The capital gains is long-term as the Software Unit is heldfor more than 36 months)

Tax liability on LTCG u/s 112 @ 20% on ₹200 lakhs 40.00

Add Surcharge 7½% 3.00

43.00

Add Education cess @ 2% and SHEC @ 1% i.e. totaling 3% 1.29

44.29

Working Note:

Computation of net worth of Software Unit ( ₹ in lakh)

(1) Book value of non-depreciable assets

(i) Land (Revaluation not to be considered) 40

(ii) Debtors 110

(iii) Inventories 35

(2) Written down value of depreciable assets u/s 43(6) (Note 1) 90

Aggregate value of total assets 275

Less Current liabilities of software unit 90

Net worth of software unit 185

Note 1: For computing net worth, the aggregate value of total assets in the case of depreciable assets shall

be the written down value of the block of assets as per section 43(6).

[CA INTER N04, 6 Marks][CMA RTP D10]

Question: Write short notes on capital gains on sale of property at less than Government value.

Answer: [S.50C] The following are the details analysis on chargeability of Capital Gains if a property is

sold at a price which is lower than Govt. Value.

1. Nature of Asset: The assessee transfers Land, or Building, or both.

Page 41: Capital Gains [Income Tax]

Capital Gains 10.37

2. Applicability: Sale Consideration is less than the value adopted or assessed or assessable by the

State Government Authority (referred to as ‘‘Stamp Valuation Authority‛) for the purpose of

payment of Stamp Duty.

Note: The word ‚Assessable‛ means the price which the Stamp Valuation Authority would have

adopted or assessed, if it were referred to such authority for the purpose of the payments of Stamp

Duty (W.e.f. 01.10.2009).

3. Consideration adopted for Capital Gains: Value adopted by the Stamp Valuation Authority.

4. Under the following conditions, a reference is made to the valuation officer :

a. The assessee can claim that the value adopted or assessed by the Stamp Valuation Authority

exceeds the Fair Market Value of the property as on the date of transfer.

b. Value adopted by the Stamp Valuation Authority is not disputed before any authority or Court.

c. Where the value determined by the Valuation Officer exceeds the value adopted by the Stamp

Valuation Authority, the Capital Gain shall be considered as follows —

Capital Gains = Value adopted by Stamp Valuation Authority Less Cost or Indexed Cost of Acquisition.

[CMA INTER SY12, D14, 4 Marks]

Question: From the following information, compute the income taxable under the head Capital gains and

Income from other sources in the hands of Sachin:

1. Sehwag gifted a vacant site to his friend Sachin on 23.05.2015 on the occasion of latter’s birthday.

2. Sehwag had acquired the said vacant site in May, 2011 for ₹30,00,000.

3. The fair market value of the site for stamp duty purposes on the date of gift i.e., on 23.05.2015 was

₹60,00,000.

4. Sachin sold the vacant site on 15.03.2016 for a consideration of ₹70 lakhs when its stamp duty value

on the date of sale was ₹90 lakhs.

For capital gains, state with reason, whether it is short-term or long-term. Also compute the capital gains

chargeable to tax in the hands of Sehwag.

Answer: Computation of total income in the hands of Sachin A.Y. 2016-17

Particulars ₹

Capital gains:

Sale consideration – Stamp duty valuation, as per section 50C 90,00,000

Less Cost of acquisition 60,00,000

Short term capital gain 30,00,000

Income from other sources:

Stamp duty value of the property on the date of receipt of gift i.e., 3.05.15 60,00,000

Total income 90,00,000

Page 42: Capital Gains [Income Tax]

Income Tax 10.38

Note: Where a property is received without consideration, the FMV i.e., stamp duty valuation is the value

taxable under the head „income from other sources‟ in the hands of Sachin. The holding period in the

hands of Sachin shall be from the date of receipt of property and not from the date of acquisition by the

previous owner. Since it was received on 23.05.2015 and sold on 15.03.2016 it is a short-term capital asset

and the resultant gain is short-term capital gain. The donor Sehwag is not subject to capital gains as the

property was not transferred but was gifted which is not regarded as transfer u/s 47.

50D If consideration received or accruing as a result of transfer of a

capital asset is not ascertainable or cannot be determined

SC = FMV on the date of

transfer [from AY 2013-14]

Question: Write Short note on advance money forfeited u/s 51 under the head Capital Gain:

Answer: If any advance money is forfeited by the assessee, then the amount so forfeited shall be

deducted from the cost for which the asset was acquired or the fair market value or written down value,

as the case may be for computing cost of acquisition.

This provision is applicable only when the transfer as per the original agreement does not take place and

the advance money is received and forfeited by the assessee as per the agreement

If advance money forfeited is more than the cost of acquisition, then such excess shall not be taxable.

Key Note: Only the amount forfeited by the assessee is deducted, the amount forfeited by the previous

owner shall not be considered.

Note: If any advance money is forfeited on after the previous year 2014-15 is included in the total income

u/s 56(2)(ix)

[CA INTER M11, 8 Marks][CMA INTER D11, 7 Marks (Aggarwal instead of Rakesh)]

Question: Mr. Rakesh purchased a house property on 14th April, 1979 for ₹1,05,000. He entered into an

agreement with Mr. B for the sale of house on 15th September, 1983 and received an advance of ₹25,000.

However, since Mr. B did not remit the balance amount, Mr. Rakesh forfeited the advance.

Later on, he gifted the house property to his friend Mr. A on 15th June, 1987.

Following renovations were carried out by Mr. Rakesh and Mr. A to the house property:

Amount (₹ )

By Mr. Aggarwal during FY 1979-80 10,000

By Mr. Aggarwal during FY 1984-85 50,000

By Mr. A during FY 1994-95 1,90,000

The fair market value of the property as on 1.4.1981 is ₹1,50,000.

Mr. A entered into an agreement with Mr. C for sale of the house on 1st June, 2000 and received an

advance of ₹80,000. The said amount was forfeited by Mr. A since Mr. C could not fulfill the terms of the

agreement.

Page 43: Capital Gains [Income Tax]

Capital Gains 10.39

Finally, the house was sold by Mr. A to Mr. Sanjay on 2nd January, 2016 for a consideration of ₹24,00,000.

Compute the capital gains chargeable to tax in the hands of Mr. A for the assessment year 2016-17. Cost

inflation indices are as under:

Answer: Computation of capital gains chargeable to tax in the hands of Mr. A

Particulars Amount (₹ )

Sale consideration 24,00,000

Less Indexed cost of acquisition ( )

7,56,700

Less Indexed cost of improvement in 1984-85 (

) 4,32,400

Less Indexed cost of improvement in 1994-95 (

) 7,93,012

Long term capital gain 4,17,888

Question: From the previous problem, Mr. A entered into an agreement with Mr. C for sale of the house

on 1st June, 2015 (instead of 1st June, 2000) and received an advance of ₹80,000. The said amount was

forfeited by Mr. A since Mr. C could not fulfill the terms of the agreement.

Answer: Computation of capital gains chargeable to tax in the hands of Mr. A

Particulars Amount (₹ )

Sale consideration 24,00,000

Less Indexed cost of acquisition ( )

16,21,500

Less Indexed cost of improvement in 1984-85 (

) 4,32,400

Less Indexed cost of improvement in 1994-95 (

) 7,93,012

Long term capital gain (loss) (4,46,912)

Income from other sources u/s. 56(2)(ix) 80,000

Question: Write short note on cost of acquisition u/s 55 under head capital gain?

Answer:

S Meaning of "adjusted", "cost of improvement" and "cost of acquisition"

55(1) "cost of any improvement" for the purposes of S.48 and S.49

55(1)(b) (1) goodwill of a business or a right to manufacture, produce or process any article or

thing or right to carry on any business

Nil

(2) other capital asset after 01.04.81 [before 31.03.81 is nil] Actual

55(2) Cost of acquisition for the purposes of S.48 and S.49

55(2)(a) goodwill of a business or a trade mark or brand name associated with a business or a

right to manufacture, produce or process any article or thing or right to carry on any

Page 44: Capital Gains [Income Tax]

Income Tax 10.40

business, tenancy rights, stage carriage permits or loom hours

(i) in case of purchase Actual

(ii) self-generated Nil

Question: Write short note on cost of acquisition of different types of Shares (S.55) under head capital

gains?

Types of Shares Actual Cost of acquisition Period of holding

55(2)(aa) Share or any other securities

(i) Shares Originally Purchased Actual payment

a. Primary market Allotment Price From allotment

b. Secondary market

Transaction through share

broker

Amount paid + Brokerage Charges +

Adjustment for ex. & com. Dividend /

interest

Date of broker’s

note

Transaction between parties As above (excluding Brokerage) Date of contract of

sale

(ii) Renouncing the shares Nil

(iii) Right Shares purchased by the

original shareholder

Price paid From Allotment

(iiia) Bonus shares Nil (but acquired before 01.04-1981,

FMV as on 01-04-1981 if available)

From Allotment

(iv) Person purchasing such offer

from original shareholder

Price paid to the seller + price paid to

the company

From Allotment

55(2)(ab) Shares under corporatisation CA of original membership of the

exchange

55(2)(b) Cost of Acquisition of assets is the value for which it was acquired by the assessee.

Expenses of capital nature for completing or acquiring the capital assets are includible in the

cost of Acquisition. It will be determined as below:

(i) If the asset is acquired before 01.04.1981, cost of acquisition shall be the expenditure incurred

by the assessee for acquiring the asset or its fair market value as on 01.04.1981, whichever is

higher.

(ii) If the asset has been acquired with effect from 01.04.1981 onwards, cost of acquisition shall be

the expenditure incurred by the assessee for acquiring the asset.

(iii) Where the capital asset became the property of the assessee on the distribution of the capital

assets of a company on its liquidation and the assessee has been assessed to the income-tax

under the head "capital gains" in respect of that asset u/s 46, means the fair market value of the

Page 45: Capital Gains [Income Tax]

Capital Gains 10.41

asset on the date of distribution.

(v) Where the capital asset, being a share or a stock of a company, become the property of the

assessee on

(a) the consolidation

(b) conversion of shares in to stocks

(c) re-conversion of any stock of the company into shares

(d) sub-division

(e) conversion of one kind of shares into another kind of shares

In all the above cases, CA = CA of the original stock or shares

55(3) If CA of previous owner is not ascertained, CA = FMA on the date of acquisition

[CA INTER N06, 6 Marks]

Question: Write short notes on cost of acquisition of self-generated assets, in the context of capital

gains, with reference to the provisions of the Income-tax Act, 1961:

Answer: Cost of acquisition of self-generated assets, in the context of capital gains

1. Cost of acquisition of a capital asset, being goodwill of a business or a trade mark or brand name

associated with a business or a right to manufacture, produce or process any article or thing, or

right to carry on any business, tenancy rights, stage carriage permits and loom hours [Section

55(2)(a)]

(i) If the above capital assets have been purchased by the assessee, the cost of acquisition is the

amount of the purchase price. For example, if Mr. A purchases a stage carriage permit from

Mr. B for ₹2 lacs, that will be the cost of acquisition for Mr. A.

(ii) If the above capital assets are self-generated, the cost of acquisition shall be taken as nil.

(iii) In case the capital asset is acquired by any mode given under clauses (i) to (iv) of section

49(1), the cost of acquisition will be the cost to the previous owner if the previous owner paid

for it. However, if it was self-generated by the previous owner, the cost of acquisition will be

taken as nil.

2. Cost of acquisition of other self-generated assets not covered u/s 55(2)(a) – In respect of self-

generated goodwill of a profession and other self-generated assets not specifically covered u/s

55(2)(a), the decision of the Supreme Court in CIT v. B.C. Srinivasa Setty [1981] 128 ITR 294 will

apply. In that case, the Supreme Court held that if the cost of acquisition of a self-generated asset

is incapable of determination, then transfer of such asset is not taxable and consequently the

gains thereon cannot be brought to charge.

[CMA INTER SY08,D12, 1 Mark]

Page 46: Capital Gains [Income Tax]

Income Tax 10.42

Question: When a person retires from a profession and receives any amount towards self-generated

goodwill, it is1

a) Taxable as income from profession

b) Exempt income u/s 10

c) Not taxable since there is no cost of acquisition

d) Taxable as capital gain

[CMA RTP J12 & D12]

Question: (a) P commenced a business on 10.5.95. The said business is sold by P on 25.8.15 and he

received ₹12 lacs towards goodwill.

(b) What will be your answer in the above case, if P had acquired the goodwill for this business for a

consideration of ₹3,00,000.

Answer:

Computation of Long Term Capital Gains for the A.Y. 2016-17 ₹

(a) Consideration for transfer 12,00,000

Less Indexed Cost of Acquisition (Self-Generated) NIL

Long term Capital Gains 12,00,000

(b) Consideration for transfer 12,00,000

Less Indexed Cost of Acquisition [₹3,00,000 × 1081/281] 11,54,093

Long term Capital Gains 45,907

[CMA RTP J12 & D12]

Question: R has been living in a rented accommodation since August 1986, and he is paying a rent of

₹4,000 per month. The landlord got the house vacated from R on 16.7.2015 and paid a sum of ₹15 lacs for

vacating the house. Compute Capital Gains, if any, in the hands of R.

Answer:

Computation of Long Term Capital Gains for the A.Y. 2016-17 ₹

Consideration for transfer 15,00,000

Less Indexed Cost of Acquisition (Self-Generated asset) NIL

Long term Capital Gains 15,00,000

[CA INTER M00, 7 Marks]

Question: A is a shareholder of X & Co Ltd., holding 1,000 shares of the face value of ₹10 each, allotted at

the time of the company’s incorporation in May, 1997. The company made a right issue in the ratio of 1:1

1 Answer: (d) taxable as capital gain

Page 47: Capital Gains [Income Tax]

Capital Gains 10.43

on 15.7.2015 at a premium of ₹40 per share. Instead of taking up the right, he renounced it in favour of B

at a price of ₹10 per share. What is the capital gain chargeable in the hands of A. What will be the cost of

the shares in the hands of B?

Answer:

Capital gain in the hand of A Section ₹

Sale consideration (₹10 x 1,000) 10,000

Less Cost of acquisition of right entitlement S.55(2)(aa)(ii) Nil

Short term Capital gain 10,000

The cost of shares in the hands of ‘B’ will be the aggregate of –

Cost of acquisition of right shares Section ₹

Price paid to A for acquiring the right ₹10,000

Add Amount paid to X & Co. Ltd. towards 1000 shares (1000 x 50) 55 ₹50,000

₹60,000

[CA IPCC M13, 8 Marks]

Question: Ms. Neelima had purchased 500 equity shares in A Ltd. at a cost of ₹30 per share (brokerage

1%) in February 1979.

She got 50 bonus shares in September 1980.

She again got 550 bonus shares by virtue of her holding on March, 1985.

Fair market value of the shares of A Ltd. on April, 1981 is ₹50.

In January 2016, she transferred all her shares @ ₹240 per share (brokerage 2%)

Compute the capital gains taxable in the hands of Ms. Neelima for the Assessment Year 2016-17

assuming.

(A) A Ltd. is an unlisted company and securities transaction tax was not applicable at the time of sale.

(B) A Ltd. is a listed company and the shares are sold in a recognised stock exchange and securities

transaction tax was paid at the time of sale.

In case A Ltd. is an unlisted company and STT was not applicable at the time of sale

Answer: Computation of capital gains of Ms. Neelima for the A.Y. 2016-17

(A) ₹

500 equity shares Original Share

Feb 1979

Bonus Share

Sep, 1980

Bonus Share

Mar, 85

Cost of acquisition WEH (Cost

or FMV 1981)

WEH (Cost

or FMV 1981)

Nil

Page 48: Capital Gains [Income Tax]

Income Tax 10.44

Sale proceeds (500 x ₹240) 1,20,000 12,000 1,32,000

Less Brokerage paid (2% of ₹120000) 2,400 240 2,640

Net sale consideration 1,17,600 11,760 1,29,360

Less Indexed cost of acquisition 2,70,2501 27,0252 NIL

Long term capital gain (loss) (1,52,650) (15,265) 1,29,360

Total Long term capital gain (loss) 22,880

(B) In case the company is listed & shares are sold in recognised stock exchange where STT is paid,

capital gain is exempt from tax. So capital loss computed shall be ignored.

[CA INTER N98, N01, M98 & N09, 5, 5, 6 & 3 Marks][CMA RTP D11]

Question: Write short note on reference to valuation officer u/s 55A under head capital gain?

Answer: For the purpose of ascertaining the fair market value of a capital asset, the assessing officer may

refer the valuation of capital asset to valuation of capital asset to valuation officer in the following cases:

1. If the report of the registered valuer is attached - If the value of the asset claimed by the assessee, is as

per the estimate made by a registered valuer and assessing officer is of opinion that the value so

claimed is less than its fair market value.

2. If the report of the registered valuer is not attached - If assessing officer is of the opinion that the

market value of the asset exceeds the value of the asset as claimed by the assessee and the difference

is more than 15% of the value claimed by assessee or more than 25,000/- of the value so claimed.

3. The assessing officer is of opinion that, having regard to the nature of asset and other relevant

circumstances, is necessary to make the reference.

[CA INTER N07, 9 Marks]

Question: Mr. Thomas inherited a house in Jaipur under will of his father in May, 2008. The house was

purchased by his father in January 1980 for ₹2,50,000. He invested an amount of ₹7,00,000 in construction

of one more floor in this house in June 2010. The house was sold by him in November 2015 for ₹37,50,000.

The valuation adopted by the registration authorities for charge of stamp duty was ₹47,25,000 which was

not contested by the buyer, but as per assessee’s request, the Assessing officer made a reference to

Valuation office. The value determined by the Valuation officer was ₹47,50,000. Brokerage @ 1% sale

consideration was paid by Mr. Thomas to Mr. Sunil. The market value of house as on 01.04.1981 was

₹2,70,000.

You are required to compute the amount of capital gain chargeable to tax for A.Y. 2016-17.

Answer: Computation of capital Gain in the Hands of Mr. Thomas for the AY. 2016-17.

1 *

+

2 *

+

Page 49: Capital Gains [Income Tax]

Capital Gains 10.45

Particulars Amounts (₹ )

Sales consideration (Being value determined by the Stamp Duty Authority) 47,25,000

Less Indexed cost of acquisitions: (

) 29,18,700

Less Indexed cost of improvement: (

) 10,64,276

Less Cost of transfer (i.e. 1% of ₹37,50,000) 37,500

Long Term Capital Gains 7,04,524

Question: Write short note on capital gains of non-residents and the method of conversion under Rule

115A under head capital gain?

Answer: In case of a non-resident assessee, the capital gains arising from the transfer of shares or

debentures in an India company, shall be computed by converting

1. The cost of acquisition of the asset.

2. The expenditure incurred wholly and exclusively in connection with such transfer and

3. Sale consideration received or accruing as a result of transfer of capital asset into the same foreign

currency as was initially utilized in the purchase of such shares or debentures. The capital gain so

computed in the foreign currency shall be reconverted into Indian Currency.

Indexation is not applicable

The method of conversion under Rule 115A is a follows:

(a) The Cost of Acquisition shall be converted at the average of the Telegraphic Transfer Buying Rate

(TIBR) and the Telegraphic Transfer Selling Rate (TTSR) as on the date of acquisition of shares /

debentures.

(b) The expenditure in connection with the transfer shall be converted at the average of TTBR and TTSR

as on the date of transfer of shares / debentures.

(c) The sale consideration shall be converted at the average of TTBR and TTSR as on the date of transfer

of shares / debentures.

(d) The capital gain computed in the foreign currency shall be converted into Indian currency by

applying TTBR as on the date of transfer of shares / debentures.

Question: Write short note on tax on STCG from listed equity shares u/s 111A u/h capital gain?

Answer: Short Term Capital Gain from transfer of equity shares or units or equity oriented mutual fund

shall be taxable @ 15% if:

They are sold through recognized stock exchange.

Security transaction tax is chargeable on such transaction.

Transfer took place on or after 01.10.2004

No Deduction under VI-A shall be allowed from such income.

Page 50: Capital Gains [Income Tax]

Income Tax 10.46

If normal income is less than basic exemption deficiency shall be allowed from short term capital

gain.

54 Profit on sale of property used for residence

54B Capital gain on transfer of land used for agricultural purposes not to be charged in certain cases

54D Capital gain on compulsory acquisition of lands and buildings not to be charged in certain cases

54EC Capital gain not to be charged on investment in certain bonds

54ED

54F Capital gain on transfer of certain capital assets not to be charged in case of investment in

residential house

54G Exemption of capital gains on transfer of assets in cases of shifting of industrial undertaking from

urban area

54GA Exemption of capital gains on transfer of assets in cases of shifting of industrial undertaking from

urban area to any Special Economic Zone

54GB Capital gain on transfer of residential property

54H Extension of time-limit for acquiring new asset

[CA INTER N05, 4 Marks]

Question: Briefly discuss about the provisions relating to claiming of exemption in respect of

short-term capital gains, in order to reduce tax liability. [54B, 54D and 54G]

Answer:

[CS EP D10 & D11, 2 marks & 3 marks]

Question: Distinguish between exemption to capital gains u/s 54G and exemption to capital gains u/s

54GA.

Question: Write short note on exemption u/s 54, 54B, 54D, 54EC, 54F, 54G, 54GA under the head capital

gain?

Answer:

54 54B 54D 54EC 54F 54G 54GA 54GB

Asset transferred

Residential house, whether stay or not

Agri. Land

Land / Build. of industrial undertaking (compulsorily acquired)

Any assets Any assets other than Residential House

Plant / Machinery or Land / Building for industrial undertaking in urban area

Plant / Machinery or Land / Building for industrial undertaking in urban area

Residential house [during 1.4.12 to 31.3.17]

Assessee I+HUF I+HUF All All I+HUF All All I+HUF

Capital Gain

LTCA LTCA / STCA

LTCA / STCA LTCA LTCA LTCA / STCA LTCA / STCA

Page 51: Capital Gains [Income Tax]

Capital Gains 10.47

Prior use Used by him or his parents for 2 years

Used for 2 years

May not own more than one residential house

New Assets One residential house

Agri. Land may be in rural or urban Area

Land / Building for industrial undertaking

Specific Security i.e., bond issued by Rural Electrification Corp. Ltd. (RECL) and National Highway Authority (max upto ₹50 lacs)

One residential house

Plant / Machinery or Land / Building for Industrial Undertaking in Non-Urban Area and meeting expenses of the shifting

Plant / Machinery or Land / Building for Industrial Undertaking in Special Economic Zone and meeting expenses of the shifting

Equity shares in eligible company

Period for investment

Purchase one residential house in India, 1 year earlier or 2 year after the date of transfer or constructed within 3 years from the date of transfer. In the case of compulsory acquisition from the date of the receipt of compensation

Within 2 years after the date of transfer

Within 3 years after the date of the receipt of compensation

6 months from the date of transfer [provided S.80C shall not be allowed]

Purchase, 1 year earlier or 2 year after the date of transfer or constructed within 3 years from the date of transfer. In case Compulsory acquisition from the date of the receipt of compensation

Within 1 year before or 3 years after transfer.

Within 1 year before or within 3 years after transfer

Shares acquired within the due date of filling of return and the eligible company

1

should purchase new asset

2

within one year from the date of subscription of shares

1 "eligible company" means a company which fulfils the following conditions, namely:—

(i) it is a company incorporated in India during the period from the 1st day of April of the previous year relevant to

the assessment year in which the capital gain arises to the due date of furnishing of return of income under sub-

section (1) of section 139 by the assessee;

(ii) it is engaged in the business of manufacture of an article or a thing;

(iii) it is a company in which the assessee has more than fifty per cent share capital or more than fifty per cent voting

rights after the subscription in shares by the assessee; and

(iv) it is a company which qualifies to be a small or medium enterprise under the Micro, Small and Medium

Enterprises Act, 2006 (27 of 2006); 2 "new asset" means new plant and machinery but does not include—

(i) any machinery or plant which, before its installation by the assessee, was used either within or outside India by

any other person;

(ii) any machinery or plant installed in any office premises or any residential accommodation, including

accommodation in the nature of a guest-house;

(iii) any office appliances including computers or computer software;

(iv) any vehicle; or

(v) any machinery or plant, the whole of the actual cost of which is allowed as a deduction (whether by way of

depreciation or otherwise) in computing the income chargeable under the head "Profits and gains of business or

profession" of any previous year.

Page 52: Capital Gains [Income Tax]

Income Tax 10.48

Withdrawal of

exemption

Transfer the new asset within 3 years

Transfer the new asset within 3 years

Transfer the new asset within 3 years

Transfer the new asset within 3 years or borrows securing such securities

Transfer the new asset within 3 years, purchases any residential house within 2 years or constructs any residential house within 3 years

Transfer the new asset within 3 years

Transfer the new asset within 3 years

If the equity shares sold within 5 years or if the new asset not purchased within 1 year or sold within 5 years by the eligible company

Exemption revoked

STCG STCG STCG LTCG LTCG STCG STCG LTCG

Note 1: Any amount remains unutilised, deposit in Capital Gain A/c scheme, on or before the date of furnishing the return of

income (except S.54EC). If the deposited amount not used for specific purpose within the due date will be charged u/s 45.

Note 2: Amount of exemption (except S.54F & S.54GB): WEL (amount invested or capital gain)

and for S.54F and S.54GB, exemption = WEL (

)

Question: Write a note on extension of time for acquiring new asset or depositing or investing amount

of capital gain. [S.54H]

Answer: the period of acquiring new asset [for claiming benefits u/s 54] is calculated from the date of

receipt of the compensation, if the original asset compulsorily acquired under any law

[CA INTER M00, 7 Marks]

Question: Raghavan owned a residential house at Madurai, the original cost of which was ₹1,00,000. It

was acquired on 1.9.1999. He sold the house on 1.6.2015 for ₹8,00,000 and purchased another house on

30.5.2016 at Tiruchi for ₹6,00,000. The second house at Tiruchi was sold by him for ₹8,00,000 on 30.6.2018.

Discuss the impact of these transactions with regard to assessment to capital gains.

Answer:

Assessment year 2016-17 ₹

Sale consideration of Madurai house 8,00,000

Less Indexed cost of acquisition *

+ 2,77,892

Long term capital gain 5,22,108

Less Exemption u/s 541 [WEL of (6,00,000 or 5,22,108)] 5,22,108

Long term capital gain Nil

Assessment Year 2019-2000

1 Assumed ₹6,00,000 was deposited in capital gain deposited account scheme

Page 53: Capital Gains [Income Tax]

Capital Gains 10.49

Sale consideration of Tiruchi house 8,00,000

Less Cost of acquisition [₹600000 – 5,22,108] being exemption availed u/s 54

as Tiruchi house is sold within 3 years

77,892

Short term Capital gain 7,22,108

[CMA RTP J12 & D12]

Question: Tina was the owner of two residential houses. On 5th April, 2015, she disposed one of the

houses and utilized the entire sale proceeds to construct first floor on her second house which he

completed by 15th March, 2016. She seeks your advice as to the taxability of transaction to capital gains

under the provisions of Income Tax Act, 1961.

Answer: Vide S.54, where an assessee transfers a residential house being a Long-term Capital Asset and

the Long-term capital gain on such transaction is utilized for construction of another residential house,

within a period of 3-years from the date of transfer, is entitled for exemption. Construction of first floor in

the existing building should be treated as independent residential unit entitled for exemption u/s 54 / 54F.

[P.V.NARASIMHAN 181 ITR 101]

[CMA RTP J11]

Question: On 16th January 2016, Suman sold agricultural land for ₹22 lacs. He incurred selling

expenses for ₹50,000. Compute capital gains:

If the land sold, was purchased on 1st February 1999 for ₹4 lacs, and the land was used for agricultural

purposes by his mother. He again purchased agricultural land of ₹8 lacs on 25th January 2016.

Amount deposited in a scheduled bank under“ Capital Gains Deposit Scheme is ₹4 lacs on 6th April, 2016.

Answer:

Computation of Capital Gains for the A.Y. 2016-17 ₹

Consideration for transfer 22,00,000

Less Expenses on transfer 50,000

Net Consideration 21,50,000

Less Indexed Cost of Acquisition ( 4,00,000 × 1081/351) 12,31,909

Long-term Capital Gains 9,18,091

Less Exemption u/s 54B Cost of New Land purchased 8,00,000

Less Amount deposited in “Capital Gains Account Scheme

before due date of furnishing return specified u/s 139(1)

₹4,00,000 4,00,000

Taxable long term Capital Gains NIL

Page 54: Capital Gains [Income Tax]

Income Tax 10.50

[CMA INTER D1, 1 Mark]

Question: On 17th January, 2016, M/s. Naina sold a house property and earned a long term capital gain

of1,02,50,000. She invested a sum of ₹50,00,000 in bonds specified in section 54EC on 8th March, 2016. She

further invested a sum of ₹50,00,000 in same bonds on 8th May, 2016. Taxable Income of M/s. Naina for

the Assessment Year 2016-17 will be1:

a) 50,00,000

b) 52,50,000

c) 1,00,50,000

d) 2,50,000

[CMA RTP J12 & D12]

Question: Saptarshi acquired shares of G Ltd. on 15.12.2001 for ₹5 lacs which were sold on 14.6.15 for ₹19

lacs. Expense on transfer of shares is ₹40,000. He invests ₹8 lacs in the bonds of Rural Electrification

Corporation Ltd., on 16.10.2015.

a. Compute capital gain for the assessment year 2016-17.

b. State the period for which the bonds should be held by the assessee. What will be the consequences if

such bonds are sold within the specified period?

c. What will be the consequences if Saptarshi takes a loan against the security of such bonds?

Answer:

(a) Computation of Capital Gains for the A.Y. 2016-17 ₹

Consideration for transfer 19,00,000

Less Expenses on Transfer 40,000

Net Consideration 18,60,000

Less Indexed Cost of Acquisition [5,00,000 × 1081/426] 12,68,779

Long-term Capital Gains 5,91,221

Less Exemption u/s 54EC 5,91,221

Taxable long-term Capital Gain NIL

(b) Saptarshi should not transfer or convert (otherwise than transfer) into money such bonds within 3

years from the date of their acquisition. If these bonds are transferred or converted into money within 3

years, capital gain exempted earlier shall attract taxability towards long-term capital gain of the previous

year in which such asset is transferred or converted into money.

(c) If any loan is taken against security of such bonds, it shall be taxable as long-term capital gains of the

previous year in which such loan is taken against the security of such bonds.

1Ans: B [52,50,000]

Page 55: Capital Gains [Income Tax]

Capital Gains 10.51

[CA IPCC M 14, 8 Marks]

Question: Mr. Roy, aged 55 years owned a Residential House in Ghaziabad. It was acquired by Mr. Roy

on 10-10-1986 for ₹6,00,000. He sold it for ₹53,00,000 on 4-11-2015. The stamp valuation authority of the

state fixed value of the property at ₹71,00,000. The Assessee paid 2% of the sale consideration as

brokerage on the sale of the said property.

Mr. Roy Acquired a Residential House property at Kolkata on 10-12-2015 for ₹10,00,000 and deposited

₹7,00,000 on 10-4-2016 and ₹5,00,000 on 15-6-2016 in the capital gains bonds of Rural Electrification

Corporation Ltd. He deposited ₹4,00,000 on 6-7-2016 and ₹3,00,000 on 1-11-2016 in the capital gain

deposit scheme in a Nationalized Bank for construction of an additional floor on the residential house

property in Kolkata.

Compute the Capital Gain chargeable to Tax for the Assessment Year 2016-17 and Income Tax chargeable

thereon assuming Mr. Roy has no other income.

Answer: Computation of taxable capital gains of Mr. Roy AY 2016-17

₹ ₹

Sale consideration (S.50C WEH of sale value or stamp duty value) 71,00,000

Less: Expenses on transfer (2% of ₹53 lacs) 1,06,000

Net sale consideration 69,94,000

Less: Indexed cost of acquisition (

) 46,32,857

Long term Capital Gains 23,61,143

Less: Exemption u/s 54 14,00,000

Exemption u/s 54 EC 7,00,000

21,00,000

Long Term Capital loss 2,61,143

Since Mr. Roy has no other income, he can set off ₹2,50,000 (the basic exemption limit) against his LTCG

income.

Tax is calculated as follows ₹

On the first ₹2,50,000 Nil

On balance ₹11,143, @ 20% 2,229

Less: Rebate u/s 87A 2,000

Balance 229

Add: 3% Education Cess 7

Total tax liability 236

Rounded off 240

Page 56: Capital Gains [Income Tax]

Income Tax 10.52

Notes:

1. ₹5 lacs worth of REC bonds purchased on 15-6-2016 are not considered for exemption u/s 54EC, as

they are purchased more than 6 months after the date of transfer.

2. ₹3 lacs deposited in Capital Gains Deposit Scheme on 1-11-2016 is not considered for exemption u/s

54 since the same is deposited after 31-7-2016, the due date of filling return of income.

[CA INTER N06, 10 Marks]

Question: Mr. Malik owns a factory building on which he had been claiming depreciation for the

past few years. It is the only asset in the block. The factory building and land appurtenant thereto

were sold during the year. The following details are available.

Building completed in September, 2008 for 10,00,000

Land appurtenant thereto purchased in April, 2006 for 12,00,000

Advance received from a prospective buyer for land in May, 2007,

forfeited in favour of assessee, as negotiations failed

50,000

WDV of the building block as on 1.4.2015 8,74,800

Sale value of factory building in November, 2015 8,00,000

Sale value of appurtenant land 25,00,000

The assessee is ready to invest in long-term specified assets u/s 54EC, within specified time.

Compute the amount of taxable capital gain for the assessment year 2016-17 and the amount to be

invested u/s 54EC for availing the maximum exemption.

Answer:

Computation of taxable capital gain of Mr. Malik for A.Y.2016-17

Particulars ₹ ₹

Factory building

Sale price of building 8,00,000

Less WDV as on 1.4.2015 8,74,800

Short-term capital loss on sale of building (S.50) (-) 74,800

Land appurtenant to the above building

Sale value of land 25,00,000

Less Indexed cost of acquisition [(12,0,000 – 50,000 u/s 51)1081/519] 23,95,279

Long-term capital gains on sale of land 1,04,721

Chargeable long term capital gain1 [amount required 29,921

1 As per S.74, short term capital loss can be set-off against any income under the head ―Capital gains‖, long-

Page 57: Capital Gains [Income Tax]

Capital Gains 10.53

to invest for availing exemption u/s 54EC]

[CA INTER M06 & M12, 8 & 8 Marks (similar in M12 Anusha instead of Mr. A)]

Question: Mr. A who transfers land and building on 2.1.2016, furnishes the following information:

1. Net consideration received ₹17 lakhs.

2. Value adopted by stamp valuation authority, which was not contested by Mr. A ₹19 lakhs.

3. Value ascertained by Valuation Officer on reference by the Assessing Officer ₹20 lakhs.

4. This land was distributed to Mr. A on the partial partition of his HUF on 1.4.1981. Fair market

value of the land as on 1.4.81 was ₹1,10,000.

5. A residential building was constructed on the above land by Mr. A at a cost of ₹3,20,000

(construction completed on 1.12.2012) during the financial year 2012-2013.

6. Short-term capital loss incurred on sale of shares during the financial year 2012-2013 ₹2,05,000.

Mr. A seeks your advice as to the amount to be invested in NABARD bonds so as to be exempt from

clutches of capital gain tax.

Answer: Computation of Capital Gains of Mr. A for the Assessment Year 2015-16.

Particulars ₹

(a) Sale consideration 17,00,000

(b) Stamp Duty Value 19,00,000

(c) Whichever is higher of (a) or (b) [S.50C(1)] 19,00,000

(d) Value as per valuation officer 20,00,000

Whichever is lower of (c) or (d) [S.50C(3)]

Full value of consideration (deemed)

19,00,000

Less Indexed cost of land (1,10,000 1081/100) 11,89,100

Indexed cost of building (3,20,000 1081/852) 4,06,009

Long-term capital gain 3,04,891

Less Brought forward short-term capital loss set off 2,05,000

Amount to be invested in NHAI / REC bonds [S.54EC] 99,891

[CA INTER N04, 6 Marks]

Question: Mr. `X’ furnishes the following data for the previous year ending 31.3.2016.

(a) Equity Shares of AB Ltd., 10,000 in number were sold on 31.5.2015, at ₹350 for each share.

(b) The above shares of 10,000 were acquired by `X’ in the following manner:

term or short-term. Therefore, in this case, short-term capital loss of ₹74,800 can be set-off against long-term

capital gain of ₹1,31,021

Page 58: Capital Gains [Income Tax]

Income Tax 10.54

(i) Received as gift from his father on 1.6.1980 (5,000 shares) the market price on 1.4.81 ₹50 per

share.

(ii) Bonus shares received from AB Ltd. on 21.7.1996 (2,000 shares).

(iii) Purchased on 1.2.2005 at the price of ₹125 per share (3,000 shares).

(c) Purchased one residential house at ₹25 lakhs, on 1.9.2016 from the sale proceeds of shares.

(d) `X’ already owns a residential house, even before the purchase of above house.

You are required to compute the taxable capital gain. He has no other source of income chargeable to

tax.

Answer: It is also assumed that the transaction of sale of shares is not exempt under section 10(38).

Computation of taxable capital gain of Mr. X for A.Y. 2016-17 ₹ ₹

Sale consideration received on sale of 10,000 shares @ ₹400 each 40,00,000

Less Indexed cost of acquisition

(a) 5,000 shares (gift from father on 1.6.1980) *

+ 27,02,500

(b) 2,000 bonus shares received from AB Ltd Nil

(c) 3000 shares purchased on 1.2.2005 *

+ 8,44,531 35,47,031

Long term capital gain 4,52,969

Less Exemption u/s 54F 2,83,106

Taxable long term capital gain 1,69,863

[CMA RTP J11]

Question: Bipasha purchased jewellery worth ₹2,20,000 during 1986-87. During the year 1991-92, she

further purchased jewellery worth ₹3,50,000. All the jewellery was sold by her on 15.6.15. The jewellery

purchased in 1986-87 was sold for ₹20 lacs and that purchased in 1991-92 was sold for ₹32 lacs. On 26.6.15

she deposited ₹50 lacs in capital gains scheme account. On 21.10.15 withdrawing from the Deposit

Account, she utilized ₹48 lacs for purchase of a residential house property in Kolkata. On the date of

transfer, she owns only one residential house.

Answer:

Computation of Capital Gains for the A.Y. 2016-17

a) On transfer of jewellery purchased during 86-87 Consideration for transfer 20,00,000

Less Indexed Cost of Acquisition (

) 16,98,714

Long-term Capital Gains 3,01,286

b) On transfer of jewellery purchased during 1991-92 Consideration for transfer 32,00,000

Less Indexed Cost of Acquisition (

) 19,01,256

Page 59: Capital Gains [Income Tax]

Capital Gains 10.55

Long term capital gains 12,98,744

In order to avail the maximum benefit u/s 54F, the exemption should be computed as follows:

Total long-term Capital Gain 16,00,030

Less Exemption u/s 54F *

+ 15,38,490

Taxable Long-term Capital Gains 61,540

[CMA RTP J11]

Question: P Ltd., owns an industrial undertaking manufacturing chemicals in Bangalore owns the

following assets

a) Plant and machinery (WDV ₹5 lacs) sold for ₹15 lacs.

b) Building (WDV ₹12 lacs) sold for ₹60 lacs.

c) Furniture and fixtures (WDV ₹50,000) sold for ₹1,80,000

d) Land cost of acquisition ₹5,00,000 during 1987-88 and sold for ₹30 lacs

The industrial undertaking was shifted as per policy of the Government from urban area to other area.

The new assets acquired during the period 1.1.16 to 31.3.16 are plant and machinery ₹20 lacs; buildings

₹40 lacs.

Compute capital gain chargeable to tax for the assessment year 2016-17.

Answer:

Short-term Capital Gains on Depreciable assets ₹

(i) Plant & Machinery (15,00,000 – 5,00,000) 10,00,000

(ii) Buildings (60,00,000 – 12,00,000) 48,00,000

(iii) Furniture & Fixtures (1,80,000 – 50,000) 1,30,000 59,30,000

Long-term capital gains on industrial land:

Consideration for transfer 30,00,000

Less Indexed Cost of Acquisition (5,00,000 × 1081/150) 36,03,333 6,03,333

Total Capital Gains 53,26,667

Less Exemption u/s 54G

Plant & Machinery 20,00,000

Building 40,00,000 65,00,000

but restricted to ₹5,91,221 [₹53,26,667 — 1,30,000, being STCG on furniture,

not eligible for the purpose of claiming exemption u/s 54G

5,91,221

Short term Capital Gains (on furniture) 1,30,000

Page 60: Capital Gains [Income Tax]

Income Tax 10.56

[CA INTER N07, 8 Mark]

Question: Mrs. Malini Hari shifted her industrial undertaking located in corporation limits of Faridabad,

to a Special Economic Zone (SEZ) on 1.12.2015.

The following particulars are available: ₹

(a) Land: Purchased on 20.01.2010 4,26,000

Sold for 22,00,000

(b) WDV of Building as on 01.04.2015 [Construction completed on 14.03.2010] 8,20,000

Sold for 11,89,000

(c ) WDV of cars as on 01.04.2015 7,40,000

Sold for 6,00,000

(d ) Expenses on shifting the undertaking 1,15,000

(e ) Assets acquired for the undertaking in the SEZ (on or before 25.06.2016) ₹

(i) Land 3,00,000

(ii) Building 5,00,000

(iii) Computers 1,00,000

(iv) Car 4,20,000

(v) Machinery (Second hand) 2,00,000

(vi) Furniture 50,000

There is no intention of investing in any other asset in this undertaking. Compute the exemption available

under Section 54GA for the assessment year 2016-17.

Answer: Computation of exemption available u/s 54GA of the Capital Gain:

Particular Land Building Cars

Sales consideration 22,00,000 11,89,000 6,00,000

Less Cost of acquisition (8,20,000) (7,40,000)

Indexed cost of requisition (Land) (

) 7,91,247

Gross Gain / Loss 14,08,753 3,69,000 (1,40,000)

(LTCG) (STCG) (STCL)

Less Exemption u/s 54 GA:

Total investment except furniture (include cost of shifting)

[3,00,000+5,00,000+1,00,000+4,20,000+2,00,000+1,15,000] = 16,35,000

12,66,000

3,69,000

Nil

Net Gain / Loss 1,42,753 Nil (1,40,000)

LTCG STCL

Page 61: Capital Gains [Income Tax]

Capital Gains 10.57

Income chargeable under the head Capital Gain:

LTCG (on Land) ₹1,42,753

Less STCL (on Car) (₹1,40,000)

LTCG ₹2,753

[CMA RTP D10]

Question: R, an individual resident in India bought 1,000 equity shares of ₹10 each of A Ltd. at ₹50 per

share on 30.5.2015. He sold 700 equity shares at ₹35 per share on 30.9.2015 and the remaining 300 shares

at ₹25 per share on 20.12.2015. A Ltd. declared a dividend of 50%, the record date being 10.8.2015. R sold

on 01.02.2016 a house from which he derived a long term capital gain of ₹75,000. Compute the capital

gain arising to ₹for the Assessment Year 2016-17.

Answer:

Assessee : Mr. R Previous Year : 2015-2016 Assessment Year : 2016-17

Analysis as per Section 94(7):

1. In the given case, shares were purchased on 30.05.2015 within 4 months prior to Record Date i.e.

10.08.2015.

2. Shares are sold on 30.09.2015 and on 20.12.2015. The first sale is within 3 months and the next one is

after that period. Dividend has been received @ 50% on Face Value ₹10 per share on the entire 1,000

shares.

3. Loss arising out of transfer of the first 700 shares should be ignored to the extent of such dividend.

Computation of Taxable Capital Gains

Particulars

Date of Transfer 30.09.15 20.12.15

No. of Shares 700 300

Sale price / share ₹35 ₹25

Consideration for transfer (₹) 24,500 7,500

Less Cost of Acquisition (₹) (35,000) (15,000)

Short Term Capital Loss (₹) (10,500) (7,500)

Add: Dividend (700 shares of Face Value ₹10 per share @ 50%) (₹) 3,500 Nil

[Transferred within 3 Months from Record Date]

Short Term Capital Loss [after considering S.94(7)] — (Note) (₹) (7,000) (7,500)

Total Short Term Capital Loss (₹) (14,500)

Long Term Capital Gains (Given) (₹) 75,000

Page 62: Capital Gains [Income Tax]

Income Tax 10.58

Taxable LTCG (after set off u/s 71) (₹) 60,000

Note: Since 300 shares are transferred on 20.12.15 which is 3 months after the reorder date, dividend

received need not be adjusted against Short Term Capital Loss.

[CMA RTP J10]

Question: ‘X’ received a vacant site under his father’s will. The value of the site on 31.3.2016 is ₹15 Lakhs.

As per terms of the ‘Will’ in the event ‘X’ wants to sell the site he should offer it to his brother for sale at

₹10 Lakhs. ‘X’, therefore, claims that the value of the site should be taken at ₹10 Lakhs as at 31.3.2016. Is

the claim correct?

Answer:

1. As per Rule 21 of Schedule III to the Act, the price or other consideration for which any property

may be acquired by or transferred to any person under the terms of a deed of trust or through or

under any restrictive agreement in any instrument of transfer shall be ignored for the purpose of

determining the value under the provisions of the Schedule.

2. In view of the above, the value of the site should be taken as ₹15 Lakhs and not as ₹10 Lakhs.

3. Therefore, claim of X is incorrect.

[CMA INTER D12, 1Mark]

Question: Unabsorbed loss under the head ‘Capital gains’ shall be carried forward for a period of ______

assessment years immediately following the assessment year in which such loss was incurred1.

[CMA INTER SY08, D13, 1 Mark]

Question: When entire net consideration has been invested by an individual towards subscription of

shares of an eligible company the exemption u/s. 54GB of the Income Tax Act 1961 would be2

a) NIL

b) 10% of capital gain

c) 50% of capital gain

d) 100% of capital gain

[CMA INTER SY08, D13, 1 Mark]

Question: Chapter VI–A deduction _______ (shall / shall not) be allowed in respect of income from short

term capital gain.3

1 Answer: 8 years

2 Answer: (d) 100% of capital gain

3 Answer: shall. However, in case it is short term capital gain in respect of transaction in equity shares in a company

chargeable to STT (u/s 111A) deduction under Chapter VIA shall not be allowed. The question does not mention the

type of transaction. Hence, both answers would be possible.

Page 63: Capital Gains [Income Tax]

Capital Gains 10.59

[CMA INTER SY08, D13, 1 Mark]

Question: A short-term capital loss in the current year can be set-off only against:1

a) Any capital gain in the current year

b) Any short-term capital gain only

c) Any long-term capital gain only

d) Any income under other heads of income

[CMA INTER SY12, D13, 1 Mark]

Question: Long-term capital gain arising from sale of listed shares in a recognized stock exchange (STT

paid) is exempt u/s _________ of the Income-tax Act, 19612:

a) 10(35)

b) 10(37)

c) 10(38)

d) 10(36)

1 Answer: (a)

2 Answer: (c)

Page 64: Capital Gains [Income Tax]

Income Tax 10.60

Additional CA Exam Questions

[CA INTER M08, 2 Marks]

Question: State true or false: Ms. Vasudha contends that sale of a work of art held by her is not

eligible to capital gains tax; is she correct?

Answer: No., Ms. Vasudha’s contention is not correct. As per the amendment made under the Finance

Act, 2007 the definition of capital asset [U/s 2(14) has got widen. Now, archaeological collections,

drawings, paintings, sculptures or any other work of an art are included within the meaning of the term

capital asset.

[CA INTER N08, 4 Marks]

Question: Answer the following with regard to the provisions of the Income-tax Act, 1961:

Mrs. X an individual resident woman wanted to know whether Income-tax is attracted on sale of gold

and jewellery gifted to her by her parent at the occasion of her marriage in the year 1979 which was

purchased at a total cost of ₹2,00,000?

Answer: Capital gains in the hands of Mrs. X - Long-term capital gains arising on transfer of gold and

jewellery is chargeable to tax in the hands of Mrs. X. Cost of acquisition will be taken as cost of the gold

and jewellery to her parents or the Fair Market Value of the asset on April 1, 1981, whichever is higher –

Long-term capital gain will be calculated after deducting indexed cost of acquisition.

[CA INTER N10, 4 Marks]

Question: How will you calculate the period of holding in case of the following assets?

(1) Shares held in a company in liquidation

(2) Bonus shares

(3) Flat in a co-operative society

(4) Transfer of a security by a depository (i.e., demat account)

Answer: Period of Holding S.2(42A) of the following asset:

1. Shares held in a company in liquidation: Period of holding will be taken from the date of acquisition

of shares to the date on which the company goes into liquidation:

2. Bonus Shares: The period of holding shall be taken from the date of allotment of shares till the date

preceding the date of transfer of shares.

3. Flat in a co-operative society: The period of holding shall be taken from the date of allotment of the

flat from the society to the date preceding the date of transfer of the flat.

4. Transfer of a security by a depository: Period of holding shall be calculated on FIFO basis. For

deciding FIFO technique the date of entry in ‘Demat Account’ is significant and the date of purchase

of security is irrelevant.

[CA INTER M08, 1 Mark]

Page 65: Capital Gains [Income Tax]

Capital Gains 10.61

Question: Fill up the blanks: Zero Coupon Bond means a bond on which no payment and benefits

are received or receivable before maturity or redemption.

Answer: True - As per section 2(48), ‘Zero Coupon Bond’ means a bond issued by any infrastructure

capital company or infrastructure capital fund or a public sector company on or after 1 st June 2005, in

respect of which no payment and benefit is received or receivable before maturity or redemption

from such issuing entity and which the Central Government may notify in this behalf.

[CA INTER M06, 2 Marks]

Question: State True or False: Zero coupon bonds of eligible corporation, held for more than 12 months,

will be long-term capital assets.

Answer: True. W.e.f. A.Y. 2007-08 zero coupon bonds held for more than 12 months shall be long term

capital assets.

[CA INTER J09, 2 Marks]

Question: State True or False, with reasons: Alienation of a residential house in a transaction of

reverse mortgage under a scheme made and notified by the Central Government is treated as

"transfer" for the purpose of capital gains.

Answer: False. As per clause (xvi) inserted by the Finance Act, 2008 in section 47, such alienation in a

transaction of reverse mortgage under a scheme made and notified by the Central Government is not

regarded as "transfer" for the purpose of capital gains.

[CA INTER M06, 2 Marks]

Question: State True or False, with reasons: Zero coupon bonds of eligible corporation, held for

more than 12 months, will be long-term capital assets.

Answer: True. U/s 2(42A), zero coupon bond held for not more than 12 months will be treated as a

short-term capital asset. Consequently, such bond held for more than 12 months will be a long-term

capital asset.

[CA INTER M06, 2 Marks]

Question: State True or False, with reasons: In the case of a dealer in shares, income by way of

dividend is taxable under the head "Profits and gains of business or profession".

Answer: False. In the case of all assesses, dividend income is taxable under the head "Income from

other sources", in view of the provisions of section 56(2)(i).

[CA INTER N05, 2 Marks]

Question: State True or False, with reasons: Where an urban agricultural land owned by an

individual, continuously used by him for agricultural purposes for a period of two years prior to the

Page 66: Capital Gains [Income Tax]

Income Tax 10.62

date of transfer, is compulsorily acquired under law and the compensation is fixed by the State

Government, resultant capital gain is exempt.

Answer: False. In this case, the compensation has been fixed by the State Government and hence

the exemption will not be available.

As per section 10(37), where an individual owns urban agricultural land which has been used for

agricultural purposes for a period of two years immediately preceding the date of transfer, and the

same is compulsorily acquired under any law and the compensation is determined or approved by

the Central Government or the Reserve Bank of India, resultant capital gain will be exempt.

[CA INTER M06, 4 Marks]

Question: X is in possession of agricultural land situated within urban limits, which is used for

agricultural purposes during the preceeding 3 years by his father. On 4.4.2015, this land is

compulsorily acquired by the Government of India on a compensation fixed and paid by it for ₹10

lakhs. Advise X as to the tax consequences, assuming that the entire amount is invested in purchase

of shares.

Answer: Section 10(37) exempts the capital gains arising to an individual or a Hindu Undivided

Family from transfer of agricultural land by way of compulsory acquisition, or a transfer, the

consideration for which is determined or approved by the RBI or the Central Government.

Such exemption is available where the compensation or the enhanced compensation or

consideration, as the case may be, is received on or after 1 st April, 2004 and the land has been used

for agricultural purposes during the preceding two years by such individual or a parent of his or by

such Hindu undivided family.

Since all the above conditions are fulfilled in this case, X is entitled to exemption u/s 10(37) of the

entire capital gains arising on sale of agricultural land.

[CA INTER N10, 5 Marks]

Question: Mr. A is a proprietor of Akash Enterprises having 2 units. He transferred on 1.4.2015 his unit 1

by way of slump sale for a total consideration of ₹25 Lacs. The expenses incurred for this transfer were

₹28,000/-. His Balance Sheet as on 31.3.2015 is as under:

Liabilities Total Assets Unit 1 Unit 2 Total

Own Capital 15,00,000 Building 12,00,000 2,00,000 14,00,000

Revaluation Reserve (for building of unit 1) 3,00,000 Machinery 3,00,000 1,00,000 4,00,000

Bank loan (70% for unit 1) 2,00,000 Debtors 1,00,000 40,000 1,40,000

Trade creditors (25% for unit 1) 1,50,000 Other assets 1,50,000 60,000 2,10,000

Total 21,50,000 Total 17,50,000 4,00,000 21,50,000

Other information:

Page 67: Capital Gains [Income Tax]

Capital Gains 10.63

1. Revaluation reserve is created by revising upward the value of the building of unit 1.

2. No individual value of any asset is considered in the transfer deed.

3. Other assets of unit 1 include patents acquired on 1.7.2013 for ₹50,000/- on which no depreciation has

been charged.

Compute the capital gain for the assessment year 2016-17.

Answer: Capital gain on transferee of unit 1 by way of slump sale u/s 50B

Amount (₹ )

Sale proceeds 25,00,000

Less Expenses of transfer 28,000

Net consideration proceeds 24,72,000

Less Cost of acquisition or improvement (See Note 1) 12,50,625

Long term Capital Gain 12,21,375

Note 1: Cost of acquisition or improvement = Net worth of the undertaking

Net worth of the undertaking

Aggregate value of total assets

Depreciable Assets

Building excluding revaluation reserve 9,00,000

Machinery 3,00,000

Patents [Note 2] 28,125 12,28,125

Other Assets

Debtors 1,00,000

Other assets except patent 1,00,000 2,00,000

14,28,125

Value of Liabilities

Bank loan (200000 x 70%) 140,000

Trade creditors (150000 x 25%) 37,500 1,77,500

Net worth 12,50,625

Note 2: Since patents have been acquired 1.7.2013 therefore depreciation @ 25% p.a. will be charged for

two years on WDV basis. Therefore book values as on 31.3.2015 = 50000 x .75 x .75 = ₹28125.

Note 3: For the purpose of computation of net worth, the WDV determined as per section 43(6) has to be

considered in the case of depreciable asset. The said problem has been solved assuming that B/S value of

₹3 lakhs and ₹9 lakhs (₹12 lakhs – 3 lakhs) represents the written down value of machinery & building

respectively, of unit 1.

Page 68: Capital Gains [Income Tax]

Income Tax 10.64

[CA INTER N09, 6 Marks]

Question: Compute the net taxable capital gains of Smt. Megha on the basis of the following information:

A house was purchased on 1.05.2003 for ₹4,50,000 and was used as a residence by the owner. The owner

had contracted to sell this property in June, 2013 for ₹10 lacs and had received an advance of ₹70,000

towards sale. The intending purchased did not proceed with the transaction and the advance was

forfeited by the owner. The property was sold in April, 2015 for ₹15,00,000. The owner, from out of sale

proceeds, invested ₹4 lacs in a new residential house in January, 2016.

Answer: Computation of net taxable capital gains of Smt. Megha for the A.Y. 2016-17

Particulars ₹

Full value of consideration 15,00,000

Less Expenses on transfer Nil

Net sale consideration 15,00,000

Less Indexed cost of acquisition (

) 8,87,214

Long term capital gain 6,12,786

Less Exemption under S.54 4,00,000

Taxable long term capital gain 2,12,786

[CA INTER M02, 8 Marks]

Question: Mr. Sunder furnishes the following particulars for the previous year ending 31.3.2015, and

requests you to compute the taxable capital gain:

(i) He had a residential house, inherited from father in 1996 the fair market value of which as on 1.4.81 is

₹5 lakhs.

(ii) In the year 1997-98, further construction and improvements cost ₹6 lakhs.

(iii) On 10.5.2015, the house was sold for ₹80 lakhs. Expenditure in connection with transfer ₹50,000.

(iv) On 20.12.2015, he purchased a residential house for ₹15 lakhs.

Answer: Computation of Capital Gains:

Sale proceeds 80,00,000

Less Indexed cost of acquisition *

+ 54,05,000

Less Indexed cost of improvement *

+ 19,59,517

Less Expenses of transfer 50,000

6,35,483

Less Exemption u/s 54 6,35,483

Page 69: Capital Gains [Income Tax]

Capital Gains 10.65

Long term capital gain NIL

The following assumption has been made:

(a) Mr. Sunder’s father purchased the property before April 1, 1981.

(b) Cost of the house to X’s father was less than ₹5,00,000.

[CA INTER N03, 4 Marks]

Question: Ms.Vimla sold a residential building at Jodhpur for ₹15,00,000 on 01-07-2015.

The building was acquired for ₹1,50,000 on 01-06-2000. She paid brokerage @ 2% at the time of sale of

the building. She invested ₹7 lakhs in purchase of a residential building in December 2014 and

deposited ₹2 lakhs in NABARD Capital Gains Bond in March, 2016. Compute her taxable capital

gain.

Answer: Computation of taxable capital gain of Ms. Vimla for A.Y.2016-17

Particulars ₹

Sale price of residential building 15,00,000

Less Brokerage @ 2% 30,000

Net consideration 14,70,000

Less Indexed cost of acquisition [1,50,000 x 1081 / 406] 3,99,384

10,70,616

Less Deduction u/s.54 for purchase of new residential house in Dec. 2013 7,00,000

Taxable long term capital gain 3,70,616

Note – One of the conditions for claiming deduction u/s 54EC for the investment in REC/NHAI

Capital Gains bonds is that the deposit should be made within 6 months from the date of transfer. In

this case, the transfer took place on 1.7.15 and the 6 months period within which the deposit should

be made for the purpose of section 54EC would expire by 1.1.16. The investment in REC/NHAI

Capital Gains bonds was made only in March 2016. Therefore, the assessee is not eligible for section

54EC deduction.

[CA INTER M04, 4 Marks]

Question: Bala sold his vacant site on 21.09.2015 for ₹7,00,000. It was acquired by him on 01.10.1999

for ₹1,50,000.

The State stamp valuation authority fixed the value of the site at the time of transfer @ ₹13,00,000.

Compute capital gains in the hands of Bala and give your reasons for computation.

Answer:

Page 70: Capital Gains [Income Tax]

Income Tax 10.66

Computation of capital gains of Shri Bala for the A.Y.2016-17 ₹

Deemed sale consideration as per section 50B 13,00,000

Less Indexed cost of acquisition (

) 4,16,838

Taxable long term capital gain 8,83,162

Note: According to section 50C(1), where the consideration received or accruing as a result of the

transfer of land or building or both is less than the value adopted or assessed by the State stamp

valuation authority for the purpose of payment of stamp duty in respect of such transfer, then the

value so adopted or assessed by the State stamp valuation authority shall be deemed to be the full

value of the consideration received or accruing as a result of the transfer.

In this case, since the consideration of ₹7,00,000 received on transfer of land is less than the value of

₹13,00,000 fixed by the State stamp valuation authority, the value adopted by the State stamp

valuation authority is deemed to be the full value of consideration and capital gains is calculated

accordingly.

[CA INTER N01, 6 Marks]

Question: Arjun furnishes the following particulars and requests your advise as to the liability to capital

gains for the Assessment Year 2016-17:

(i) Jewellery purchased by his on 10.3.2001 for ₹1,05,000 was sold by him for a consideration of ₹3,15,000

on 2.11.2015.

(ii) He incurred expenses:

(a) At the time of purchase ₹2,000

(b) At the time of sale (for brokerage) ₹4,000

(iii) He invested ₹50,000 in bonds with National Highway Authority of India out of sale consideration.

On these facts:

(a) Compute the capital gains chargeable to tax.

(b) Whether Arjun would be entitled to any exemption?

Answer: Computation of Capital Gains for Mr. Arjun for Assessment Year 2016-17.

Particulars ₹

Sale Consideration 3,15,000

Less Indexed Cost of Acquisition * ( )

+ 2,84,894

Less Expense on transfer 4,000

Long Term Capital Gain 26,106

Less Exemption u/s 54EC Investment in bonds of NHAI 26,106

Taxable Long Term Capital Gain Nil

Page 71: Capital Gains [Income Tax]

Capital Gains 10.67

[CA INTER M01, 10 Marks]

Question: Mr. A sold shares of a public limited company for ₹5,00,000 on 1.10.2015, which had been

acquired by him in October, 1990 for ₹50,000. He wants to utilize the said amount of sale consideration

for purchase or construction of a new residential house. He already owns one residential house at the

time of sale of the shares i.e., on 1.10.2015. He has deposited ₹4,00,000 under the Capital Gains Deposit

Account scheme with a specified Bank on 30.4.2015. Ascertain the Capital gain taxable in A’s hands for

Assessment Year 2016-17 and advice him as to what further action he has to take; to avail of the

exemption.

Answer: If A has transferred these share in a recognized stock exchange in India he will be subject to

securities transaction tax and consequently, the long term capital gain will be exempt from tax. Otherwise

if these shares are transferred outside a recognized stock exchange the manner of income computation

will be as under:

Sales Consideration 5,00,000

Less Indexed cost of acquisition *

+ 3,14,244

Long term capital gain 1,85,756

Less Exemption under Section 54F *

+ 1,48,605

Long term capital gain 37,151

[CA INTER N08, 12 Marks]

Question: Paulomi has transferred 1,000 shares of Hetal Ltd., (which she acquired at a cost of ₹10,000

in the financial year 2009-10) to Dhaval, her brother, at a consideration of ₹3,00,000 on 15.5.2015

privately.

During the financial year 2015-16, she has paid through e-banking ₹15,000 towards medical

premium, ₹50,000 towards L.I.P. and ₹25,000 towards PPF.

Assuming she has no other source of income, compute her total income and tax payable for the

Assessment Year 2016-17.

Answer: Computation of total income and tax liability of Paulomi for Assessment year 2016-17

Particulars ₹

Sale consideration 3,00,000

Less Indexed cost of acquisition *

+ 17,104

Long term capital gain 2,82,896

Total income 2,82,896

Tax liability ₹

Income-tax @ 20% on ₹32,896 [₹2,82,896 – ₹2,50,000] 6,579

Page 72: Capital Gains [Income Tax]

Income Tax 10.68

Less Rebate u/s 87A 2,000

4,579

Add Education Cess @ 2% 91

Secondary and Higher Education Cess @ 1% 46

Total tax liability 4,716

Notes

1. As per section 112 of Income-tax Act, 1961, deductions under Chapter VI-A are not allowable

against long term capital gain. Therefore, Paulomi is not entitled to deduction u/s 80C in respect

of payment of LIP and contribution to PPF. She is also not entitled to deduction u/s 80D in

respect of medical insurance premium paid by her.

2. Since Paulomi has not transferred her shares through the Stock Exchange and, therefore, has not

paid securities transaction tax, she is not entitled to claim exemption u/s 10(38) in respect of long

term capital gain.

3. She is, however, entitled to reduce the long-term capital gain by the unexhausted basic

exemption limit and pay tax on the balance @20% as per section 112. In this case, since she has no

other source of income, the entire basic exemption limit of ₹2,50,000 can be reduced from the

long-term capital gain and rebate u/s 87A ₹2,000 also availed. It is presumed that Paulomi is a

resident assessee.

[CA IPCC N12, 8 Marks]

Question: Mr. C inherited from his father 8 plots of land in 1980. His father had purchased the plots in

1960 for ₹5 lakhs. The fair market value of the plots as on 1-4-1981 was ₹8 lakhs. (₹1 lakh for each plot)

On 1st June 2001, C started a business of dealer in plots and converted the 8 plots as stock in trade of his

business. He recorded the plots in his books and ₹45 lakhs being the fair market value on that date. In

June 2005, C sold the 8 plots for ₹50 lakhs.

In the same year he acquired a residential house property for ₹45 lakhs. He invested an amount of ₹5

lakhs in construction of one more floor in his house in June 2006. The house was sold by him in June 2015

for ₹63,50,000/-

The valuation adopted by the registration authorities for charges of stamp duty was ₹86,50,000. As per

the assessee’ request the assessing officer made a reference to a valuation officer. The value determined

by the valuation officer was ₹87,20,000. Brokerage of 1% of sale consideration was paid by C.

Give the tax computation for the relevant assessment years with reasoning.

Computation of total income and tax liability of Mr. C for A.Y. 2016-17 ₹

Capital Gains on sale of residential house property

Sale consideration (S.50C WEL of (WEH of Sale value or Stamp duty) 86,50,000

Page 73: Capital Gains [Income Tax]

Capital Gains 10.69

or value as per valuation officer)

Less Brokerage @ 1% of sale consideration 63,500

Net Sale consideration 85,86,500

Less Indexed cost of acquisition *

+ 97,87,726

Indexed cost of improvement

10,41,426

1,08,29,152

Less Long-term capital Loss (Total Income) 22,42,652

Tax on total income Nil

[CA INTER M98, 6 Marks]

Question: In April 2003, S subscribed to the first issue of equity capital of a public limited company (face

value of each share was ₹100) to the extent of ₹25,000. In 2011, the company converted the face value of its

shares from ₹100 to ₹10 each. Half of the holdings of the shares held by S was sold by him in October,

2015 for ₹50,000. S had to pay a brokerage of 2% on sale.

What is the nature of gains realized and compute the same.

Answer:

Computation of Capital gain ₹

Sale proceeds of half holding 50,000

Less Indexed cost of acquisition [₹12,500 × 1081/463] 29,185

Less Brokerage (2% of 50,000) 1,000

Long term Capital gain 19,815

[CA IPCC N11, 8 Marks]

Question: Mr. Selvan, acquired a residential house in January, 2001 for ₹10,00,000 and made some

improvements by way of additional construction to the house, incurring expenditure of ₹2,00,000 in

October, 2005. He sold the house property in October, 2015 for ₹75,00,000. The value of property was

adopted as ₹80,00,000 by the State stamp valuation authority for registration purpose. He acquired a

residential house in January, 2016 for ₹25,00,000. He deposited ₹20,00,000 in capital gains bonds issued by

National Highway Authority of India (NHAI) in June, 2016. Compute the capital gain chargeable to tax

for the assessment year 2016-17.

What would be the tax consequence and in which assessment year it would be taxable, if the house

property acquired in January, 2016 is sold for ₹40,00,000 in March, 2018 ?

Answer: Computation of Capital Gains Chargeable to tax for A.Y. 2016-17.

Particulars ₹

Page 74: Capital Gains [Income Tax]

Income Tax 10.70

Sale consideration (S.50C WEH of CA or Stamp Duty Value) 80,00,000

Less: Indexed Cost of Acquisition

26,62,562

Indexed Cost of Improvement

4,35,010

49,02,428

Less: Capital Gains exemption u/s 54 25,00,000

Less Capital gains exemption u/s 54EC is available only if deposited within 6 months.

In this case deposit made after 6 months is disqualified

Nil

Taxable Capital Gains 24,02,428

Taxable in AY 2018-19, if newly purchased

HP deducted u/s 54 sold within 3 years

₹ ₹

Sale consideration 40,00,000

Less Cost of acquisition 25,00,000

Capital gains exempted earlier (25,00,000) Nil

Short-term Capital Gains 40,00,000

[CA INTER M09, 8 Marks]

Question: Mr. Kumar is the owner of a residential house which was purchased in September 1997 for

₹5,00,000. He sold the said house on 5th August, 2015 for ₹24,00,000. Valuation as per stamp valuation

authority of the said plot of land was ₹45,00,000. He invested ₹8,00,000 in NHAI Bonds on 12th January

2016. He purchased a residential house on 8th September, 2015 for ₹12,00,000. He gives other particulars

as follows:

Interest on Bank Deposit ₹32,000

Investment in public provident fund ₹12,000

You are requested to calculate the taxable income for the assessment year 2016-17 and the tax liability, if

any.

Answer: Computation of taxable income

Sales consideration 45,00,000

Less Indexed cost of acquisition *

+ 16,32,931

Long-term capital gain 28,67,069

Less Exemption under section 54 12,00,000

Exemption u/s 54EC not available

as invested after 6 months from the date of sale

Nil

Page 75: Capital Gains [Income Tax]

Capital Gains 10.71

Taxable long-term capital gain 16,67,069

Income from other sources 32,000

Gross total income 16,99,069

Less Deduction under section 80C 12,000

Taxable income (rounded off) 16,87,069

[CA IPCC M15, 8 Marks]

Question: Ms. Mohini transferred a house to her friend Mr. Ragini for ₹3500000 on 01-10-2015. The Sub

Registrar valued the land at ₹4800000. Ms. Mohini contested the valuation and the matter was referred to

Divisional Revenue Officer, who valued the house at ₹4100000. Accepting the said value, differential

stamp duty was also paid and the transfer was completed.

The total incomes of Mohini and Ragini for the assessment year 2016-17, before considering the transfer

of said house are ₹280000 and ₹345000, respectively. Ms. Mohini had purchased the house on 15th May

2010 for ₹2500000 and registration expenses were ₹150000.

You are required to explain provisions of Income-tax Act, 1961 applicable to present case and also

determine the total income of both Ms. Mohini and Ms. Ragini taking into account the above said

transactions.

Answer:

Total income of Ms. Mohinin ₹

Sale Consideration 41,00,000

Less: Indexed COA

40,29,044

70,956

Other Income 2,80,000

Capital Gain 3,50,956

Total Income of Ms. Ragini

Income from other Sources

Less Immovable Property for inadequate consideration (41 lac – 35 lac) 6,00,000

Other Income 3,45,000

9,45,000

[CA INTER M05, 6 Marks]

Question: Mr. Y submits the following information pertaining to the year ended 31st March, 2016:

Page 76: Capital Gains [Income Tax]

Income Tax 10.72

(1) On 30.11.2015, when he attained the age of 60, his friends in India gave a flat at Surat as a gift, each

contributing a sum of ₹20,000 in cash. The cost of the flat purchased using the various gifts was ₹3.4

lacs.

(2) His close friend abroad sent him a cash gift of ₹25,000 through his relative, for the above occasion.

(3) Mr. Y sold the above flat on 30.1.2016 for ₹3.6 lacs. The Registrar’s valuation for stamp duty purposes

was ₹3.7 lacs. Neither Mr. Y nor the buyer, questioned the value fixed by the Registrar.

(4) He had purchased some equity shares in X Pvt. Ltd. on 5.2.2014 for ₹3.5 lacs. These shares were sold

on 15.3.2016 for ₹2.8 lacs.

You are requested to calculate the total income of Mr. Y for the assessment year 2016-17.

Answer: Computation of Total Income

Gift of Surat Flat (WN 1) 3,40,000

Add Cash gift from Relative (WN 2) ------

Add Short term capital gain (WN3) 30,000

Add Long term capital loss (WN4) (1,22,929)

3,70,000

Less Deduction under chapter VI-A Nil

Total Income 3,70,000

Notes:

1. It is now taxable as it is a gift-in-kind received on or after 1st October 2013.

2. Gift received from unrelated person is not taxable if the amount received from each such unrelated

person is less than ₹50000 per annum.

3. Computation of short term capital gains [on Sale of Flats]

4. Computation of long term capital gains [on sale of Equity Shares of X Private ltd.]

Sale consideration 2,80,000

Less Indexed cost of acquisition *

+ 4,02,929

Long term capital loss 1,22,929

Note: STT is not applicable as shares are of a private co.

Shares held for not more than 12 months immediately preceding the date of transfer are short-term

capital assets as per Section 2(42A). In this case, shares have been held for more than 12 months and

Sale Consideration S.50C (WEH of Stamp duty value or sale consideration) 3,70,000

Less Cost of acquisition [cost to Previous owner as per Sec. 49(1)(iii)] 3,40,000

Short term capital gains 30,000

Page 77: Capital Gains [Income Tax]

Capital Gains 10.73

therefore it constitute a LTCA. The loss arising from sale of such shares, is therefore a long-term capital

loss. As per Section 70(3), long term capital loss can be set-off only against long-term capital gains. So,

long-term capital loss cannot be set-off against short-term capital gains. But, such long-term capital loss

can be carried forward to the next year for set-off against long-term capital gains arising in that year &

they can be carried forward for a maximum of 8 assessment years.

[CA INTER M10 & N13, 6 & 4 Marks (Similar in N13 Vaibhav & Dhanush instead of Rajkumar &

Dhuruv)]

Question: Mr. Rajkumar sold a house to his friend Mr. Dhuruv on 1st November 2015 for a consideration

of ₹25,00,000. The Sub-Registrar refused to register the document for the said value, as according to him,

stamp duty had to be paid on ₹45,00,000, which was the Government guideline value. Mr. Raj Kumar

preferred an appeal to the Revenue Divisional Officer, who fixed the value of the house as ₹32,00,000

(₹22,00,000 for land balance for building portion). The differential stamp duty was paid, accepting the

said value determined. Assuming that the fair market value is ₹32,00,000, what are the tax implications in

the hands of Mr. Raj Kumar and Mr. Dhuruv for the assessment year 2016-17? Mr. Raj Kumar had

purchased the land on 1st June 2012 for ₹5,19,000 and completed the construction of house on 1st October,

2013 for ₹14,00,000.

Answer: In the hands of the seller, Mr. Raj Kumar

As per section 50C(1), where the consideration received or accruing as a result of transfer of land or

building or both, is less than value adopted or assessed or assessable by the stamp valuation authority,

the value of adopted or assessed or assessable by the stamp valuation authority shall be deemed to be the

full value of consideration received or accruing as a result of transfer.

Where the assessee appeals against the stamp valuation and the value is reduced in appeal by the

appellate authority (Revenue Divisional Officer, in this case), such value will be regarded as the

consideration received or accruing as a result of transfer.

In the given problem, land has been held for a period exceeding 36 months and building for a period less

than 36 months immediately preceding the date of transfer. So land is a long-term capital asset, while

building is a short-term capital asset.

Particulars ₹

Long term capital gain on sale of land

Consideration received or accruing as a result of transfer of land 22,00,000

Less Indexed cost of acquisition (

) 6,58,496

Long-term capital gain (A) 21,99,352

Short-term capital loss on sale of building

Consideration received or accruing from transfer of building 10,00,000

Less Cost of acquisition 14,00,000

Short term capital loss (B) 4,00,000

Page 78: Capital Gains [Income Tax]

Income Tax 10.74

As per section 70, short-term capital loss can be set-off against long term capital gains. Therefore, the net

taxable long-term capital gains would be ₹17,99,342 (i.e.) [₹21,99,352 – 4,00,000].

In the hands of the buyer Mr. Dhuruv.

As per Section 56 (2)(vii), inserted by the Finance Act, 2009, where an individual receives from a non-

relative, any immovable property for inadequate consideration, and the difference between the stamp

value (or the value reduced by the appellate authority, as in this case) and the consideration exceeds

₹50,000, such difference is chargeable to tax as income from other sources. The problem states that the

immovable property in question was received on 1st November, 2014 from a non- relative. Hence the

above provisions will be attracted. The inadequate consideration to the tune of ₹7 lacs (32 lacs less 25 lacs)

will be assessed as income from other sources.

[CA IPCC N15, 8 Marks]

Question: Mr. Martin sold his residential house property on 08-06-2015 for 70 lakhs which was purchased

by him for 20 lakhs on 05-05-2005. He paid ₹1 lakh as brokerage for the said property. The stamp duty

valuation assessed by sub registrar was ₹80 lakhs.

He bought another house property on 25-12-2015 for 15 lakhs.

He deposited 10 lakhs on 10-11-2015 in the capital gain bond of National Highway Authority of India

(NHAI).

He deposited another 10 lakhs on 10-07-2016 in the capital gain deposit scheme with SBI for construction

of additional floor of house property.

Compute income under the head ‚Capital Gains‛ for A.Y. 2016-17 as per Income Tax Act, 1961 and also

Income tax payable on the assumption that he has no other income chargeable to tax.

[CA INTER N08, 2 Marks]

Question: State with reason, true or false: capital gain of ₹75 lacs arising from transfer of long term capital

assets will be exempt from tax if such capital gain is invested in the bonds redeemable after three years, issued by

NHAI u/s 54EC.

Answer: True. as per Sec. 54EC

1. The assets transfer should be a long term assets.

2. Within a period of 6 months the capital gain must be invested in long term asset for a locking period

of 3 years in a National highway authority of India.

[CA INTER M05, 1 Mark]

Question: Choose the correct answer: In respect of shares held as investment, while computing the

capital gains, securities transaction tax paid in respect of sale of listed shares sold in a recognized stock

exchange,1

1 D - Is not deductible at all

Page 79: Capital Gains [Income Tax]

Capital Gains 10.75

(a) Is deductible upto ₹1,00,000

(b) Is deductible upto ₹2,00,000

(c) Is deductible where capital gains is below ₹5,00,000

(d) Is not deductible at all.

[CA INTER M05, 1 Marks]

Question: Choose the correct answer: For an individual who has derived short-term capital gains of

₹40,000 from transfer of listed equity shares after 1.10.2010, with other income of ₹20,000 (these two items

above making up his total income), income-tax payable in respect of short-term capital gains is1

(a) ₹4000

(b) ₹8000

(c) ₹1000

(d) None of the above.

[CA INTER N05, 1 Mark]

Question: Fill in the blanks having regard to the provisions of the Income-tax Act, 1961: Where a person

transfers capital asset to a firm in which he becomes partner, the full value of consideration in the context

of capital gain computation, will be _______

Answer: The amount recorded in the books of account of the firm as the value of the capital asset.

[CA INTER M11 …… Marks]

Question: Mr. Rakesh purchased a house property on 14th April 1979 for ₹1,05,000. He entered into an

agreement with Mr. B for the sale of house on 15th September, 1983 and received an advance of ₹25,000.

However, since Mr. B did not remit the balance amount, Mr. Rakesh forfeited the advance.

Later on, he gifted the house property to his friend Mr. a on 15th June 1987.

Following renovations were carried out by Mr. Rakesh and Mr. A to the house property.

1 D - None of the above

Page 80: Capital Gains [Income Tax]

Income Tax 10.76

Additional CMA Exam Questions

CMA INTER SY12, D13, 5 Marks

Question: Chirag, an individual, purchased 5,000 shares of X. Limited at ₹50 per share and 4,000 shares of

Y. Limited at ₹60 per share in the previous year 2010-11 and held them as capital assets. In the previous

year 2012-13, he converted the shares into his stock-in-trade. The fair market value of the shares of both

the companies on the date of conversion was ₹300 per share.

In the previous year 2015-16, he sold the shares of the two companies at ₹380 per share. Shares were sold

by way of private sale and hence securities transactions tax was not payable. Ascertain chargeable capital

gain and business income from the above-noted transactions in the hands of Chirag.

Answer: Computation of Income for Assessment Year 2016- 2017

Capital Gain ₹

Fair Market value of Y Ltd. on the date of conversion 9,000 x ₹300 27,00,000

Less Indexed cost of Acquisition (₹2,50,000 + ₹2,40,000) x 852/711 5,87,173

Long term capital gain 21,12,827

Business Income

Sale proceeds of Shares (9,000 x ₹380) 34,20,000

Less Fair market Value of Shares on the date of conversion 27,00,000

Business Income 7,20,000

Note: As securities transaction tax was not paid, exemption u/s 10(38) in respect of long term capital gain

is not available.

[CMA INTER SY08, J14, 1 Mark]

Question: The investment made in the long term specified asset by an assessee to claim exemption u/s

54EC from capital gain tax cannot exceed ₹in lakhs1

a) 10

b) 15

c) 25

d) 50

[CMA INTER SY08, J14, 3 Marks]

Question: Mr. Sanjay running a proprietary business by name "Kannan Departmental Store" wants to

convert it into a company. State the conditions to be satisfied by Mr. Sanjay for availing exemption from

capital gains tax by treating the transaction not being regarded as "transfer".

1 Answer: (d)

Page 81: Capital Gains [Income Tax]

Capital Gains 10.77

Answer: Mr. Sanjay must satisfy the conditions contained in section 47(xiv) in order to be eligible for

exemption in respect of conversion of proprietary concern into company. The conditions are –

1. All the assets and liabilities of the sole proprietary concern relating to the business immediately

before the succession must become the assets and liabilities of the successor company.

2. The shareholding of the proprietor relating to the business must not be less than 50% of the total

voting power and it should continue to remain so for a period of five years from the date of

succession; and

3. The sole proprietor (Sanjay in this case) must not receive any consideration or benefit, directly or

indirectly, in any form or manner, other than by way of allotment of shares in the company.

[CMA INTER SY08, J14, 1 Mark]

Question: Where any land is located within aerial distance of 7 kms from municipal limits, to be regarded

as capital asset u/s 2(14), the population of the municipality as per last census done before 1.3.201 should

be more than1

a) 9 lacs

b) 8 lacs

c) 10 lacs

d) None of these

[CMA INTER SY08, D14, 1 Mark]

Question: In order to avail exemption u/s 54EC of the Income-tax Act, 1961, the time limit for making

investment in specified bonds from the date of transfer of long-term capital asset is2

a) 12 months

b) 36 months

c) 60 months

d) 6 months

[CMA INTER SY08, D14, 1 Mark]

Question: When an asset owned by father for 10 years is inherited by son in June, 2015 and is

subsequently sold in December, 2015, the resultant capital gain is _________ in nature3.

[CMA INTER SY08, D14, 1 Mark]

Question: When a factory building owned by an individual used for business purposes is sold after 48

months from the date of purchase, no other building being owned by the assessee, the resultant capital

gain is _________ in nature1.

1 Answer: (c)

2 Answer: (d)

3 Answer: Long term

Page 82: Capital Gains [Income Tax]

Income Tax 10.78

[CMA INTER S12 & SY08, J14, 9 Marks]

Question: Mr. Ahuja purchased one plot of land in 1996-97 at a cost of ₹1,00,000 in Delhi. The land was

held by him as capital asset. He converted the plot into his stock in trade on 1st April, 2013, on which date

the fair market value of the plot was ₹15,50,000.

He started constructing a building consisting of eight flats of equal size and dimension on the plot on 1st

April, 2013. Cost of construction of each flat is ₹6,00,000. Construction was completed in June 2014. He

sold five flats at ₹12,00,000 per flat from June, 2015 to March, 2016. The remaining three flats were held as

stock on 31st March, 2016.

Compute Capital Gain and Business Income arising from above transactions for Assessment year 2016-17.

Answer: Computation of Capital Gain for Assessment Year 2016-17

Particulars ₹

Proportionate Fair market value of land-deemed consideration u/s 45(2): ₹15,50,000 x 5 /

8 (Note 1) 9,68,750

Less Proportionate indexed cost of acquisition (

) (Note 2) 1,92,418

Long term capital gain 7,76,332

Computation of Business Income for Assessment Year 2016-17

Particulars ₹ ₹

Sale proceeds of 5 flats : ₹12,00,000 x 5 60,00,000

Less Proportionate fair market value of land 9,68,750

Cost of construction (₹6,00,000 x 5) 30,00,000 39,68,750

Business Income 20,31,250

Notes:

1. Capital gain arising on conversion of capital asset into stock-in-trade is taxed in the year in which the

converted asset is transferred. Therefore, capital gain in respect of proportionate land for 5 flats is

chargeable to tax in assessment year 2016-17.

2.Although tax liability for capital gain is taxed in the year of actual transfer, cost inflation index of the

year in which capital asset is converted is to be used for determining capital gain, as transfer u/s 2(47) is

recognized in the year of conversion.

[CMA INTER SY08, J14, 6 Marks]

Question: Dev is engaged in the business of real estate. During the previous year 2014-15, he sold a flat

forming part of his stock-in-trade to Jeet for ₹45 lakhs. The value assessed by the Stamp Valuation

Authority for stamp duty purpose is ₹55 lakhs. Jeet holds the flat as capital asset.

1 Answer: Short-term

Page 83: Capital Gains [Income Tax]

Capital Gains 10.79

State with reasons the implications of the above transaction on taxable income of Dev and Jeet for the

Assessment Year 2015-16.

Answer: Implication in the hands of Dev: As the flat sold by Dev formed part of his stock-in-trade, the

provision of section 43CA shall apply.

As per section 43CA where consideration as a result of transfer by an assessee of any immovable asset

(other than a capital asset) is less than the stamp duty value, then the stamp duty value shall be deemed

to be the full value of the consideration for the purpose of computing business profit from transfer of

such asset.

Therefore, the difference between the stamp duty value and the actual consideration i.e. ₹10 lacs (₹55 lacs

– ₹45 lacs) shall be charged to tax under the head ‚profits and gains from business or profession‛.

Implication in the hand of Jeet: As per section 56(2)(vii) if any individual receive any immovable

property (being capital asset) for a consideration which is less than its stamp duty value by more than

₹5000, then the excess of stamp duty value over the actual consideration shall be taxed under the head

‚income from other sources‛

Therefore, the amount of ₹10 lacs, being the excess of stamp duty value over the actual consideration

shall be taxed in the hands of Jeet under the head ‚income from other sources‛.

[CMA INTER SY12, J14, 1 Mark]

Question: Choose the best option: To avail exemption u/s 54, an individual should purchase a new

residential house within ________ years from the date of sale.1

(A) 2 (B) 3 (C) 1 (D) 4

[CMA INTER SY12 & SY08, J14, 5 Marks]

Question: State the provisions relating to claiming of exemption in order to reduce tax liability on short-

term capital gains.

Answer: Provisions relating to claiming of exemption in order to reduce tax liability on short term capital

gains.

Applicable for Exemption U/S

1 Transfer of Agricultural land 54B

2 Transfer by way of compulsory acquisition by Government 54D

3 Shifting of industrial undertaking from urban to Rural area 54G

4 Compulsory acquisition of Agricultural land by Central Government / RBI 10(37)

5 Transfer by companies engaged in power sector business 10(41)

1 A

Page 84: Capital Gains [Income Tax]

Income Tax 10.80

[CMA INTER SY12, D14, 1 Mark]

Question: Fill up the blanks: Buy back of unlisted shares by a company is _______ even if it is incurred

in the course of shareholder1.

[CMA INTER SY08, D14, 1 Mark]

Question: Choose the best option: Basic rate of tax applicable to domestic company on distributed

income in the case of buyback of shares not listed in stock exchange is2

(A) 15% on the distributed income (B) 20% on the distributed income (C) 30% on the distributed income

(D) Nil

[CMA INTER SY08, D14, 4 Marks]

Question: State with brief reason, whether the following assets are long-term or short-term capital assets

(Mere conclusion will not receive any credit):

1. Renunciations of right to subscribe to right share, immediately after the offer was made by the

company.

2. Sale of surgery equipments by a doctor on discontinuance of profession.

3. Slump sale of undertaking offer operating for 4 years.

4. Sale of bonus shares within 6 months after the allotment; original shares were acquired 24 menthes e

earlier.

Answer:

1. As per clause (e) of the Explanation 1 to section 2(42A) right to subscribe to any financial asset when

renounced in favour of any other person, the period shall be reckoned from the date of offer of such

right by the company. Since the time between the offer made by the company and its renunciation in

no case could exceed 12 months, it is always a short-term capital asset.

2. Surgery equipment used by a surgeon in profession might have been allowed depreciation under

section 32. Hence, in view of section 50 it shall deemed to be a short-term capital asset and the gain or

loss if any sale be short-term capital gain or loss.

3. As per proviso to section 50B(1) an undertaking owned and held for not more than 36 months shall

be short-term capital asset and when the undertaking was operated for 4 years before slump sale, it

shall be a long-term capital asset.

4. As per clause (f) of the Explanation 1 to section 2(42A), a financial asset allotted without payment

shall be treated as long-term or short-term by reckoning the time period from the date of allotment of

such financial asset. Thus bonus shares sold within 6 months after allotment will be short-term

capital asset.

1 Exempt

2 (B) 20% on the distributed income

Page 85: Capital Gains [Income Tax]

Capital Gains 10.81

[CMA INTER SY12, J15, 1 Marks]

Question: Fill up the blanks: A Zero coupon bond is a long-term capital asset, if it is held for more than

______ months before transfer1.

[CMA INTER SY12, J15, 1 Marks]

Question: Choose the best option: Long-term capital gain on off-market sale of shares of a listed

company without availing of indexation benefit is taxed at 2

(A) 5% (B) 10% (C) 15% (D) 20%

[CMA INTER SY12, J15, 3 Marks]

Question: Can Mr. Ajit who has long-term capital gain from sale of vacant site in India buy a residential

house outside India to claim exemption under section 54F? Assume that he has no residential property in

India.

Answer: Exemption u/s 54F

When the assessee has no residential house property or has not more than one residential house

property, he is eligible to claim exemption under section 54F.

When a long-term capital asset other than a residential house is transferred, the exemption by way of

investment in residential house could be obtained by deploying the net consideration.

The exemption is subject to the condition that the assessee has within a period of one year before or

two years after the date of transfer of long-term capital asset acquires a residential house or three

years after transfer constructs a new residential house in India for the purpose of availing the

exemption under section 54F.

A person transferring a long-term capital asset in India cannot acquire a residential house outside

India and be eligible for exemption under section 54F.

Thus the person cannot claim exemption in respect of long-term capital gain on sale of vacant site in

India by acquiring a residential house outside India.

[CMA INTER SY12, J15, 5 Marks]

Question: Mr. Dhoni sold a residential building at Cochin for ₹65 lakhs in December 2015. The stamp

valuation authority determined the value at ₹80 lakhs which was not contested by Mr. Dhoni. The

property was acquired in April, 2005 for ₹6 lakhs. He acquired a residential flat at Ranchi for ₹55 lakhs

and another residential house at Cuttack for ₹25 lakhs before March, 2016. Compute the capital gain of

Mr. Dhoni for the assessment year 2016-17.

1 12 months

2 (B) 10%

Page 86: Capital Gains [Income Tax]

Income Tax 10.82

Answer: You are required to plan in such a way that the incidence of tax is the least.

Particulars

Sale consideration of residential building of Cochin 65,00,000

Stamp valuation authority has determined the value of ₹80 lakhs 80,00,000

The higher of the two is to be adopted as deemed sale consideration u/s 50C 80,00,000

Less Indexed cost of acquisition

13,05,030

Capital gains before exemption 66,94,970

Less Exemption u/s 54

In respect of residential property acquired of Ranchi 55,00,000

Taxable long-term capital gain 11,94,970

Note: From the assessment year 2016-17 onwards, the exemption u/s. 54 is limited to one residential

house in India. Out of the two residential properties acquired by him, he can opt for exemption u/s. 54 in

respect of one property only. The higher of the value has been adopted for maximizing the benefit to the

assessee.

Page 87: Capital Gains [Income Tax]

Capital Gains 10.83

Bare Act E.—Capital gains

Capital gains.

45. (1) Any profits or gains arising from the transfer of a capital asset effected in the previous year shall, save as

otherwise provided in sections 54, 54B, 54D, 54E, 54EA, 54EB, 54F, 54G and 54H, be chargeable to income-tax

under the head "Capital gains", and shall be deemed to be the income of the previous year in which the transfer took

place.

(1A) Notwithstanding anything contained in sub-section (1), where any person receives at any time during any

previous year any money or other assets under an insurance from an insurer on account of damage to, or destruction

of, any capital asset, as a result of—

(i) flood, typhoon, hurricane, cyclone, earthquake or other convulsion of nature; or

(ii) riot or civil disturbance; or

(iii) accidental fire or explosion; or

(iv) action by an enemy or action taken in combating an enemy (whether with or without a declaration of war),

then, any profits or gains arising from receipt of such money or other assets shall be chargeable to income-tax under

the head "Capital gains" and shall be deemed to be the income of such person of the previous year in which such

money or other asset was received and for the purposes of section 48, value of any money or the fair market value of

other assets on the date of such receipt shall be deemed to be the full value of the consideration received or accruing

as a result of the transfer of such capital asset.

Explanation.—For the purposes of this sub-section, the expression "insurer" shall have the meaning assigned to it in

clause (9) of section 2 of the Insurance Act, 1938 (4 of 1938).

(2) Notwithstanding anything contained in sub-section (1), the profits or gains arising from the transfer by way of

conversion by the owner of a capital asset into, or its treatment by him as stock-in-trade of a business carried on by

him shall be chargeable to income-tax as his income of the previous year in which such stock-in-trade is sold or

otherwise transferred by him and, for the purposes of section 48, the fair market value of the asset on the date of

such conversion or treatment shall be deemed to be the full value of the consideration received or accruing as a

result of the transfer of the capital asset.

(2A) Where any person has had at any time during previous year any beneficial interest in any securities, then, any

profits or gains arising from transfer made by the depository or participant of such beneficial interest in respect of

securities shall be chargeable to income-tax as the income of the beneficial owner of the previous year in which such

transfer took place and shall not be regarded as income of the depository who is deemed to be the registered owner

of securities by virtue of sub-section (1) of section 10 of the Depositories Act, 1996, and for the purposes of—

(i) section 48; and

(ii) proviso to clause (42A) of section 2,

the cost of acquisition and the period of holding of any securities shall be determined on the basis of the first-in-

first-out method.

Explanation.—For the purposes of this sub-section, the expressions "beneficial owner", "depository" and "security"

shall have the meanings respectively assigned to them in clauses (a), (e) and (l) of sub-section (1) of section 2 of the

Depositories Act, 1996.

(3) The profits or gains arising from the transfer of a capital asset by a person to a firm or other association of

persons or body of individuals (not being a company or a co-operative society) in which he is or becomes a partner

or member, by way of capital contribution or otherwise, shall be chargeable to tax as his income of the previous year

in which such transfer takes place and, for the purposes of section 48, the amount recorded in the books of account

of the firm, association or body as the value of the capital asset shall be deemed to be the full value of the

consideration received or accruing as a result of the transfer of the capital asset.

Page 88: Capital Gains [Income Tax]

Income Tax 10.84

(4) The profits or gains arising from the transfer of a capital asset by way of distribution of capital assets on the

dissolution of a firm or other association of persons or body of individuals (not being a company or a co-operative

society) or otherwise, shall be chargeable to tax as the income of the firm, association or body, of the previous year

in which the said transfer takes place and, for the purposes of section 48, the fair market value of the asset on the

date of such transfer shall be deemed to be the full value of the consideration received or accruing as a result of the

transfer.

(5) Notwithstanding anything contained in sub-section (1), where the capital gain arises from the transfer of a capital

asset, being a transfer by way of compulsory acquisition under any law, or a transfer the consideration for which was

determined or approved by the Central Government or the Reserve Bank of India, and the compensation or the

consideration for such transfer is enhanced or further enhanced by any court, Tribunal or other authority, the capital

gain shall be dealt with in the following manner, namely :—

(a) the capital gain computed with reference to the compensation awarded in the first instance or, as the case

may be, the consideration determined or approved in the first instance by the Central Government or the

Reserve Bank of India shall be chargeable as income under the head "Capital gains" of the previous year

in which such compensation or part thereof, or such consideration or part thereof, was first received; and

(b) the amount by which the compensation or consideration is enhanced or further enhanced by the court,

Tribunal or other authority shall be deemed to be income chargeable under the head "Capital gains" of the

previous year in which such amount is received by the assessee :

91[Provided that any amount of compensation received in pursuance of an interim order of a court,

Tribunal or other authority shall be deemed to be income chargeable under the head "Capital gains" of

the previous year in which the final order of such court, Tribunal or other authority is made;]

(c) where in the assessment for any year, the capital gain arising from the transfer of a capital asset is computed

by taking the compensation or consideration referred to in clause (a) or, as the case may be, enhanced

compensation or consideration referred to in clause (b), and subsequently such compensation or

consideration is reduced by any court, Tribunal or other authority, such assessed capital gain of that year

shall be recomputed by taking the compensation or consideration as so reduced by such court, Tribunal or

other authority to be the full value of the consideration.

Explanation.—For the purposes of this sub-section,—

(i) in relation to the amount referred to in clause (b), the cost of acquisition and the cost of improvement shall

be taken to be nil;

(ii) the provisions of this sub-section shall apply also in a case where the transfer took place prior to the 1st day

of April, 1988;

(iii) where by reason of the death of the person who made the transfer, or for any other reason, the enhanced

compensation or consideration is received by any other person, the amount referred to in clause (b) shall

be deemed to be the income, chargeable to tax under the head "Capital gains", of such other person.

(6) Notwithstanding anything contained in sub-section (1), the difference between the repurchase price of the units

referred to in sub-section (2) of section 80CCB and the capital value of such units shall be deemed to be the capital

gains arising to the assessee in the previous year in which such repurchase takes place or the plan referred to in that

section is terminated and shall be taxed accordingly.

Explanation.—For the purposes of this sub-section, "capital value of such units" means any amount invested by the

assessee in the units referred to in sub-section (2) of section 80CCB.

Capital gains on distribution of assets by companies in liquidation.

Page 89: Capital Gains [Income Tax]

Capital Gains 10.85

46. (1) Notwithstanding anything contained in section 45, where the assets of a company are distributed to its

shareholders on its liquidation, such distribution shall not be regarded as a transfer by the company for the purposes

of section 45.

(2) Where a shareholder on the liquidation of a company receives any money or other assets from the company, he

shall be chargeable to income-tax under the head "Capital gains", in respect of the money so received or the market

value of the other assets on the date of distribution, as reduced by the amount assessed as dividend within the

meaning of sub-clause (c) of clause (22) of section 2 and the sum so arrived at shall be deemed to be the full value

of the consideration for the purposes of section 48.

Capital gains on purchase by company of its own shares or other specified securities.

46A. Where a shareholder or a holder of other specified securities receives any consideration from any company for

purchase of its own shares or other specified securities held by such shareholder or holder of other specified

securities, then, subject to the provisions of section 48, the difference between the cost of acquisition and the value

of consideration received by the shareholder or the holder of other specified securities, as the case may be, shall be

deemed to be the capital gains arising to such shareholder or the holder of other specified securities, as the case may

be, in the year in which such shares or other specified securities were purchased by the company.

Explanation.—For the purposes of this section, "specified securities" shall have the meaning assigned to it

in Explanation to section 77A of the Companies Act, 1956 (1 of 1956).

Transactions not regarded as transfer.

47. Nothing contained in section 45 shall apply to the following transfers :—

(i) any distribution of capital assets on the total or partial partition of a Hindu undivided family;

(ii) [***]

(iii) any transfer of a capital asset under a gift or will or an irrevocable trust :

Provided that this clause shall not apply to transfer under a gift or an irrevocable trust of a capital asset

being shares, debentures or warrants allotted by a company directly or indirectly to its employees under

any Employees' Stock Option Plan or Scheme of the company offered to such employees in accordance

with the guidelines issued by the Central Government in this behalf;

(iv) any transfer of a capital asset by a company to its subsidiary company, if—

(a) the parent company or its nominees hold the whole of the share capital of the subsidiary company, and

(b) the subsidiary company is an Indian company;

(v) any transfer of a capital asset by a subsidiary company to the holding company, if—

(a) the whole of the share capital of the subsidiary company is held by the holding company, and

(b) the holding company is an Indian company :

Provided that nothing contained in clause (iv) or clause (v) shall apply to the transfer of a capital asset

made after the 29th day of February, 1988, as stock-in-trade;

(vi) any transfer, in a scheme of amalgamation, of a capital asset by the amalgamating company to the

amalgamated company if the amalgamated company is an Indian company;

(via) any transfer, in a scheme of amalgamation, of a capital asset being a share or shares held in an Indian

company, by the amalgamating foreign company to the amalgamated foreign company, if—

(a) at least twenty-five per cent of the shareholders of the amalgamating foreign company continue to

remain shareholders of the amalgamated foreign company, and

Page 90: Capital Gains [Income Tax]

Income Tax 10.86

(b) such transfer does not attract tax on capital gains in the country, in which the amalgamating company

is incorporated;

(viaa) any transfer, in a scheme of amalgamation of a banking company with a banking institution sanctioned and

brought into force by the Central Government under sub-section (7) of section 45 of the Banking

Regulation Act, 1949 (10 of 1949), of a capital asset by the banking company to the banking institution.

Explanation.—For the purposes of this clause,—

(i) "banking company" shall have the same meaning assigned to it in clause (c) of section 5 of the Banking

Regulation Act, 1949 (10 of 1949);

(ii) "banking institution" shall have the same meaning assigned to it in sub-section (15) of section 45 of the

Banking Regulation Act, 1949 (10 of 1949);]

Following clause (viab) shall be inserted after clause (viaa) of section 47 by the Finance Act, 2015,

w.e.f. 1-4-2016 :

(viab) any transfer, in a scheme of amalgamation, of a capital asset, being a share of a foreign company, referred

to in the Explanation 5 to clause (i) of sub-section (1) of section 9, which derives, directly or indirectly, its

value substantially from the share or shares of an Indian company, held by the amalgamating foreign

company to the amalgamated foreign company, if—

(A) at least twenty-five per cent of the shareholders of the amalgamating foreign company continue to

remain shareholders of the amalgamated foreign company; and

(B) such transfer does not attract tax on capital gains in the country in which the amalgamating company

is incorporated;

(vib) any transfer, in a demerger, of a capital asset by the demerged company to the resulting company, if the

resulting company is an Indian company;

(vic) any transfer in a demerger, of a capital asset, being a share or shares held in an Indian company, by the

demerged foreign company to the resulting foreign company, if—

(a) the shareholders holding not less than three-fourths in value of the shares of the demerged foreign

company continue to remain shareholders of the resulting foreign company; and

(b) such transfer does not attract tax on capital gains in the country, in which the demerged foreign

company is incorporated :

Provided that the provisions of sections 391 to 394 of the Companies Act, 1956 (1 of 1956) shall not

apply in case of demergers referred to in this clause;

(vica) any transfer in a business reorganisation, of a capital asset by the predecessor co-operative bank to the

successor co-operative bank;

(vicb) any transfer by a shareholder, in a business reorganisation, of a capital asset being a share or shares held by

him in the predecessor co-operative bank if the transfer is made in consideration of the allotment to him of

any share or shares in the successor co-operative bank.

Explanation.—For the purposes of clauses (vica) and (vicb), the expressions "business reorganisation",

"predecessor co-operative bank" and "successor co-operative bank" shall have the meanings respectively

assigned to them in section 44DB;

Following clause (vicc) shall be inserted after clause (vicb) of section 47 by the Finance Act, 2015,

w.e.f. 1-4-2016 :

(vicc) any transfer in a demerger, of a capital asset, being a share of a foreign company, referred to in

the Explanation 5 to clause (i) of sub-section (1) of section 9, which derives, directly or indirectly, its

value substantially from the share or shares of an Indian company, held by the demerged foreign company

to the resulting foreign company, if—

Page 91: Capital Gains [Income Tax]

Capital Gains 10.87

(a) the shareholders, holding not less than three-fourths in value of the shares of the demerged foreign

company, continue to remain shareholders of the resulting foreign company; and

(b) such transfer does not attract tax on capital gains in the country in which the demerged foreign

company is incorporated:

Provided that the provisions of sections 391 to 394 of the Companies Act, 1956 (1 of 1956) shall not

apply in case of demergers referred to in this clause;

(vid) any transfer or issue of shares by the resulting company, in a scheme of demerger to the shareholders of the

demerged company if the transfer or issue is made in consideration of demerger of the undertaking;

(vii) any transfer by a shareholder, in a scheme of amalgamation, of a capital asset being a share or shares held

by him in the amalgamating company, if—

(a) the transfer is made in consideration of the allotment to him of any share or shares in the amalgamated

company except where the shareholder itself is the amalgamated company, and

(b) the amalgamated company is an Indian company;

(viia) any transfer of a capital asset, being bonds or Global Depository Receipts referred to in sub-section (1)

of section 115AC, made outside India by a non-resident to another non-resident;

92[(viib) any transfer of a capital asset, being a Government Security carrying a periodic payment of interest, made

outside India through an intermediary dealing in settlement of securities, by a non-resident to another

non-resident.

Explanation.—For the purposes of this clause, "Government Security" shall have the meaning assigned to

it in clause (b) of section 2 of the Securities Contracts (Regulation) Act, 1956 (42 of 1956);]

(viii) any transfer of agricultural land in India effected before the 1st day of March, 1970;

(ix) any transfer of a capital asset, being any work of art, archaeological, scientific or art collection, book,

manuscript, drawing, painting, photograph or print, to the Government or a University or the National

Museum, National Art Gallery, National Archives or any such other public museum or institution as may

be notified by the Central Government in the Official Gazette to be of national importance or to be of

renown throughout any State or States.

Explanation.—For the purposes of this clause, "University" means a University established or

incorporated by or under a Central, State or Provincial Act and includes an institution declared under

section 3 of the University Grants Commission Act, 1956 (3 of 1956), to be a University for the purposes

of that Act;

(x) any transfer by way of conversion of bonds or debentures, debenture-stock or deposit certificates in any

form, of a company into shares or debentures of that company;

(xa) any transfer by way of conversion of bonds referred to in clause (a) of sub-section (1) of section

115AC into shares or debentures of any company;

(xi) any transfer made on or before the 31st day of December, 1998 by a person (not being a company) of a

capital asset being membership of a recognised stock exchange to a company in exchange of shares

allotted by that company to the transferor.

Explanation.—For the purposes of this clause, the expression "membership of a recognised stock

exchange" means the membership of a stock exchange in India which is recognised under the provisions

of the Securities Contracts (Regulation) Act, 1956 (42 of 1956);

(xii) any transfer of a capital asset, being land of a sick industrial company, made under a scheme prepared and

sanctioned under section 18 of the Sick Industrial Companies (Special Provisions) Act, 1985 (1 of 1986)

where such sick industrial company is being managed by its workers' co-operative :

Provided that such transfer is made during the period commencing from the previous year in which the

said company has become a sick industrial company under sub-section (1) of section 17 of that Act and

Page 92: Capital Gains [Income Tax]

Income Tax 10.88

ending with the previous year during which the entire net worth of such company becomes equal to or

exceeds the accumulated losses.

Explanation.—For the purposes of this clause, "net worth" shall have the meaning assigned to it in clause

(ga) of sub-section (1) of section 3 of the Sick Industrial Companies (Special Provisions) Act, 1985 (1 of

1986);

(xiii) any transfer of a capital asset or intangible asset by a firm to a company as a result of succession of the

firm by a company in the business carried on by the firm, or any transfer of a capital asset to a company in

the course of demutualisation or corporatisation of a recognised stock exchange in India as a result of

which an association of persons or body of individuals is succeeded by such company :

Provided that—

(a) all the assets and liabilities of the firm or of the association of persons or body of individuals relating

to the business immediately before the succession become the assets and liabilities of the company;

(b) all the partners of the firm immediately before the succession become the shareholders of the company

in the same proportion in which their capital accounts stood in the books of the firm on the date of

the succession;

(c) the partners of the firm do not receive any consideration or benefit, directly or indirectly, in any form

or manner, other than by way of allotment of shares in the company; and

(d) the aggregate of the shareholding in the company of the partners of the firm is not less than fifty per

cent of the total voting power in the company and their shareholding continues to be as such for a

period of five years from the date of the succession;

(e) the demutualisation or corporatisation of a recognised stock exchange in India is carried out in

accordance with a scheme for demutualisation or corporatisation which is approved by the Securities

and Exchange Board of India established under section 3 of the Securities and Exchange Board of

India Act, 1992 (15 of 1992);

(xiiia) any transfer of a capital asset being a membership right held by a member of a recognised stock exchange

in India for acquisition of shares and trading or clearing rights acquired by such member in that recognised

stock exchange in accordance with a scheme for demutualisation or corporatisation which is approved by

the Securities and Exchange Board of India established under section 3 of the Securities and Exchange

Board of India Act, 1992 (15 of 1992);

(xiiib) any transfer of a capital asset or intangible asset by a private company or unlisted public company

(hereafter in this clause referred to as the company) to a limited liability partnership or any transfer of a

share or shares held in the company by a shareholder as a result of conversion of the company into a

limited liability partnership in accordance with the provisions of section 56 or section 57 of the Limited

Liability Partnership Act, 2008 (6 of 2009):

Provided that—

(a) all the assets and liabilities of the company immediately before the conversion become the assets and

liabilities of the limited liability partnership;

(b) all the shareholders of the company immediately before the conversion become the partners of the

limited liability partnership and their capital contribution and profit sharing ratio in the limited

liability partnership are in the same proportion as their shareholding in the company on the date of

conversion;

(c) the shareholders of the company do not receive any consideration or benefit, directly or indirectly, in

any form or manner, other than by way of share in profit and capital contribution in the limited

liability partnership;

Page 93: Capital Gains [Income Tax]

Capital Gains 10.89

(d) the aggregate of the profit sharing ratio of the shareholders of the company in the limited liability

partnership shall not be less than fifty per cent at any time during the period of five years from the

date of conversion;

(e) the total sales, turnover or gross receipts in the business of the company in any of the three previous

years preceding the previous year in which the conversion takes place does not exceed sixty lakh

rupees; and

(f) no amount is paid, either directly or indirectly, to any partner out of balance of accumulated profit

standing in the accounts of the company on the date of conversion for a period of three years from

the date of conversion.

Explanation.—For the purposes of this clause, the expressions "private company" and "unlisted public

company" shall have the meanings respectively assigned to them in the Limited Liability Partnership Act,

2008 (6 of 2009);

(xiv) where a sole proprietary concern is succeeded by a company in the business carried on by it as a result of

which the sole proprietary concern sells or otherwise transfers any capital asset or intangible asset to the

company :

Provided that—

(a) all the assets and liabilities of the sole proprietary concern relating to the business immediately before

the succession become the assets and liabilities of the company;

(b) the shareholding of the sole proprietor in the company is not less than fifty per cent of the total voting

power in the company and his shareholding continues to remain as such for a period of five years

from the date of the succession; and

(c) the sole proprietor does not receive any consideration or benefit, directly or indirectly, in any form or

manner, other than by way of allotment of shares in the company;

(xv) any transfer in a scheme for lending of any securities under an agreement or arrangement, which the

assessee has entered into with the borrower of such securities and which is subject to the guidelines issued

by the Securities and Exchange Board of India, established under section 3 of the Securities and Exchange

Board of India Act, 1992 (15 of 1992) or the Reserve Bank of India constituted under sub-section (1) of

section 3 of the Reserve Bank of India Act, 1934 (2 of 1934), in this regard;

(xvi) any transfer of a capital asset in a transaction of reverse mortgage under a scheme made and notified by the

Central Government;

93[(xvii) any transfer of a capital asset, being share of a special purpose vehicle to a business trust in exchange of

units allotted by that trust to the transferor.

Explanation.—For the purposes of this clause, the expression "special purpose vehicle" shall have the

meaning assigned to it in theExplanation to clause (23FC) of section 10.]

Following clause (xviii) shall be inserted after clause (xvii) of section 47 by the Finance Act, 2015,

w.e.f. 1-4-2016 :

(xviii) any transfer by a unit holder of a capital asset, being a unit or units, held by him in the consolidating

scheme of a mutual fund, made in consideration of the allotment to him of a capital asset, being a unit or

units, in the consolidated scheme of the mutual fund:

Provided that the consolidation is of two or more schemes of equity oriented fund or of two or more

schemes of a fund other than equity oriented fund.

Explanation.— For the purposes of this clause,—

(a) "consolidated scheme" means the scheme with which the consolidating scheme merges or which is

formed as a result of such merger;

Page 94: Capital Gains [Income Tax]

Income Tax 10.90

(b) "consolidating scheme" means the scheme of a mutual fund which merges under the process of

consolidation of the schemes of mutual fund in accordance with the Securities and Exchange Board

of India (Mutual Funds) Regulations, 1996 made under the Securities and Exchange Board of India

Act, 1992 (15 of 1992);

(c) "equity oriented fund" shall have the meaning assigned to it in clause (38)of section 10;

(d) "mutual fund" means a mutual fund specified under clause (23D)of section 10.

Withdrawal of exemption in certain cases.

47A. (1) Where at any time before the expiry of a period of eight years from the date of the transfer of a capital asset

referred to in clause (iv) or, as the case may be, clause (v) of section 47,—

(i) such capital asset is converted by the transferee company into, or is treated by it as, stock-in-trade of its

business; or

(ii) the parent company or its nominees or, as the case may be, the holding company ceases or cease to hold the

whole of the share capital of the subsidiary company,

the amount of profits or gains arising from the transfer of such capital asset not charged under section 45 by virtue of

the provisions contained in clause (iv) or, as the case may be, clause (v) of section 47 shall, notwithstanding

anything contained in the said clauses, be deemed to be income chargeable under the head "Capital gains" of the

previous year in which such transfer took place.

(2) Where at any time, before the expiry of a period of three years from the date of the transfer of a capital asset

referred to in clause (xi) ofsection 47, any of the shares allotted to the transferor in exchange of a membership in a

recognised stock exchange are transferred, the amount of profits and gains not charged under section 45 by virtue of

the provisions contained in clause (xi) of section 47 shall, notwithstanding anything contained in the said clause, be

deemed to be the income chargeable under the head "Capital gains" of the previous year in which such shares are

transferred.

(3) Where any of the conditions laid down in the proviso to clause (xiii) or the proviso to clause (xiv) of section

47 are not complied with, the amount of profits or gains arising from the transfer of such capital asset or intangible

asset not charged under section 45 by virtue of conditions laid down in the proviso to clause (xiii) or the proviso to

clause (xiv) of section 47 shall be deemed to be the profits and gains chargeable to tax of the successor company for

the previous year in which the requirements of the proviso to clause (xiii) or the proviso to clause (xiv), as the case

may be, are not complied with.

(4) Where any of the conditions laid down in the proviso to clause (xiiib) of section 47 are not complied with, the

amount of profits or gains arising from the transfer of such capital asset or intangible assets or share or shares not

charged under section 45 by virtue of conditions laid down in the said proviso shall be deemed to be the profits and

gains chargeable to tax of the successor limited liability partnership or the shareholder of the predecessor company,

as the case may be, for the previous year in which the requirements of the said proviso are not complied with.

Mode of computation.

48. The income chargeable under the head "Capital gains" shall be computed, by deducting from the full value of the

consideration received or accruing as a result of the transfer of the capital asset the following amounts, namely :—

(i) expenditure incurred wholly and exclusively in connection with such transfer;

(ii) the cost of acquisition of the asset and the cost of any improvement thereto:

94Provided that in the case of an assessee, who is a non-resident, capital gains arising from the transfer of a capital

asset being shares in, or debentures of, an Indian company shall be computed by converting the cost of acquisition,

expenditure incurred wholly and exclusively in connection with such transfer and the full value of the consideration

received or accruing as a result of the transfer of the capital asset into the same foreign currency as was initially

Page 95: Capital Gains [Income Tax]

Capital Gains 10.91

utilised in the purchase of the shares or debentures, and the capital gains so computed in such foreign currency shall

be reconverted into Indian currency, so, however, that the aforesaid manner of computation of capital gains shall be

applicable in respect of capital gains accruing or arising from every reinvestment thereafter in, and sale of, shares in,

or debentures of, an Indian company :

Provided further that where long-term capital gain arises from the transfer of a long-term capital asset, other than

capital gain arising to a non-resident from the transfer of shares in, or debentures of, an Indian company referred to

in the first proviso, the provisions of clause (ii) shall have effect as if for the words "cost of acquisition" and "cost of

any improvement", the words "indexed cost of acquisition" and "indexed cost of any improvement" had respectively

been substituted:

Provided also that nothing contained in the second proviso shall apply to the long-term capital gain arising from the

transfer of a long-term capital asset being bond or debenture other than capital indexed bonds issued by the

Government :

Provided also that where shares, debentures or warrants referred to in the proviso to clause (iii) of section 47 are

transferred under a gift or an irrevocable trust, the market value on the date of such transfer shall be deemed to be

the full value of consideration received or accruing as a result of transfer for the purposes of this section :

Provided also that no deduction shall be allowed in computing the income chargeable under the head "Capital

gains" in respect of any sum paid on account of securities transaction tax under Chapter VII of the Finance (No. 2)

Act, 2004.

Explanation.—For the purposes of this section,—

(i) "foreign currency" and "Indian currency" shall have the meanings respectively assigned to them in section 2

of 95

[the Foreign Exchange Management Act, 1999 (42 of 1999)];

(ii) the conversion of Indian currency into foreign currency and the reconversion of foreign currency into

Indian currency shall be at the rate of exchange prescribed in this behalf;

(iii) "indexed cost of acquisition" means an amount which bears to the cost of acquisition the same proportion

as Cost Inflation Index for the year in which the asset is transferred bears to the Cost Inflation Index for

the first year in which the asset was held by the assessee or for the year beginning on the 1st day of April,

1981, whichever is later;

(iv) "indexed cost of any improvement" means an amount which bears to the cost of improvement the same

proportion as Cost Inflation Index for the year in which the asset is transferred bears to the Cost Inflation

Index for the year in which the improvement to the asset took place;

(v) "Cost Inflation Index", in relation to a previous year, means such Index as the Central Government may,

having regard to seventy-five per cent of average rise in the 96

[Consumer Price Index for urban non-

manual employees] for the immediately preceding previous year to such previous year, by notification97

in

the Official Gazette, specify, in this behalf.

Cost with reference to certain modes of acquisition.

49. (1) Where the capital asset became the property of the assessee—

(i) on any distribution of assets on the total or partial partition of a Hindu undivided family;

(ii) under a gift or will;

(iii) (a) by succession, inheritance or devolution, or

(b) on any distribution of assets on the dissolution of a firm, body of individuals, or other association of

persons, where such dissolution had taken place at any time before the 1st day of April, 1987, or

(c) on any distribution of assets on the liquidation of a company, or

(d) under a transfer to a revocable or an irrevocable trust, or

Page 96: Capital Gains [Income Tax]

Income Tax 10.92

(e) under any such transfer as is referred to in clause (iv) or clause (v) or clause (vi) or clause (via) 98

[or

clause (viaa) or clause (vica) or clause (vicb)] or clause (xiii) or clause (xiiib) or clause (xiv)

of section 47;

(iv) such assessee being a Hindu undivided family, by the mode referred to in sub-section (2) of section 64 at

any time after the 31st day of December, 1969,

the cost of acquisition of the asset shall be deemed to be the cost for which the previous owner of the property

acquired it, as increased by the cost of any improvement of the assets incurred or borne by the previous owner or the

assessee, as the case may be.

Explanation.—In this sub-section the expression "previous owner of the property" in relation to any capital asset

owned by an assessee means the last previous owner of the capital asset who acquired it by a mode of acquisition

other than that referred to in clause (i) or clause (ii) or clause (iii) or clause (iv) of this sub-section.

(2) Where the capital asset being a share or shares in an amalgamated company which is an Indian company became

the property of the assessee in consideration of a transfer referred to in clause (vii) of section 47, the cost of

acquisition of the asset shall be deemed to be the cost of acquisition to him of the share or shares in the

amalgamating company.

(2A) Where the capital asset, being a share or debenture of a company, became the property of the assessee in

consideration of a transfer referred to in clause (x) or clause (xa) of section 47, the cost of acquisition of the asset to

the assessee shall be deemed to be that part of the cost of debenture, debenture-stock, bond or deposit certificate in

relation to which such asset is acquired by the assessee.

(2AA) Where the capital gain arises from the transfer of specified security or sweat equity shares referred to in sub-

clause (vi) of clause (2) ofsection 17, the cost of acquisition of such security or shares shall be the fair market value

which has been taken into account for the purposes of the said sub-clause.

(2AAA) Where the capital asset, being rights of a partner referred to in section 42 of the Limited Liability

Partnership Act, 2008 (6 of 2009), became the property of the assessee on conversion as referred to in clause (xiiib)

of section 47, the cost of acquisition of the asset shall be deemed to be the cost of acquisition to him of the share or

shares in the company immediately before its conversion.

(2AB) Where the capital gain arises from the transfer of specified security or sweat equity shares, the cost of

acquisition of such security or shares shall be the fair market value which has been taken into account while

computing the value of fringe benefits under clause (ba) of sub-section (1) of section 115WC.

(2ABB) Where the capital asset, being share or shares of a company, is acquired by a non-resident assessee on

redemption of Global Depository Receipts referred to in clause (b) of sub-section (1) of section 115AC held by such

assessee, the cost of acquisition of the share or shares shall be the price of such share or shares prevailing on any

recognised stock exchange on the date on which a request for such redemption was made.

Explanation.—For the purposes of this sub-section, "recognised stock exchange" shall have the meaning assigned to

it in clause (ii) of theExplanation 1 to sub-section (5) of section 43.

99[(2AC) Where the capital asset, being a unit of a business trust, became the property of the assessee in

consideration of a transfer as referred to in clause (xvii) of section 47, the cost of acquisition of the asset shall be

deemed to be the cost of acquisition to him of the share referred to in the said clause.]

Following sub-section (2AD) shall be inserted after sub-section (2AC) of section 49 by the Finance Act, 2015,

w.e.f. 1-4-2016 :

(2AD)Where the capital asset, being a unit or units in a consolidated scheme of a mutual fund, became the property

of the assessee in consideration of a transfer referred to in clause (xviii)of section 47, the cost of acquisition of the

asset shall be deemed to be the cost of acquisition to him of the unit or units in the consolidating scheme of the

mutual fund.

(2B) [***]

Page 97: Capital Gains [Income Tax]

Capital Gains 10.93

(2C) The cost of acquisition of the shares in the resulting company shall be the amount which bears to the cost of

acquisition of shares held by the assessee in the demerged company the same proportion as the net book value of the

assets transferred in a demerger bears to the net worth of the demerged company immediately before such demerger.

(2D) The cost of acquisition of the original shares held by the shareholder in the demerged company shall be

deemed to have been reduced by the amount as so arrived at under sub-section (2C).

(2E) The provisions of sub-section (2), sub-section (2C) and sub-section (2D) shall, as far as may be, also apply in

relation to business reorganisation of a co-operative bank as referred to in section 44DB.

Explanation.—For the purposes of this section, "net worth" shall mean the aggregate of the paid up share capital and

general reserves as appearing in the books of account of the demerged company immediately before the demerger.

(3) Notwithstanding anything contained in sub-section (1), where the capital gain arising from the transfer of a

capital asset referred to in clause (iv) or, as the case may be, clause (v) of section 47 is deemed to be income

chargeable under the head "Capital gains" by virtue of the provisions contained in section 47A, the cost of

acquisition of such asset to the transferee-company shall be the cost for which such asset was acquired by it.

(4) Where the capital gain arises from the transfer of a property, the value of which has been subject to income-tax

under clause (vii) or clause (viia)of sub-section (2) of section 56, the cost of acquisition of such property shall be

deemed to be the value which has been taken into account for the purposes of the said clause (vii) or clause (viia).

Special provision for computation of capital gains in case of depreciable assets.

50. Notwithstanding anything contained in clause (42A) of section 2, where the capital asset is an asset forming part

of a block of assets in respect of which depreciation has been allowed under this Act or under the Indian Income-tax

Act, 1922 (11 of 1922), the provisions of sections 48 and 49 shall be subject to the following modifications :—

(1) where the full value of the consideration received or accruing as a result of the transfer of the asset together

with the full value of such consideration received or accruing as a result of the transfer of any other capital

asset falling within the block of the assets during the previous year, exceeds the aggregate of the following

amounts, namely :—

(i) expenditure incurred wholly and exclusively in connection with such transfer or transfers;

(ii) the written down value of the block of assets at the beginning of the previous year; and

(iii) the actual cost of any asset falling within the block of assets acquired during the previous year,

such excess shall be deemed to be the capital gains arising from the transfer of short-term capital assets;

(2) where any block of assets ceases to exist as such, for the reason that all the assets in that block are

transferred during the previous year, the cost of acquisition of the block of assets shall be the written down

value of the block of assets at the beginning of the previous year, as increased by the actual cost of any

asset falling within that block of assets, acquired by the assessee during the previous year and the income

received or accruing as a result of such transfer or transfers shall be deemed to be the capital gains arising

from the transfer of short-term capital assets.

Special provision for cost of acquisition in case of depreciable asset.

50A. Where the capital asset is an asset in respect of which a deduction on account of depreciation under clause (i)

of sub-section (1) of section 32has been obtained by the assessee in any previous year, the provisions of sections

48 and 49 shall apply subject to the modification that the written down value, as defined in clause (6) of section 43,

of the asset, as adjusted, shall be taken as the cost of acquisition of the asset.

Special provision for computation of capital gains in case of slump sale.

Page 98: Capital Gains [Income Tax]

Income Tax 10.94

50B. (1) Any profits or gains arising from the slump sale effected in the previous year shall be chargeable to

income-tax as capital gains arising from the transfer of long-term capital assets and shall be deemed to be the

income of the previous year in which the transfer took place :

Provided that any profits or gains arising from the transfer under the slump sale of any capital asset being one or

more undertakings owned and held by an assessee for not more than thirty-six months immediately preceding the

date of its transfer shall be deemed to be the capital gains arising from the transfer of short-term capital assets.

(2) In relation to capital assets being an undertaking or division transferred by way of such sale, the "net worth" of

the undertaking or the division, as the case may be, shall be deemed to be the cost of acquisition and the cost of

improvement for the purposes of sections 48 and 49and no regard shall be given to the provisions contained in the

second proviso to section 48.

(3) Every assessee, in the case of slump sale, shall furnish in the prescribed form1 along with the return of income, a

report of an accountant as defined in the Explanation below sub-section (2) of section 288, indicating the

computation of the net worth of the undertaking or division, as the case may be, and certifying that the net worth of

the undertaking or division, as the case may be, has been correctly arrived at in accordance with the provisions of

this section.

Explanation 1.—For the purposes of this section, "net worth" shall be the aggregate value of total assets of the

undertaking or division as reduced by the value of liabilities of such undertaking or division as appearing in its

books of account :

Provided that any change in the value of assets on account of revaluation of assets shall be ignored for the purposes

of computing the net worth.

Explanation 2.—For computing the net worth, the aggregate value of total assets shall be,—

(a) in the case of depreciable assets, the written down value of the block of assets determined in accordance

with the provisions contained in sub-item (C) of item (i) of sub-clause (c) of clause (6) of section 43;

(b) in the case of capital assets in respect of which the whole of the expenditure has been allowed or is

allowable as a deduction undersection 35AD, nil; and

(c) in the case of other assets, the book value of such assets.

Special provision for full value of consideration in certain cases.

50C. (1) Where the consideration received or accruing as a result of the transfer by an assessee of a capital asset,

being land or building or both, is less than the value adopted or assessed or assessable by any authority of a State

Government (hereafter in this section referred to as the "stamp valuation authority") for the purpose of payment of

stamp duty in respect of such transfer, the value so adopted or assessed or assessable shall, for the purposes

of section 48, be deemed to be the full value of the consideration received or accruing as a result of such transfer.

(2) Without prejudice to the provisions of sub-section (1), where—

(a) the assessee claims before any Assessing Officer that the value adopted or assessed or assessable by the

stamp valuation authority under sub-section (1) exceeds the fair market value of the property as on the

date of transfer;

(b) the value so adopted or assessed or assessable by the stamp valuation authority under sub-section (1) has

not been disputed in any appeal or revision or no reference has been made before any other authority,

court or the High Court,

the Assessing Officer may refer the valuation of the capital asset to a Valuation Officer and where any such

reference is made, the provisions of sub-sections (2), (3), (4), (5) and (6) of section 16A, clause (i) of sub-section (1)

and sub-sections (6) and (7) of section 23A, sub-section (5) of section 24, section 34AA, section 35 and section 37

of the Wealth-tax Act, 1957 (27 of 1957), shall, with necessary modifications, apply in relation to such reference as

they apply in relation to a reference made by the Assessing Officer under sub-section (1) of section 16A of that Act.

Page 99: Capital Gains [Income Tax]

Capital Gains 10.95

Explanation 1.—For the purposes of this section, "Valuation Officer" shall have the same meaning as in clause (r) of

section 2 of the Wealth-tax Act, 1957 (27 of 1957).

Explanation 2.—For the purposes of this section, the expression "assessable" means the price which the stamp

valuation authority would have, notwithstanding anything to the contrary contained in any other law for the time

being in force, adopted or assessed, if it were referred to such authority for the purposes of the payment of stamp

duty.

(3) Subject to the provisions contained in sub-section (2), where the value ascertained under sub-section (2) exceeds

the value adopted or assessed or assessable by the stamp valuation authority referred to in sub-section (1), the value

so adopted or assessed or assessable by such authority shall be taken as the full value of the consideration received

or accruing as a result of the transfer.

Fair market value deemed to be full value of consideration in certain cases.

50D. Where the consideration received or accruing as a result of the transfer of a capital asset by an assessee is not

ascertainable or cannot be determined, then, for the purpose of computing income chargeable to tax as capital gains,

the fair market value of the said asset on the date of transfer shall be deemed to be the full value of the consideration

received or accruing as a result of such transfer.

Advance money received.

51. Where any capital asset was on any previous occasion the subject of negotiations for its transfer, any advance or

other money received and retained by the assessee in respect of such negotiations shall be deducted from the cost for

which the asset was acquired or the written down value or the fair market value, as the case may be, in computing

the cost of acquisition :

2[Provided that where any sum of money, received as an advance or otherwise in the course of negotiations for

transfer of a capital asset, has been included in the total income of the assessee for any previous year in accordance

with the provisions of clause (ix) of sub-section (2) of section 56, then, such sum shall not be deducted from the cost

for which the asset was acquired or the written down value or the fair market value, as the case may be, in

computing the cost of acquisition.]

Profit on sale of property used for residence.

54. (1) Subject to the provisions of sub-section (2), where, in the case of an assessee being an individual or a Hindu

undivided family, the capital gain arises from the transfer of a long-term capital asset, being buildings or lands

appurtenant thereto, and being a residential house, the income of which is chargeable under the head "Income from

house property" (hereafter in this section referred to as the original asset), and the assessee has within a period of

one year before or two years after the date on which the transfer took place purchased, or has within a period of

three years after that date 3[constructed, one residential house in India], then, instead of the capital gain being

charged to income-tax as income of the previous year in which the transfer took place, it shall be dealt with in

accordance with the following provisions of this section, that is to say,—

(i) if the amount of the capital gain is greater than the cost of the residential house so purchased or constructed

(hereafter in this section referred to as the new asset), the difference between the amount of the capital

gain and the cost of the new asset shall be charged undersection 45 as the income of the previous year; and

for the purpose of computing in respect of the new asset any capital gain arising from its transfer within a

period of three years of its purchase or construction, as the case may be, the cost shall be nil; or

(ii) if the amount of the capital gain is equal to or less than the cost of the new asset, the capital gain shall not

be charged under section 45; and for the purpose of computing in respect of the new asset any capital gain

Page 100: Capital Gains [Income Tax]

Income Tax 10.96

arising from its transfer within a period of three years of its purchase or construction, as the case may be,

the cost shall be reduced by the amount of the capital gain.

(2) The amount of the capital gain which is not appropriated by the assessee towards the purchase of the new asset

made within one year before the date on which the transfer of the original asset took place, or which is not utilised

by him for the purchase or construction of the new asset before the date of furnishing the return of income

under section 139, shall be deposited by him before furnishing such return [such deposit being made in any case not

later than the due date applicable in the case of the assessee for furnishing the return of income under sub-section (1)

ofsection 139] in an account in any such bank or institution as may be specified in, and utilised in accordance with,

any scheme which the Central Government may, by notification in the Official Gazette, frame in this behalf and

such return shall be accompanied by proof of such deposit; and, for the purposes of sub-section (1), the amount, if

any, already utilised by the assessee for the purchase or construction of the new asset together with the amount so

deposited shall be deemed to be the cost of the new asset :

Provided that if the amount deposited under this sub-section is not utilised wholly or partly for the purchase or

construction of the new asset within the period specified in sub-section (1), then,—

(i) the amount not so utilised shall be charged under section 45 as the income of the previous year in which the

period of three years from the date of the transfer of the original asset expires; and

(ii) the assessee shall be entitled to withdraw such amount in accordance with the scheme aforesaid.

Explanation.—[Omitted by the Finance Act, 1992, w.e.f. 1-4-1993.]

Capital gain on transfer of land used for agricultural purposes not to be charged in certain cases.

54B. (1) Subject to the provisions of sub-section (2), where the capital gain arises from the transfer of a capital asset

being land which, in the two years immediately preceding the date on which the transfer took place, was being used

by the assessee being an individual or his parent, or a Hindu undivided family for agricultural purposes (hereinafter

referred to as the original asset), and the assessee has, within a period of two years after that date, purchased any

other land for being used for agricultural purposes, then, instead of the capital gain being charged to income-tax as

income of the previous year in which the transfer took place, it shall be dealt with in accordance with the following

provisions of this section, that is to say,—

(i) if the amount of the capital gain is greater than the cost of the land so purchased (hereinafter referred to as

the new asset), the difference between the amount of the capital gain and the cost of the new asset shall be

charged under section 45 as the income of the previous year; and for the purpose of computing in respect

of the new asset any capital gain arising from its transfer within a period of three years of its purchase, the

cost shall be nil; or

(ii) if the amount of the capital gain is equal to or less than the cost of the new asset, the capital gain shall not

be charged under section 45; and for the purpose of computing in respect of the new asset any capital gain

arising from its transfer within a period of three years of its purchase, the cost shall be reduced, by the

amount of the capital gain.

(2) The amount of the capital gain which is not utilised by the assessee for the purchase of the new asset before the

date of furnishing the return of income under section 139, shall be deposited by him before furnishing such return

[such deposit being made in any case not later than the due date applicable in the case of the assessee for furnishing

the return of income under sub-section (1) of section 139] in an account in any such bank or institution as may be

specified in, and utilised in accordance with, any scheme which the Central Government may, by notification in the

Official Gazette, frame in this behalf and such return shall be accompanied by proof of such deposit; and, for the

purposes of sub-section (1), the amount, if any, already utilised by the assessee for the purchase of the new asset

together with the amount so deposited shall be deemed to be the cost of the new asset :

Provided that if the amount deposited under this sub-section is not utilised wholly or partly for the purchase of the

new asset within the period specified in sub-section (1), then,—

Page 101: Capital Gains [Income Tax]

Capital Gains 10.97

(i) the amount not so utilised shall be charged under section 45 as the income of the previous year in which the

period of two years from the date of the transfer of the original asset expires; and

(ii) the assessee shall be entitled to withdraw such amount in accordance with the scheme aforesaid.

Explanation.—[Omitted by the Finance Act, 1992, w.e.f. 1-4-1993.]

Capital gain on compulsory acquisition of lands and buildings not to be charged in certain cases.

54D. (1) Subject to the provisions of sub-section (2), where the capital gain arises from the transfer by way of

compulsory acquisition under any law of a capital asset, being land or building or any right in land or building,

forming part of an industrial undertaking belonging to the assessee which, in the two years immediately preceding

the date on which the transfer took place, was being used by the assessee for the purposes of the business of the said

undertaking (hereafter in this section referred to as the original asset), and the assessee has within a period of three

years after that date purchased any other land or building or any right in any other land or building or constructed

any other building for the purposes of shifting or re-establishing the said undertaking or setting up another industrial

undertaking, then, instead of the capital gain being charged to income-tax as the income of the previous year in

which the transfer took place, it shall be dealt with in accordance with the following provisions of this section, that

is to say,—

(i) if the amount of the capital gain is greater than the cost of the land, building or right so purchased or the

building so constructed (such land, building or right being hereafter in this section referred to as the new

asset), the difference between the amount of the capital gain and the cost of the new asset shall be charged

under section 45 as the income of the previous year; and for the purpose of computing in respect of the

new asset any capital gain arising from its transfer within a period of three years of its purchase or

construction, as the case may be, the cost shall be nil; or

(ii) if the amount of the capital gain is equal to or less than the cost of the new asset, the capital gain shall not

be charged under section 45; and for the purpose of computing in respect of the new asset any capital gain

arising from its transfer within a period of three years of its purchase or construction, as the case may be,

the cost shall be reduced by the amount of the capital gain.

(2) The amount of the capital gain which is not utilised by the assessee for the purchase or construction of the new

asset before the date of furnishing the return of income under section 139, shall be deposited by him before

furnishing such return [such deposit being made in any case not later than the due date applicable in the case of the

assessee for furnishing the return of income under sub-section (1) of section 139] in an account in any such bank or

institution as may be specified in, and utilised in accordance with, any scheme which the Central Government may,

by notification in the Official Gazette, frame in this behalf and such return shall be accompanied by proof of such

deposit; and, for the purposes of sub-section (1), the amount, if any, already utilised by the assessee for the purchase

or construction of the new asset together with the amount so deposited shall be deemed to be the cost of the new

asset:

Provided that if the amount deposited under this sub-section is not utilised wholly or partly for the purchase or

construction of the new asset within the period specified in sub-section (1), then,—

(i) the amount not so utilised shall be charged under section 45 as the income of the previous year in which the

period of three years from the date of the transfer of the original asset expires; and

(ii) the assessee shall be entitled to withdraw such amount in accordance with the scheme aforesaid.

Explanation.—[Omitted by the Finance Act, 1992, w.e.f. 1-4-1993.]

Capital gain not to be charged on investment in certain bonds.

54EC. (1) Where the capital gain arises from the transfer of a long-term capital asset (the capital asset so transferred

being hereafter in this section referred to as the original asset) and the assessee has, at any time within a period of six

Page 102: Capital Gains [Income Tax]

Income Tax 10.98

months after the date of such transfer, invested the whole or any part of capital gains in the long-term specified

asset, the capital gain shall be dealt with in accordance with the following provisions of this section, that is to say,—

(a) if the cost of the long-term specified asset is not less than the capital gain arising from the transfer of the

original asset, the whole of such capital gain shall not be charged under section 45;

(b) if the cost of the long-term specified asset is less than the capital gain arising from the transfer of the original

asset, so much of the capital gain as bears to the whole of the capital gain the same proportion as the cost of

acquisition of the long-term specified asset bears to the whole of the capital gain, shall not be charged

under section 45 :

Provided that the investment made on or after the 1st day of April, 2007 in the long-term specified asset by an

assessee during any financial year does not exceed fifty lakh rupees :

4[Provided further that the investment made by an assessee in the long-term specified asset, from capital gains

arising from transfer of one or more original assets, during the financial year in which the original asset or assets

are transferred and in the subsequent financial year does not exceed fifty lakh rupees.]

(2) Where the long-term specified asset is transferred or converted (otherwise than by transfer) into money at any

time within a period of three years from the date of its acquisition, the amount of capital gains arising from the

transfer of the original asset not charged under section 45 on the basis of the cost of such long-term specified asset

as provided in clause (a) or, as the case may be, clause (b) of sub-section (1) shall be deemed to be the income

chargeable under the head "Capital gains" relating to long-term capital asset of the previous year in which the long-

term specified asset is transferred or converted (otherwise than by transfer) into money.

Explanation.—In a case where the original asset is transferred and the assessee invests the whole or any part of the

capital gain received or accrued as a result of transfer of the original asset in any long-term specified asset and such

assessee takes any loan or advance on the security of such specified asset, he shall be deemed to have converted

(otherwise than by transfer) such specified asset into money on the date on which such loan or advance is taken.

(3) Where the cost of the long-term specified asset has been taken into account for the purposes of clause (a) or

clause (b) of sub-section (1),—

(a) a deduction from the amount of income-tax with reference to such cost shall not be allowed under section

88 for any assessment year ending before the 1st day of April, 2006;

(b) a deduction from the income with reference to such cost shall not be allowed under section 80C for any

assessment year beginning on or after the 1st day of April, 2006.

Explanation.—For the purposes of this section,—

(a) "cost", in relation to any long-term specified asset, means the amount invested in such specified asset out of

capital gains received or accruing as a result of the transfer of the original asset;

(b) "long-term specified asset" for making any investment under this section during the period commencing from

the 1st day of April, 2006 and ending with the 31st day of March, 2007, means any bond, redeemable after

three years and issued on or after the 1st day of April, 2006, but on or before the 31st day of March, 2007,—

(i) by the National Highways Authority of India constituted under section 3 of the National Highways

Authority of India Act, 1988 (68 of 1988); or

(ii) by the Rural Electrification Corporation Limited, a company formed and registered under the

Companies Act, 1956 (1 of 1956),

and notified by the Central Government in the Official Gazette for the purposes of this section with such

conditions (including the condition for providing a limit on the amount of investment by an assessee in such

bond) as it thinks fit:

Provided that where any bond has been notified before the 1st day of April, 2007, subject to the conditions

specified in the notification, by the Central Government in the Official Gazette under the provisions of clause

Page 103: Capital Gains [Income Tax]

Capital Gains 10.99

(b) as they stood immediately before their amendment by the Finance Act, 2007, such bond shall be deemed

to be a bond notified under this clause;

(ba) "long-term specified asset" for making any investment under this section on or after the 1st day of April, 2007

means any bond, redeemable after three years and issued on or after the 1st day of April, 2007 by the

National Highways Authority of India constituted under section 3 of the National Highways Authority of

India Act, 1988 (68 of 1988) or by the Rural Electrification Corporation Limited, a company formed and

registered under the Companies Act, 1956 (1 of 1956).

Capital gain on transfer of certain capital assets not to be charged in case of investment in residential house.

54F. (1) Subject to the provisions of sub-section (4), where, in the case of an assessee being an individual or a Hindu

undivided family, the capital gain arises from the transfer of any long-term capital asset, not being a residential

house (hereafter in this section referred to as the original asset), and the assessee has, within a period of one year

before or two years after the date on which the transfer took place purchased, or has within a period of three years

after that date 5[constructed, one residential house in India] (hereafter in this section referred to as the new asset),

the capital gain shall be dealt with in accordance with the following provisions of this section, that is to say,—

(a) if the cost of the new asset is not less than the net consideration in respect of the original asset, the whole of

such capital gain shall not be charged under section 45 ;

(b) if the cost of the new asset is less than the net consideration in respect of the original asset, so much of the

capital gain as bears to the whole of the capital gain the same proportion as the cost of the new asset bears to

the net consideration, shall not be charged under section 45:

Provided that nothing contained in this sub-section shall apply where—

(a) the assessee,—

(i) owns more than one residential house, other than the new asset, on the date of transfer of the original

asset; or

(ii) purchases any residential house, other than the new asset, within a period of one year after the date of

transfer of the original asset; or

(iii) constructs any residential house, other than the new asset, within a period of three years after the date

of transfer of the original asset; and

(b) the income from such residential house, other than the one residential house owned on the date of transfer of

the original asset, is chargeable under the head "Income from house property".

Explanation.—For the purposes of this section,—

"net consideration", in relation to the transfer of a capital asset, means the full value of the consideration

received or accruing as a result of the transfer of the capital asset as reduced by any expenditure incurred

wholly and exclusively in connection with such transfer.

(2) Where the assessee purchases, within the period of two years after the date of the transfer of the original asset, or

constructs, within the period of three years after such date, any residential house, the income from which is

chargeable under the head "Income from house property", other than the new asset, the amount of capital gain

arising from the transfer of the original asset not charged under section 45 on the basis of the cost of such new asset

as provided in clause (a), or, as the case may be, clause (b), of sub-section (1), shall be deemed to be income

chargeable under the head "Capital gains" relating to long-term capital assets of the previous year in which such

residential house is purchased or constructed.

(3) Where the new asset is transferred within a period of three years from the date of its purchase or, as the case may

be, its construction, the amount of capital gain arising from the transfer of the original asset not charged

under section 45 on the basis of the cost of such new asset as provided in clause (a) or, as the case may be, clause

Page 104: Capital Gains [Income Tax]

Income Tax 10.100

(b), of sub-section (1) shall be deemed to be income chargeable under the head "Capital gains" relating to long-term

capital assets of the previous year in which such new asset is transferred.

(4) The amount of the net consideration which is not appropriated by the assessee towards the purchase of the new

asset made within one year before the date on which the transfer of the original asset took place, or which is not

utilised by him for the purchase or construction of the new asset before the date of furnishing the return of income

under section 139, shall be deposited by him before furnishing such return [such deposit being made in any case not

later than the due date applicable in the case of the assessee for furnishing the return of income under sub-section (1)

of section 139] in an account in any such bank or institution as may be specified in, and utilised in accordance with,

any scheme which the Central Government may, by notification in the Official Gazette, frame in this behalf and

such return shall be accompanied by proof of such deposit; and, for the purposes of sub-section (1), the amount, if

any, already utilised by the assessee for the purchase or construction of the new asset together with the amount so

deposited shall be deemed to be the cost of the new asset :

Provided that if the amount deposited under this sub-section is not utilised wholly or partly for the purchase or

construction of the new asset within the period specified in sub-section (1), then,—

(i) the amount by which—

(a) the amount of capital gain arising from the transfer of the original asset not charged under section

45 on the basis of the cost of the new asset as provided in clause (a) or, as the case may be, clause

(b) of sub-section (1),

exceeds

(b) the amount that would not have been so charged had the amount actually utilised by the assessee for

the purchase or construction of the new asset within the period specified in sub-section (1) been the

cost of the new asset,

shall be charged under section 45 as income of the previous year in which the period of three years from the

date of the transfer of the original asset expires; and

(ii) the assessee shall be entitled to withdraw the unutilised amount in accordance with the scheme aforesaid.

Explanation.—[Omitted by the Finance Act, 1992, w.e.f. 1-4-1993.]

Exemption of capital gains on transfer of assets in cases of shifting of industrial undertaking from urban

area.

54G. (1) Subject to the provisions of sub-section (2), where the capital gain arises from the transfer of a capital

asset, being machinery or plant or building or land or any rights in building or land used for the purposes of the

business of an industrial undertaking situate in an urban area, effected in the course of, or in consequence of, the

shifting of such industrial undertaking (hereafter in this section referred to as the original asset) to any area (other

than an urban area) and the assessee has within a period of one year before or three years after the date on which the

transfer took place,—

(a) purchased new machinery or plant for the purposes of business of the industrial undertaking in the area to

which the said undertaking is shifted;

(b) acquired building or land or constructed building for the purposes of his business in the said area ;

(c) shifted the original asset and transferred the establishment of such undertaking to such area; and

(d) incurred expenses on such other purpose as may be specified in a scheme framed by the Central Government

for the purposes of this section,

then, instead of the capital gain being charged to income-tax as income of the previous year in which the transfer

took place, it shall be dealt with in accordance with the following provisions of this section, that is to say,—

(i) if the amount of the capital gain is greater than the cost and expenses incurred in relation to all or any of the

purposes mentioned in clauses (a) to (d) (such cost and expenses being hereafter in this section referred to as

Page 105: Capital Gains [Income Tax]

Capital Gains 10.101

the new asset), the difference between the amount of the capital gain and the cost of the new asset shall be

charged under section 45 as the income of the previous year; and for the purpose of computing in respect of

the new asset any capital gain arising from its transfer within a period of three years of its being purchased,

acquired, constructed or transferred, as the case may be, the cost shall be nil ; or

(ii) if the amount of the capital gain is equal to, or less than, the cost of the new asset, the capital gain shall not be

charged under section 45; and for the purpose of computing in respect of the new asset any capital gain

arising from its transfer within a period of three years of its being purchased, acquired, constructed or

transferred, as the case may be, the cost shall be reduced by the amount of the capital gain.

Explanation.—In this sub-section, "urban area" means any such area within the limits of a municipal corporation or

municipality as the Central Government may, having regard to the population, concentration of industries, need for

proper planning of the area and other relevant factors, by general or special order, declare to be an urban area for the

purposes of this sub-section.

(2) The amount of capital gain which is not appropriated by the assessee towards the cost and expenses incurred in

relation to all or any of the purposes mentioned in clauses (a) to (d) of sub-section (1) within one year before the

date on which the transfer of the original asset took place, or which is not utilised by him for all or any of the

purposes aforesaid before the date of furnishing the return of income under section 139, shall be deposited by him

before furnishing such return [such deposit being made in any case not later than the due date applicable in the case

of the assessee for furnishing the return of income under sub-section (1) of section 139] in an account in any such

bank or institution as may be specified in, and utilised in accordance with, any scheme which the Central

Government may, by notification in the Official Gazette, frame in this behalf and such return shall be accompanied

by proof of such deposit ; and, for the purposes of sub-section (1), the amount, if any, already utilised by the

assessee for all or any of the purposes aforesaid together with the amount, so deposited shall be deemed to be the

cost of the new asset :

Provided that if the amount deposited under this sub-section is not utilised wholly or partly for all or any of the

purposes mentioned in clauses (a) to (d) of sub-section (1) within the period specified in that sub-section, then,—

(i) the amount not so utilised shall be charged under section 45 as the income of the previous year in which the

period of three years from the date of the transfer of the original asset expires ; and

(ii) the assessee shall be entitled to withdraw such amount in accordance with the scheme aforesaid.

Explanation.—[Omitted by the Finance Act, 1992, w.e.f. 1-4-1993.]

Exemption of capital gains on transfer of assets in cases of shifting of industrial undertaking from urban area

to any Special Economic Zone.

54GA. (1) Notwithstanding anything contained in section 54G, where the capital gain arises from the transfer of a

capital asset, being machinery or plant or building or land or any rights in building or land used for the purposes of

the business of an industrial undertaking situate in an urban area, effected in the course of, or in consequence of the

shifting of such industrial undertaking to any Special Economic Zone, whether developed in any urban area or any

other area and the assessee has within a period of one year before or three years after the date on which the transfer

took place,—

(a) purchased machinery or plant for the purposes of business of the industrial undertaking in the Special

Economic Zone to which the said undertaking is shifted;

(b) acquired building or land or constructed building for the purposes of his business in the Special Economic

Zone;

(c) shifted the original asset and transferred the establishment of such undertaking to the Special Economic Zone;

and

Page 106: Capital Gains [Income Tax]

Income Tax 10.102

(d) incurred expenses on such other purposes as may be specified in a scheme framed by the Central Government

for the purposes of this section,

then, instead of the capital gain being charged to income-tax as income of the previous year in which the transfer

took place, it shall, subject to the provisions of sub-section (2), be dealt with in accordance with the following

provisions of this section, that is to say,—

(i) if the amount of the capital gain is greater than the cost and expenses incurred in relation to all or any of the

purposes mentioned in clauses (a) to (d) (such cost and expenses being hereafter in this section referred to as

the new asset), the difference between the amount of the capital gain and the cost of the new asset shall be

charged under section 45 as the income of the previous year; and for the purpose of computing in respect of

the new asset any capital gain arising from its transfer within a period of three years of its being purchased,

acquired, constructed or transferred, as the case may be, the cost shall be Nil; or

(ii) if the amount of the capital gain is equal to, or less than, the cost of the new asset, the capital gain shall not be

charged under section 45, and for the purpose of computing in respect of the new asset any capital gain

arising from its transfer within a period of three years of its being purchased, acquired, constructed or

transferred, as the case may be, the cost shall be reduced by the amount of the capital gain.

Explanation.—In this sub-section,—

(a) "Special Economic Zone" shall have the meaning assigned to it in clause (za) of *[section 2 of] the Special

Economic Zones Act, 2005;

(b) "urban area" means any such area within the limits of a municipal corporation or municipality as the Central

Government may, having regard to the population, concentration of industries, need for proper planning of

the area and other relevant factors, by general or special order, declare to be an urban area for the purposes of

this sub-section.

(2) The amount of capital gain which is not appropriated by the assessee towards the cost and expenses incurred in

relation to all or any of the purposes mentioned in clauses (a) to (d) of sub-section (1) within one year before the

date on which the transfer of the original asset took place, or which is not utilised by him for all or any of the

purposes aforesaid before the date of furnishing the return of income under section 139, shall be deposited by him

before furnishing such return [such deposit being made in any case not later than the due date applicable in the case

of the assessee for furnishing the return of income under sub-section (1) of section 139] in an account in any such

bank or institution as may be specified in, and utilised in accordance with, any scheme which the Central

Government may, by notification, frame in this behalf and such return shall be accompanied by proof of such

deposit; and, for the purposes of sub-section (1), the amount, if any, already utilised by the assessee for all or any of

the aforesaid purposes together with the amount so deposited shall be deemed to be the cost of the new asset :

Provided that if the amount deposited under this sub-section is not utilised wholly or partly for all or any of the

purposes mentioned in clauses (a) to (d) of sub-section (1) within the period specified in that sub-section, then,—

(i) the amount not so utilised shall be charged under section 45 as the income of the previous year in which the

period of three years from the date of the transfer of the original asset expires; and

(ii) the assessee shall be entitled to withdraw such amount in accordance with the scheme aforesaid.

Capital gain on transfer of residential property not to be charged in certain cases

54GB. (1) Where,—

(i) the capital gain arises from the transfer of a long-term capital asset, being a residential property (a house or a

plot of land), owned by the eligible assessee (herein referred to as the assessee); and

(ii) the assessee, before the due date of furnishing of return of income under sub-section (1) of section 139,

utilises the net consideration for subscription in the equity shares of an eligible company (herein-referred to

as the company); and

Page 107: Capital Gains [Income Tax]

Capital Gains 10.103

(iii) the company has, within one year from the date of subscription in equity shares by the assessee, utilised this

amount for purchase of new asset,

then, instead of the capital gain being charged to income-tax as the income of the previous year in which the transfer

takes place, it shall be dealt with in accordance with the following provisions of this section, that is to say,—

(a) if the amount of the net consideration is greater than the cost of the new asset, then, so much of the capital

gain as it bears to the whole of the capital gain the same proportion as the cost of the new asset bears to the

net consideration, shall not be charged under section 45 as the income of the previous year; or

(b) if the amount of the net consideration is equal to or less than the cost of the new asset, the capital gain shall

not be charged under section 45 as the income of the previous year.

(2) The amount of the net consideration, which has been received by the company for issue of shares to the assessee,

to the extent it is not utilised by the company for the purchase of the new asset before the due date of furnishing of

the return of income by the assessee under section 139, shall be deposited by the company, before the said due date

in an account in any such bank or institution as may be specified and shall be utilised in accordance with any scheme

which the Central Government may, by notification in the Official Gazette, frame in this behalf and the return

furnished by the assessee shall be accompanied by proof of such deposit having been made.

(3) For the purposes of sub-section (1), the amount, if any, already utilised by the company for the purchase of the

new asset together with the amount deposited under sub-section (2) shall be deemed to be the cost of the new asset:

Provided that if the amount so deposited is not utilised, wholly or partly, for the purchase of the new asset within

the period specified in sub-section (1), then,—

(i) the amount by which—

(a) the amount of capital gain arising from the transfer of the residential property not charged

under section 45 on the basis of the cost of the new asset as provided in sub-section (1),

exceeds—

(b) the amount that would not have been so charged had the amount actually utilised for the purchase of

the new asset within the period specified in sub-section (1) been the cost of the new asset,

shall be charged under section 45 as income of the assessee for the previous year in which the period of one

year from the date of the subscription in equity shares by the assessee expires; and

(ii) the company shall be entitled to withdraw such amount in accordance with the scheme.

(4) If the equity shares of the company or the new asset acquired by the company are sold or otherwise transferred

within a period of five years from the date of their acquisition, the amount of capital gain arising from the transfer of

the residential property not charged under section 45 as provided in sub-section (1) shall be deemed to be the income

of the assessee chargeable under the head "Capital gains" of the previous year in which such equity shares or such

new asset are sold or otherwise transferred, in addition to taxability of gains, arising on account of transfer of shares

or of the new asset, in the hands of the assessee or the company, as the case may be.

(5) The provisions of this section shall not apply to any transfer of residential property made after the 31st day of

March, 2017.

(6) For the purposes of this section,—

(a) "eligible assessee" means an individual or a Hindu undivided family;

(b) "eligible company" means a company which fulfils the following conditions, namely:—

(i) it is a company incorporated in India during the period from the 1st day of April of the previous year

relevant to the assessment year in which the capital gain arises to the due date of furnishing of return

of income under sub-section (1) of section 139 by the assessee;

(ii) it is engaged in the business of manufacture of an article or a thing;

Page 108: Capital Gains [Income Tax]

Income Tax 10.104

(iii) it is a company in which the assessee has more than fifty per cent share capital or more than fifty per

cent voting rights after the subscription in shares by the assessee; and

(iv) it is a company which qualifies to be a small or medium enterprise under the Micro, Small and

Medium Enterprises Act, 2006 (27 of 2006);

(c) "net consideration" shall have the meaning assigned to it in the Explanation to section 54F;

(d) "new asset" means new plant and machinery but does not include—

(i) any machinery or plant which, before its installation by the assessee, was used either within or outside

India by any other person;

(ii) any machinery or plant installed in any office premises or any residential accommodation, including

accommodation in the nature of a guest-house;

(iii) any office appliances including computers or computer software;

(iv) any vehicle; or

(v) any machinery or plant, the whole of the actual cost of which is allowed as a deduction (whether by

way of depreciation or otherwise) in computing the income chargeable under the head "Profits and

gains of business or profession" of any previous year.

Extension of time for acquiring new asset or depositing or investing amount of capital gain.

54H. Notwithstanding anything contained in sections 54, 54B, 54D, 54EC and 54F, where the transfer of the

original asset is by way of compulsory acquisition under any law and the amount of compensation awarded for such

acquisition is not received by the assessee on the date of such transfer, the period for acquiring the new asset by the

assessee referred to in those sections or, as the case may be, the period available to the assessee under those sections

for depositing or investing the amount of capital gain in relation to such compensation as is not received on the date

of the transfer, shall be reckoned from the date of receipt of such compensation :

Provided that where the compensation in respect of transfer of the original asset by way of compulsory acquisition

under any law is received before the 1st day of April, 1991, the aforesaid period or periods, if expired, shall extend

up to the 31st day of December, 1991.

Meaning of "adjusted", "cost of improvement" and "cost of acquisition".

55. (1) For the purposes of sections 48 and 49,—

(a) [***]

(b) "cost of any improvement",—

(1) in relation to a capital asset being goodwill of a business or a right to manufacture, produce or process

any article or thing or right to carry on any business shall be taken to be nil ; and

(2) in relation to any other capital asset,—

(i) where the capital asset became the property of the previous owner or the assessee before the

1st day of April, 1981, means all expenditure of a capital nature incurred in making any

additions or alterations to the capital asset on or after the said date by the previous owner or

the assessee, and

(ii) in any other case, means all expenditure of a capital nature incurred in making any additions

or alterations to the capital asset by the assessee after it became his property, and, where the

capital asset became the property of the assessee by any of the modes specified in sub-section

(1) of section 49, by the previous owner,

but does not include any expenditure which is deductible in computing the income chargeable under

the head "Interest on securities", "Income from house property", "Profits and gains of business or

Page 109: Capital Gains [Income Tax]

Capital Gains 10.105

profession", or "Income from other sources", and the expression "improvement" shall be construed

accordingly.

(2) For the purposes of sections 48 and 49, "cost of acquisition",—

(a) in relation to a capital asset, being goodwill of a business or a trade mark or brand name associated

with a business or a right to manufacture, produce or process any article or thing or right to carry on

any business, tenancy rights, stage carriage permits or loom hours,—

(i) in the case of acquisition of such asset by the assessee by purchase from a previous owner,

means the amount of the purchase price ; and

(ii) in any other case [not being a case falling under sub-clauses (i) to (iv) of sub-section (1)

of section 49], shall be taken to benil ;

(aa) in a case where, by virtue of holding a capital asset, being a share or any other security, within the

meaning of clause (h) of section 2 of the Securities Contracts (Regulation) Act, 1956 (42 of 1956)

(hereafter in this clause referred to as the financial asset), the assessee—

(A) becomes entitled to subscribe to any additional financial asset ; or

(B) is allotted any additional financial asset without any payment,

then, subject to the provisions of sub-clauses (i) and (ii) of clause (b),—

(i) in relation to the original financial asset, on the basis of which the assessee becomes entitled to

any additional financial asset, means the amount actually paid for acquiring the original

financial asset ;

(ii) in relation to any right to renounce the said entitlement to subscribe to the financial asset,

when such right is renounced by the assessee in favour of any person, shall be taken to

be nil in the case of such assessee ;

(iii) in relation to the financial asset, to which the assessee has subscribed on the basis of the said

entitlement, means the amount actually paid by him for acquiring such asset ;

(iiia) in relation to the financial asset allotted to the assessee without any payment and on the basis

of holding of any other financial asset, shall be taken to be nil in the case of such assessee ;

and

(iv) in relation to any financial asset purchased by any person in whose favour the right to

subscribe to such asset has been renounced, means the aggregate of the amount of the

purchase price paid by him to the person renouncing such right and the amount paid by him to

the company or institution, as the case may be, for acquiring such financial asset ;

(ab) in relation to a capital asset, being equity share or shares allotted to a shareholder of a recognised

stock exchange in India under a scheme for demutualisation or corporatisation approved by the

Securities and Exchange Board of India established under section 3 of the Securities and Exchange

Board of India Act, 1992 (15 of 1992), shall be the cost of acquisition of his original membership of

the exchange:

Provided that the cost of a capital asset, being trading or clearing rights of the recognised stock

exchange acquired by a shareholder who has been allotted equity share or shares under such scheme

of demutualisation or corporatisation, shall be deemed to be nil;

(b) in relation to any other capital asset,—

(i) where the capital asset became the property of the assessee before the 1st day of April, 1981,

means the cost of acquisition of the asset to the assessee or the fair market value of the asset

on the 1st day of April, 1981, at the option of the assessee ;

(ii) where the capital asset became the property of the assessee by any of the modes specified in

sub-section (1) of section 49, and the capital asset became the property of the previous owner

Page 110: Capital Gains [Income Tax]

Income Tax 10.106

before the 1st day of April, 1981, means the cost of the capital asset to the previous owner or

the fair market value of the asset on the 1st day of April, 1981, at the option of the assessee ;

(iii) where the capital asset became the property of the assessee on the distribution of the capital

assets of a company on its liquidation and the assessee has been assessed to income-tax under

the head "Capital gains" in respect of that asset undersection 46, means the fair market value

of the asset on the date of distribution ;

(iv) [***]

(v) where the capital asset, being a share or a stock of a company, became the property of the

assessee on—

(a) the consolidation and division of all or any of the share capital of the company into

shares of larger amount than its existing shares,

(b) the conversion of any shares of the company into stock,

(c) the re-conversion of any stock of the company into shares,

(d) the sub-division of any of the shares of the company into shares of smaller amount, or

(e) the conversion of one kind of shares of the company into another kind,

means the cost of acquisition of the asset calculated with reference to the cost of acquisition

of the shares or stock from which such asset is derived.

(3) Where the cost for which the previous owner acquired the property cannot be ascertained, the cost of acquisition

to the previous owner means the fair market value on the date on which the capital asset became the property of the

previous owner.

Reference to Valuation Officer.

55A. With a view to ascertaining the fair market value of a capital asset for the purposes of this Chapter, the

Assessing Officer may refer the valuation of capital asset to a Valuation Officer—

(a) in a case where the value of the asset as claimed by the assessee is in accordance with the estimate made by a

registered valuer, if the Assessing Officer is of opinion that the value so claimed is at variance with its fair

market value;

(b) in any other case, if the Assessing Officer is of opinion—

(i) that the fair market value of the asset exceeds the value of the asset as claimed by the assessee by

more than such percentage6 of the value of the asset as so claimed or by more than such amount

6 as

may be prescribed in this behalf ; or

(ii) that having regard to the nature of the asset and other relevant circumstances, it is necessary so to do,

7and where any such reference is made, the provisions of sub-sections (2), (3), (4), (5) and (6) of section 16A,

clauses (ha) and (i) of sub-section (1) and sub-sections (3A) and (4) of section 23, sub-section (5) of section 24,

section 34AA, section 35 and section 37 of the Wealth-tax Act, 1957 (27 of 1957), shall with the necessary

modifications, apply in relation to such reference as they apply in relation to a reference made by the Assessing

Officer under sub-section (1) of section 16A of that Act.

Explanation.—In this section, "Valuation Officer" has the same meaning, as in clause (r) of section 2 of the Wealth-

tax Act, 1957 (27 of 1957).